0% found this document useful (0 votes)
31 views

PreCalculus - 03 - Coordinate Geometry

Uploaded by

c singh
Copyright
© © All Rights Reserved
We take content rights seriously. If you suspect this is your content, claim it here.
Available Formats
Download as PDF, TXT or read online on Scribd
0% found this document useful (0 votes)
31 views

PreCalculus - 03 - Coordinate Geometry

Uploaded by

c singh
Copyright
© © All Rights Reserved
We take content rights seriously. If you suspect this is your content, claim it here.
Available Formats
Download as PDF, TXT or read online on Scribd
You are on page 1/ 95

COORDINATE GEOMETRY

22 JANUARY 2022
REVISION: 1804

AZIZ MANVA
[email protected]

ALL RIGHTS RESERVED


Get all the files at: https://bit.ly/azizhandouts
Aziz Manva ([email protected])

TABLE OF CONTENTS
TABLE OF CONTENTS ................................. 2 2. LINES ..................................................... 52
2.1 Basics 52
Part I: Coordinate Geometry 3 2.2 Slope Intercept Form 61
2.3 Other Forms 69
1. BASICS ...................................................... 3 2.4 Multiple Lines 73
1.1 Co-ordinate Plane 3
1.2 Translations 13
3. CONICS ................................................... 76
1.3 Reflection 16 3.1 Conic Sections and Circles 76
1.4 Dilations 21 3.2 Ellipse 81
1.5 Rotations 22 3.3 Parabola 83
1.6 Symmetry and Combining Transformations 26 3.4 Hyperbola 83
1.7 Distance Formula 28
1.8 Midpoint and Section Formula 44 4. 3D GEOMETRY ..................................... 84
1.9 Triangles 48 4.1 Lines and Planes 84
1.10 Area 50 4.2 AMC Questions 86

P a g e 2 | 95
Get all the files at: https://bit.ly/azizhandouts
Aziz Manva ([email protected])

PART I: COORDINATE GEOMETRY


1. BASICS
1.1 Co-ordinate Plane
A. Introduction
Co-ordinate geometry was introduced by Descartes, who was the first mathematician to put the connection
between geometry and algebra on firm ground. In his honor, the coordinate plane is called the Cartesian
coordinate plane (and points on this plane are called Cartesian coordinates).

He introduced co-ordinate axes. The x-axis is horizontal, and the y-axis is vertical. The axes are perpendicular
to each other.

Location of Coordinates as an Ordered Pair


Every point in the co-ordinate plane is uniquely determined by its 𝑥 coordinate and its 𝑦 coordinate. The
combination of the two coordinates is called an ordered pair written like this
(𝑥, 𝑦)
For example, (1,3) is an ordered pair consisting of 𝑥 co-ordinate 1, and 𝑦 coordinate 3.
Because (1,3) is an ordered pair, it is not the same as (3,1).

If we are referring to multiple points, we will generally name them using subscripts. For example
𝐴 = (𝑥1 , 𝑦1 ), 𝐵 = (𝑥2 , 𝑦2 )
Capital letters are generally used to refer to points.

B. Finding the axes

Example 1.1
In the graph alongside:
A. Identify the 𝑥-axis and the 𝑦-axis.
B. Identify the origin
C. Write the coordinates of the origin.
x-axis and y-axis
The blue, horizontal line is the 𝑥-axis. The red, vertical line is the 𝑦-axis.
Origin
The origin is the intersection of the blue and the red lines. In other words, it is the intersection of the 𝑥-axis and
the 𝑦-axis.
Coordinates of the Origin
At the origin
➢ the x coordinate is zero.
➢ the y coordinate is also zero.
Hence, the coordinates of the origin are
(0,0)
Example 1.2
Write the coordinates of the point graphed alongside.

P a g e 3 | 95
Get all the files at: https://bit.ly/azizhandouts
Aziz Manva ([email protected])

Example 1.3

Example 1.4

Example 1.5
In the diagram shown, what are the coordinates of point P? (CEMC Gauss 7 2020/2)

Example 1.6
In the diagram, point F has coordinates (5, 5). The point with coordinates (2, 4) is
located at (CEMC Gauss 7 2019/2)

Example 1.7
Micheal has made a diagram showing the location of the library and the school.
Michael’s house is located at the origin.
A. Find the coordinates of Michael’s house.
B. Find the coordinates of the library and the school. The library is to the

P a g e 4 | 95
Get all the files at: https://bit.ly/azizhandouts
Aziz Manva ([email protected])

left of the school and higher than the school.

C. Distance Between Coordinates


Consider the graph alongside. We cannot see the x coordinate, but it is the same for both the
points.

The first point has a y coordinate of 3. The


second point has a y coordinate of 7.
The distance between the two point is
7 − 3 = 4 𝑈𝑛𝑖𝑡𝑠
Example 1.8
Find the distance between the two points graphed.

D. Stating Coordinates

Example 1.9: Stating Co-ordinates


State the 𝑥 coordinate and the 𝑦 coordinate of the following points:
1 8
(2,3), (−4,3), (− , − )
2 9

Example 1.10: Writing Co-ordinates


Write the coordinates of the point that has:
A. 𝑥 coordinate 5, and 𝑦 coordinate 7
2 5
B. 𝑦 coordinate 3, and 𝑥 coordinate − 4

(5,7)

Example 1.11: Ordered Pairs


State, with reasons, which points represent the same location
12 27 −8 −81 27 12 12 27
𝐴 = ( , ), 𝐵=( , ), 𝐶 = ( , ), 𝐷 = (− ,− )
6 9 −4 −27 9 6 6 9

We simplify the fractions and get:


𝐴 = (2,3), 𝐵 = (2, −3), 𝐶 = (3,2), 𝐷 = (−2, −3)
None of the points have both the same x coordinate and the same y coordinate.

P a g e 5 | 95
Get all the files at: https://bit.ly/azizhandouts
Aziz Manva ([email protected])

Hence, none of the points represent the same location.


E. Quadrants
The coordinate axes divide the coordinate plane into
four quadrants.

Note that the numbers of the quadrants are written


using Roman Numerals. This is, by convention, and
helps ensure that the numbers related to coordinates
are not mixed up with number related to Quadrants.

Example 1.12: Finding Quadrants of Points


Find the Quadrants in which the following points lie:
A. (3,4)
B. (−2, −5)
C. (−3,2)
D. (−4, −6)
23 87
E. (17 , 16)
62 53
F. (41 , − 41)

Quadrant II Quadrant I
𝑥 < 0, 𝑦 > 0 𝑥 > 0, 𝑦 > 0
(−𝟑, 𝟐) (𝟑, 𝟒)
23 87
( , )
17 16
Quadrant III Quadrant IV
𝑥 < 0, 𝑦 < 0 𝑥 > 0, 𝑦 < 0

(−𝟐, −𝟓) 𝟔𝟐 𝟓𝟑
(, −4, −6) ( ,− )
𝟒𝟏 𝟒𝟏

Example 1.13
Give an example of four points, each in a different quadrant. Mention the quadrant that the point lies in.

Example 1.14
In the graph shown, which of the following statements is true about the coordinates
of the point P(x, y)?
A. The values of both x and y are positive.
B. The value of x is positive and the value of y is negative.
C. The value of x is negative and the value of y is positive.
D. The values of both x and y are negative.
E. The value of x is 0 and the value of y is negative. (CEMC Gauss 7 2020/7)

Option B.

Example 1.15
In the diagram, the coordinates of point P could be (CEMC Gauss 7 2009/5)

P a g e 6 | 95
Get all the files at: https://bit.ly/azizhandouts
Aziz Manva ([email protected])

A. (1,3)
B. (1,-3)
C. (-3,1)
D. (3,-1)
E. (-1,3)

F. Coordinates of the axes


Throughout the 𝑥-axis, the 𝑦 coordinate is zero. Therefore, any point on the 𝑥-axis must have coordinates
(𝑥, 0)
for some value of 𝑥.

Throughout the 𝑦-axis, the 𝑥 coordinate is zero. Therefore, similar to the above, any point on the 𝑦-axis must
have coordinates
(0, 𝑦)
for some value of 𝑦.

This fact is very useful when finding the places where a curve will cut the axes. These are called 𝑥-intercepts
(where a curve cuts the x-axis), and 𝑦-intercepts (where a curve cuts the y-axis).

Example 1.16: Identifying axes of coordinates


State which axes do the following points lie on:
A. (0,4)
B. (3,0)
C. (0, −2)
D. (−7,0)

(0,4) = 𝑦 − 𝑎𝑥𝑖𝑠
(3,0) = 𝑥 − 𝑎𝑥𝑖𝑠
(0, −2) = 𝑦 − 𝑎𝑥𝑖𝑠
(−7,0) = 𝑥 − 𝑎𝑥𝑖𝑠
G. Axes versus Quadrants
A point which lies on an axis does not lie in any quadrant.

Example 1.17: Quadrants and Axes


State the quadrant or the axis that the following points lie on.
A. (2,3)
B. (0,4)
C. (−5,7)
D. (−2,0)
E. (3, −2)
F. (−3, −9)

H. Origin
Suppose a point 𝑂 lies on both the x-axis and the y-axis.
Since it lies on the x-axis
➢ it must have y-coordinate zero

P a g e 7 | 95
Get all the files at: https://bit.ly/azizhandouts
Aziz Manva ([email protected])

Since it lies on the y-axis


➢ it must have x-coordinate zero

Therefore, both the 𝑥 coordinate and the 𝑦 coordinate of the point are zero.
𝑂 = (0,0)
This point, in fact, is called the origin, and is the intersection of the x and the y-axis.

The letter 𝑂 is commonly used to represent the origin.

I. Distance between two points with one coordinate same


An important thing to remember is that distance is never negative. Hence, when defining the distance below,
we use the absolute value function to remove the negative sign, if it exists.
If one of the coordinates is the same, then the distance is simply the absolute value of the difference of the other
coordinate.
𝑉𝑒𝑟𝑡𝑖𝑐𝑎𝑙 𝑑𝑖𝑠𝑡𝑎𝑛𝑐𝑒 𝑏𝑒𝑡𝑤𝑒𝑒𝑛 (𝑥1 , 𝑦1 ) 𝑎𝑛𝑑 (𝑥1 , 𝑦2 ) = |𝑦2 − 𝑦1 |
𝐷𝑖𝑠𝑡𝑎𝑛𝑐𝑒 𝑏𝑒𝑡𝑤𝑒𝑒𝑛 (𝑥1 , 𝑦1 ) 𝑎𝑛𝑑 (𝑥2 , 𝑦1 ) = |𝑥2 − 𝑥1 |

Example 1.18: Finding Distance


Find the distance between the following pairs of points:
A. 𝐴 = (3,4) and 𝐵 = (3,7)
B. 𝑃 = (2,5) and 𝑄 = (8,5)

𝐴𝐵 = |7 − 4| = 3 = |4 − 7|, 𝑃𝑄 = |2 − 8| = |8 − 2| = 6

Example 1.19
In \DeltaPQR shown, side PR is horizontal and side PQ is vertical. The coordinates
of P are (CEMC Gauss 8 2016/6)

Example 1.20
In the diagram, which of the following is the largest? (CEMC Gauss 8 2004/20)
A. AE
B. CD+CF
C. AC+CF
D. FD
E. AC+CE
J. 3D Coor dinate System
The (𝑥, 𝑦) coordinate in two dimensions can be extended to three dimensions by adding a 𝑧 variable. The
concepts that we have learnt in two dimensions can usually be extended in a straight forward manner to three
dimensions.

The axes divide 3D dimensional coordinate space into eight octants.

Example 1.21
A. Identify the x coordinate, the y coordinate, and the 𝑧 coordinate in the point (7, 𝜋, 𝑒).

P a g e 8 | 95
Get all the files at: https://bit.ly/azizhandouts
Aziz Manva ([email protected])

B. Is the point (7, 𝜋, 𝑒) the same as the point (𝜋, 𝑒, 7)?


C. Find the distance between the following pairs of points (1,2,3) and (1,2,5).
D. Find the distance between the following pairs of points (𝑥1 , 𝑦1 , 𝑧1 ) and (𝑥1 , 𝑦2 , 𝑧1 ).

Part A
𝑥 = 7, 𝑦 = 𝜋, 𝑧 = 𝑒
Part B
These two points are not the same.
Coordinates are stated as ordered triplets, and the order matters.
Part C
Since the 𝑥 coordinate and the 𝑦 coordinate of both the points are the same, we can find the distance by finding
the absolute value of the difference between the z coordinates
𝐷𝑖𝑠𝑡𝑎𝑛𝑐𝑒 = |3 − 5| = |−2| = 2
Part D
𝐷𝑖𝑠𝑡𝑎𝑛𝑐𝑒 = |𝑦1 − 𝑦2 |

K. Counting

Example 1.22
In the diagram, the points are evenly spaced vertically and horizontally. A segment AB is
drawn using two of the points, as shown. Point C is chosen to be one of the remaining 18
points. For how many of these 18 possible points is triangle ABC isosceles? (CEMC
Gauss 8 2006/23)

Example 1.23
Consider the rectangle with vertices
𝑃 = (3,6), 𝑄 = (9,6), 𝑅 = (9,2), 𝑆 = (3,2)
A. What is the number of points inside 𝑃𝑄𝑅𝑆?
B. What is the number of lattice points that lie on or inside 𝑃𝑄𝑅𝑆?
C. What is the number of lattice points that lie on 𝑃𝑄𝑅𝑆?
D. What is the number of lattice points that lie inside 𝑃𝑄𝑅𝑆?

Part A
𝐼𝑛𝑓𝑖𝑛𝑖𝑡𝑒
Part B
The lattice points that lie along the length of 𝑃𝑄𝑅𝑆 are all of
the form:
(3, 𝑦), (4, 𝑦), … , (9, 𝑦)
The number of lattice points is the number of elements in the
set:
{3,4, … ,9} = 9 − 3 + 1 = 7 𝑃𝑜𝑖𝑛𝑡𝑠

Similarly, the lattice points that lie along the width of 𝑃𝑄𝑅𝑆
are all of the form:
(𝑥, 2), (𝑥, 3), … (𝑥, 6) ⇒ {2,3, … ,6} ⇒ 6 − 2 + 1 = 5

The total number of points

P a g e 9 | 95
Get all the files at: https://bit.ly/azizhandouts
Aziz Manva ([email protected])


7 × ⏟
5 = 35
𝐿𝑒𝑛𝑔𝑡ℎ 𝑊𝑖𝑑𝑡ℎ
Part C
Number of Lattice Points

7 + ⏟
5 +⏟
7+⏟
5 = 24
𝑃𝑄 𝑄𝑅 𝑅𝑆 𝑆𝑃
But this method of counting counts the vertices twice. Hence, the vertices need to be subtracted to get the final
answer:
= 24 − 4 = 20

Another way to do it is to recognize that, in this case, the number of lattice points on the polygon is equal to the
perimeter of the polygon
= 20
Part D
We use complementary counting:
35
⏟ − 20 ⏟ = 15
𝑇𝑜𝑡𝑎𝑙 𝑃𝑜𝑖𝑛𝑡𝑠
𝑃𝑜𝑖𝑛𝑡𝑠 𝑜𝑛 𝑆𝑖𝑑𝑒

Example 1.24
What is the number of lattice points that lie on the cuboid 𝐴𝐵𝐶𝐷𝐸𝐹𝐺𝐻 with vertices:
𝐴 = (1,1,1), 𝐵 = (1,1,9), 𝐶 = (7,1,1), 𝐷 = (7,1,9), 𝐸 = (1,8,1), 𝐹 = (1,8,9), 𝐺 = (7,8,1), 𝐻 = (7,8,9)

The total number of points that lie on or inside the cuboid


= ⏟7 × ⏟
8 × ⏟
9 = 504
𝑥 𝑑𝑖𝑟𝑒𝑐𝑡𝑖𝑜𝑛 𝑦 𝑑𝑖𝑟𝑒𝑐𝑡𝑖𝑜𝑛 𝑧 𝑑𝑖𝑟𝑒𝑐𝑡𝑖𝑜𝑛

The total number of points that lie inside the cuboid


= ⏟ 7−2 × ⏟ 8−2 × 9−2
⏟ = 5 × 6 × 7 = 210
𝑥 𝑑𝑖𝑟𝑒𝑐𝑡𝑖𝑜𝑛 𝑦 𝑑𝑖𝑟𝑒𝑐𝑡𝑖𝑜𝑛 𝑧 𝑑𝑖𝑟𝑒𝑐𝑡𝑖𝑜𝑛

Using complementary counting


504 − 210 = 294

Example 1.25
Find the number of lattice points that lie within a circle of area 25𝜋 with center at the origin.

𝐴 = 25𝜋 ⇒ 𝜋𝑟 2 = 25𝜋 ⇒ 𝑟 2 = 25 ⇒ 𝑟 = 5

The origin is a lattice point within the circle.


1 𝑃𝑜𝑖𝑛𝑡

The number of points on the 𝑥 − 𝑎𝑥𝑖𝑠 and to the right of the origin is
4 𝑃𝑜𝑖𝑛𝑡𝑠
The distance from (4,1) to the origin is:
√(4 − 0)2 + (1 − 0)2 = √16 + 1 = √17 < √25

The distance from (4,2) to the origin is:


√42 + 22 = √16 + 4 = √20 < √25
The distance from (4,3) to the origin is:

P a g e 10 | 95
Get all the files at: https://bit.ly/azizhandouts
Aziz Manva ([email protected])

√42 + 32 = √16 + 9 = √25 ⇒ (4,3) 𝑙𝑖𝑒𝑠 𝑜𝑛 𝑡ℎ𝑒 𝑐𝑖𝑟𝑐𝑙𝑒

The distance from (3,4) to the origin is:


√32 + 42 = √9 + 16 = √25 ⇒ (3,4) 𝑙𝑖𝑒𝑠 𝑜𝑛 𝑡ℎ𝑒 𝑐𝑖𝑟𝑐𝑙𝑒

4(4 + 4 + 4 + 3 + 2) + 1 = 4 × 17 + 1 = 68 + 1 = 69

L. Probability

Example 1.26
A 10 by 10 grid is created using 100 points, as shown. Point 𝑃 is given. One
of the other 99 points is randomly chosen to be 𝑄. What is the probability
that the line segment 𝑃𝑄 is vertical or horizontal? (CEMC Gauss 7 2016/21)

Mark the dots that make PQ vertical with a blue vertical box. There are 10
points in the column, but we can’t include P itself, so have
10 − 1 = 9 𝑃𝑜𝑖𝑛𝑡𝑠
Mark the dots that make PQ horizontal with a blue horizontal box. There are
10 points in the row, but we can’t include P itself, so again we have
10 − 1 = 9 𝑃𝑜𝑖𝑛𝑡𝑠

Then, the required probability is


𝑆𝑢𝑐𝑒𝑠𝑠𝑓𝑢𝑙 𝑂𝑢𝑡𝑐𝑜𝑚𝑒𝑠 9 + 9 18 2
= = =
𝑇𝑜𝑡𝑎𝑙 𝑂𝑢𝑡𝑐𝑜𝑚𝑒𝑠 99 99 11

(Continuation) Example 1.27


What is the probability that the line segment 𝑃𝑄 is diagonal? (Diagonal in this case means that for every unit
that it goes right, it also goes up or down 𝑒𝑥𝑎𝑐𝑡𝑙𝑦 1 unit).

13
99

Example 1.28
I pick a random lattice point from the points that lie on or inside the square with non-adjacent vertices
(0,0), (5,5). Then I pick a second lattice point at random (with the same conditions as before). If the second
point is the same as the first, then I repick until the second point is different.
A. What is the number of lattice points in the given region?
B. What is the probability that I need to repick?
C. What is the number of ways to choose two distinct points from the given region?
D. What is the probability that the points lie on a line that is horizontal or vertical?
E. What is the probability that the points lie on a line that is at a 45° angle with the 𝑥-axis or a 45° angle
with the 𝑦-axis?
F. What is the probability that the points lie on a line that makes an angle of 𝑎° with the 𝑥-axis or the 𝑦-
axis, 𝑎 ≠ 45𝑛, 𝑛 ∈ ℤ?

P a g e 11 | 95
Get all the files at: https://bit.ly/azizhandouts
Aziz Manva ([email protected])

Part A
The number of lattice points will be
6 × 6 = 36
Part B
We can pick any point that we want for the first point. We will need to repick only if
the second point is the same as the first point, which will happen with probability
1
𝑃=
36
Part C
The number of ways to choose two points from the 36 points in the region is:
36 36 × 35
( )= = 18 × 35 = 630
2 2
Part D
Number of Horizontal Lines
=6
Number of Ways for two points to be on a single Horizontal Line
6 6×5
=( )= = 15
2 2
Total Number of Ways
= 6 × 15 = 90
By the same logic, since the width of the region is equal to the height of the
region, the number of ways for the two points to lie on a vertical line is also
90
The probability of the two points lying on a line that is either horizontal or vertical is
90 + 90 180 18 2
𝑃= = = =
630 630 63 7
Part D: Shortcut Method
Choose any point as the first point. For instance, pick the red dot in the diagram
alongside.
Pick the second point. The points that result in the line being formed as vertical
or horizontal are colored green.
10 2
𝑃= =
35 7
But, this condition is true irrespective of whichever point you pick in the
diagram. Hence, it is also the overall probability.

Part E
The number of ways to get choose a line that goes up from left to right is:
2(1 + 3 + 6 + 10) + 15 = 2(20) + 15 = 40 + 15 = 55

By symmetry, the number of ways to get a line that goes down from left to
right is also
55

Total Number of Ways to choose two points such the points lie on a lie
that is 45° to either the 𝑥-axis or the 𝑦-axis is
2 × 55 = 110

Hence, the required probability is:

P a g e 12 | 95
Get all the files at: https://bit.ly/azizhandouts
Aziz Manva ([email protected])

𝑆𝑢𝑐𝑐𝑒𝑠𝑠𝑓𝑢𝑙 𝑂𝑢𝑡𝑐𝑜𝑚𝑒𝑠 110 11


= =
𝑇𝑜𝑡𝑎𝑙 𝑂𝑢𝑡𝑐𝑜𝑚𝑒𝑠 630 63
Part F
We use complementary counting.
4 11 12 11 23 40
𝑃 =1− − =1−( + )=1− =
21 63 63 63 63 63

1.2 Translations
A. Translations

Example 1.29
Zeus starts at the origin (0, 0) and can make repeated moves of one unit either up, down, left or right, but
cannot make a move in the same direction twice in a row. For example, he cannot move from (0, 0) to (1,0) to
(2,0). What is the smallest number of moves that he can make to get to the point (1056, 1007)? (CEMC Gauss 8
2016/23)

Example 1.30
A translation moves point A(-3, 2) to the right 5 units and up 3 units. This translation is done a total of 6 times.
After these translations, the point is at (x, y). What is the value of x + y? (CEMC Gauss 8 2018/17)

Example 1.31
A. When the shaded triangle shown is translated, which of the following triangles
can be obtained? (CEMC Gauss 7 2017/13)
B. Identify the vector used for the translation.

Example 1.32
The diagram shows Lori's house located at (6, 3). If Alex's house is located at
(-2,-4), what translation is needed to get from Lori's house to Alex's house?
(CEMC Gauss 7 2014/15)

A. 4 units left, 1 unit up


B. 8 units right, 7 units up
C. 4 units left, 1 unit down
D. 8 units left, 7 units down
E. 7 units right, 8 units down

Example 1.33
On a coordinate grid, Paul draws a line segment of length 1 from the origin to the right, stopping at (1, 0). He
then draws a line segment of length 2 up from this point, stopping at (1; 2). He continues to draw line segments
to the right and up, increasing the length of the line segment he draws by 1 each time. One of his line segments
stops at the point (529, 506). What is the endpoint of the next line segment that he draws? (CEMC Gauss 7
2014/25)

1.34: Translations using Vectors


A point translated using the vector (𝑥1 , 𝑦1 ) moves:
➢ 𝑥1 units in the x direction

P a g e 13 | 95
Get all the files at: https://bit.ly/azizhandouts
Aziz Manva ([email protected])

➢ 𝑦1 units in the y direction

Example 1.35
Draw a diagram for each part below.
A. Translate the point (2,1) using the vector (3, −2).
B. Translate the square with vertices at (0,0), (4,1), (3,5), (−1,4) using the vector (−1, +2).
C. Consider a square with vertices (2,1)(5,2)(4,5)(1,4). Introduce a coordinate system so that the lower
left vertex of the square is at the origin.

Part A

Part B
We can do this visually. The vector (−1,2) is equivalent to moving left by 1 unit, and moving up by two units.
Hence, take each vertex, and move it as per above.
We can also do this algebraically:
(0 − 1,0 + 2) = (−1,2)
(4 − 1,1 + 2) = (3,3)
(3 − 1,5 + 2) = (2,7)
(−1 − 1,4 + 2) = (−2,6)

Part C
If we translate the given square using the vector (−2, −1), then the lower left
vertex will be at the origin.
And doing this, we get:
(0,0, )(3,1)(2,3)(−1,3)

Example 1.36
A. The point (2,5) is translated to the point (7, −1). Identify the vector used
for the translation.
B. The triangle (2,5), (3𝑥, 4𝑦), (5𝑎 + 2,9𝑏 − 4) is translated to (3,4), (7,2), (2𝑎 − 6,9𝑏 − 6). Identify the
vector used for the translation. Find the values of 𝑥, 𝑦, 𝑎, 𝑏.

Part A
(2 + 𝑥, 5 + 𝑦) = (7, −1)
2+𝑥 =7⇒𝑥 =7−2=5
5 + 𝑦 = −1 ⇒ 𝑦 = −6
𝑉𝑒𝑐𝑡𝑜𝑟 = (𝑥, 𝑦) = (5, −6)
Part B
𝑉𝑒𝑐𝑡𝑜𝑟 = (1, −1)

P a g e 14 | 95
Get all the files at: https://bit.ly/azizhandouts
Aziz Manva ([email protected])

3𝑥 + 1 = 7 ⇒ 3𝑥 = 6 ⇒ 𝑥 = 2
1
4𝑦 − 1 = 2 ⇒ 4𝑦 = 1 ⇒ 𝑦 =
4
5𝑎 + 2 + 1 = 2𝑎 − 6 ⇒ 3𝑎 = −9 ⇒ 𝑎 = −3
9𝑏 − 4 − 1 = 9𝑏 − 6 ⇒ −5 = −6 ⇒ 𝑁𝑜 𝑆𝑜𝑙𝑢𝑡𝑖𝑜𝑛𝑠 𝑓𝑜𝑟 𝑏 ⇒ 𝑏 ∈ 𝜙

1.37: Lattice Points


A lattice point is a point on the coordinate plane with coordinates
(𝑥1 , 𝑦1 ), 𝑥, 𝑦 ∈ ℤ

Example 1.38
A square of side 1 has one vertex at the origin, and all vertices are lattice points. Let 𝐴 be the set of possible
positions that the square can be in. The square is translated using the vector (𝑥, 𝑦), 𝑥, 𝑦 ∈ {±1}. Let 𝐵 be the set
of possible positions that the square can be in after the translation. Find the number of elements in Set 𝐵, but
not in Set 𝐴.

Example 1.39
A dot starts at (20, 19). It can move one unit vertically or horizontally to one of
the points (21, 19), (19, 19), (20, 20), or (20, 18). From there it can move two units in either direction that is
perpendicular to the first move. All moves thereafter increase in length by one unit (three units, four units, five
units, etc.) and must be perpendicular to the direction of the previous move. The dot stops after ten moves.
Which of the following final locations is not possible? (CEMC Gauss 7 2019/24)
A. (27,33)
B. (30,40)
C. (21,21)
D. (42,44)
E. (37,37)

Example 1.40
Tiles 𝐼, 𝐼𝐼, 𝐼𝐼𝐼 and 𝐼𝑉 are translated so one tile coincides with each of the rectangles 𝐴, 𝐵, 𝐶 and 𝐷. In the final
arrangement, the two numbers on any side common to two adjacent tiles must be the same. Which of the tiles is
translated to Rectangle 𝐶? (AMC 8 2007/11)

P a g e 15 | 95
Get all the files at: https://bit.ly/azizhandouts
Aziz Manva ([email protected])

Example 1.41
The five pieces shown below can be arranged to form four of the five figures shown in the choices. Which figure
cannot be formed? (AMC 9 2009/4)

(𝐴) (𝐵) (𝐶) (𝐷) (𝐸)]

1.3 Reflection
A. Reflection

1.42: Reflecting across the axes


To reflect a point across
➢ 𝒙 axis, we negate the 𝒚 coordinate.
➢ 𝒚 axis, we negate the 𝒙 coordinate.
➢ origin, we negate both coordinates

Example 1.43
Consider the point (2,1). Reflect the point as directed. In each case, draw the graph, and state the coordinates as
well.
A. Across the x axis.
B. Across the y axis
C. Across the origin

Part A
Algebraically, to reflect a point across the x-axis, we negate the y coordinate
(2,1) → (2, −1)

P a g e 16 | 95
Get all the files at: https://bit.ly/azizhandouts
Aziz Manva ([email protected])

Part B
Algebraically, to reflect a point across the y-axis, we negate the x
coordinate
(2,1) → (−2,1)

Part C
Algebraically, to reflect a point across the y-axis, we negate both the
x and the y coordinate:
(2,1) → (−2, −1)

Example 1.44
A. If the point (3,4) is reflected in the x-axis, what are the coordinates of its
image? (CEMC Gauss 7 2011/9)
B. The point A(1, 2) is reflected in the y-axis. The new coordinates are (CEMC
Gauss 8 2014/15)
C. The point (-2,-3) is reflected in the x-axis. What are the coordinates of its
image after the reflection? (CEMC Gauss 8 2013/9)
D. In the graph shown, which of the following represents the image of the line
segment PQ after a reflection across the x-axis? (CEMC Gauss 7 2015/18,
CEMC Gauss 8 2015/13)

Example 1.45
The letter F is reflected in Line 1. The image is then reflected in Line 2. The shape
that results is (CEMC Gauss 8 2006/15)

1.46: Reflecting across a vertical/horizontal line

Example 1.47
Reflect the point (2,1)
A. Across the line 𝑥 = 4.
B. Across the line 𝑦 = 3

P a g e 17 | 95
Get all the files at: https://bit.ly/azizhandouts
Aziz Manva ([email protected])

Part A
First, draw the line
𝑥=4
Then, the point (2,1) has to be the same distance from 𝑥 = 4 after
being reflected as before. But, it has to be on the other side of the line.
And, hence it will have coordinates
(6,1)
Part B
First, draw the line
𝑦=4
Then, the point (2,1) has to be the same distance from 𝑦 = 3 after being reflected as
before. But, it has to be on the other side of the line.
And, hence it will have coordinates
(3,1)

1.48: Reflecting across 𝒚 = 𝒙


To reflect a point across the line 𝒚 = 𝒙
➢ We interchange the x and the y coordinates

Example 1.49
A. Reflect the point in the diagram alongside across the line 𝑦 = 𝑥.
B. Reflect line segment 𝐴𝐵, with 𝐴(−1,2) and 𝐵(1,2) across the line 𝑦 = 𝑥.

1.50: Reflecting across any line

Example 1.51

Example 1.52: Reflecting Shapes


Δ𝐴𝐵𝐶 has vertices 𝐴 = (1,1), 𝐵 = (5,1), 𝐶 = (5, −4).
A. Δ𝐴𝐵𝐶 is reflected across the 𝑥-axis to get Δ𝑋𝑌𝑍. Draw Δ𝑋𝑌𝑍.
B. Δ𝑋𝑌𝑍 is reflected across the 𝑦-axis to get Δ𝑃𝑄𝑅. Draw Δ𝑃𝑄𝑅.
C. Δ𝐴𝐵𝐶 is reflected across the line 𝑥 = 6 to get Δ𝐿𝑀𝑁. Draw Δ𝐿𝑀𝑁.

P a g e 18 | 95
Get all the files at: https://bit.ly/azizhandouts
Aziz Manva ([email protected])

D. Δ𝐴𝐵𝐶 is reflected across the line 𝑦 = −1 to get Δ𝐷𝐸𝐹. Draw Δ𝐷𝐸𝐹.

Parts A and B
We can reflect across the x-axis by negating the y coordinate of each point
of Δ𝐴𝐵𝐶:
𝑋 = (1, −1), 𝑌 = (5, −1), 𝑍 = (5,4)

We can reflect across the y-axis by negating the x coordinate of each point
of Δ𝑋𝑌𝑍:
𝑃 = (−1, −1), 𝑄 = (−5, −1), 𝑅 = (−5,4)

Parts C
We can reflect across the line 𝑥 = 6 by calculating the horizontal
distance of the point from 𝑥 = 6, and moving it double that many
points to the right.
𝐴 = (1,1), 𝐵 = (5,1), 𝐶 = (5, −4)
𝐿 = (1 + 10,1) = (11,1)
𝑀 = (5 + 2,1) = (7,1)
𝑁 = (5 + 2, −4) = (7, −4)

Part D
We can reflect across the line 𝑦 = −1 by calculating the vertical distance of
the point from 𝑦 = −1, and moving it double that many points down
𝐴 = (1,1), 𝐵 = (5,1), 𝐶 = (5, −4)
𝐷 = (1,1 − 4) = (1, −3)
𝐸 = (5,1 − 4) = (5, −3)
𝐹 = (5, −4 + 6) = (5,2)

Example 1.53: Identifying Transformations


A. The line segment 𝐴𝐵 = (2,3), (3,9) is reflected across an axis to get 𝐴′ 𝐵′ = (−2,3), (−3,9). Then, 𝐴′ 𝐵′ is
reflected across an axis to get 𝐴′′ 𝐵′′ = (−2, −3), (−3, −9). Identify the transformations.

Example 1.54
The polygon 𝑃 with 𝑛 vertices (𝑝1 , 𝑝2 ), (𝑝3 , 𝑝4 ), … (𝑝𝑛 , 𝑝𝑛+1 ) has distinct prime numbers 𝑝1 , 𝑝2 , … , 𝑝𝑛 for the
coordinates of its vertices, with 𝑝1 < 𝑝2 < 𝑝3 < ⋯.
A. State the coordinates of the polygon after it is reflected across the 𝑥-axis 𝑛 times.
B. State the coordinates of the polygon after it is reflected across the 𝑦-axis 𝑛 + 1 times.

𝑛 𝑖𝑠 𝑜𝑑𝑑 ⇒ 𝑛 + 1 𝑖𝑠 𝑒𝑣𝑒𝑛
Part A
By parity, reflecting across the 𝑥 axis an odd number of times is the same as reflecting it once.
We negate the 𝑦 coordinates of the points:

P a g e 19 | 95
Get all the files at: https://bit.ly/azizhandouts
Aziz Manva ([email protected])

(𝑝1 , −𝑝2 ), (𝑝3 , −𝑝4 ), … (𝑝𝑛 , −𝑝𝑛+1 )


Part B
By parity, reflecting across the 𝑦 axis an even number of times is the same as not reflecting it at all.
There is no change in the coordinates.

Example 1.55
A triangle with vertices 𝐴(0,2), 𝐵(−3,2), and 𝐶(−3,0) is reflected about the 𝑥-axis, then the image △ 𝐴′ 𝐵′ 𝐶 ′ is
rotated counterclockwise about the origin by 90∘ to produce △ 𝐴′′ 𝐵′′ 𝐶 ′′ . Which of the following
transformations will return △ 𝐴′′ 𝐵′′ 𝐶 ′′ to △ 𝐴𝐵𝐶? (AMC 10A 2016/16)
(A) counterclockwise rotation about the origin by 90∘ .
(B) clockwise rotation about the origin by 90∘ .
(C) reflection about the 𝑥-axis
(D) reflection about the line 𝑦 = 𝑥
(E) reflection about the 𝑦-axis.

Example 1.56
The figure below shows line l with a regular, infinite, recurring pattern of squares and line segments.

How many of the following four kinds of rigid motion transformations of the plane in which this figure is
drawn, other than the identity transformation, will transform this figure into itself?(AMC 10A 2019/7)

➢ some rotation around a point of line l


➢ some translation in the direction parallel to line l
➢ the reflection across line l
➢ some reflection across a line perpendicular to line l

Example 1.57
The point in the xy-plane with coordinates (1000,2012) is reflected across the line 𝑦 = 2000. What are the
coordinates of the reflected point? (AMC 10B 2012/3)

Example 1.58
Triangle 𝐴𝐵𝐶 lies in the first quadrant. Points 𝐴, 𝐵, and 𝐶 are reflected across the line 𝑦 = 𝑥 to points 𝐴′ , 𝐵′ , and
𝐶 ′ , respectively. Assume that none of the vertices of the triangle lie on the line 𝑦 = 𝑥. Which of the following
statements is not always true? (AMC 10B 2019/5)
(𝐴) Triangle 𝐴′ 𝐵′ 𝐶 ′ lies in the first quadrant.
(B) Triangles 𝐴𝐵𝐶 and 𝐴′ 𝐵′ 𝐶 ′ have the same area.
(C) The slope of line 𝐴𝐴′ is −1.
(D) The slopes of lines 𝐴𝐴′ and 𝐶𝐶 ′ are the same.
(E) Lines 𝐴𝐵 and 𝐴′ 𝐵′ are perpendicular to each other.

P a g e 20 | 95
Get all the files at: https://bit.ly/azizhandouts
Aziz Manva ([email protected])

1.4 Dilations
A. Dilations

1.59: Dilation
A dilation will stretch the position of a point.

Example 1.60
A. Identify the coordinates of point (2,1) after a dilation by a factor of 2, with respect to the origin.
1
B. Δ𝐴𝐵𝐶 has vertices 𝐴 = (1,3), 𝐵 = (5,1), 𝐶 = (5, −4). It is dilated by a factor of , with respect to the
2
origin to get Δ𝐴′𝐵′𝐶′. Draw Δ𝐴′𝐵′𝐶′.

Part A
(2,1) → (2 × 2,1 × 2) = (4,2)
Part B
1 1 1 3
𝐴′ = (1 × , 3 × ) = ( , )
2 2 2 2

1 1 5 1
𝐵 = (5 × , 1 × ) = ( , )
2 2 2 2
1 1 5
𝐶 ′ = (5 × , −4 × ) = ( , −2)
2 2 2

Example 1.61
Δ𝐴𝐵𝐶 has vertices (2,3), (5,7), (11,13). It is dilated with respect to the origin to get Δ𝑋𝑌𝑍 with vertices
3 5 7 11 13
(1, 2 ) , (2 , 2) , ( 2 ,
2
). What is the scale factor?

The scale factor will be the same across all points. Check the x coordinate of the first point.
1
2 → 1 ⇒ 𝐷𝑖𝑣𝑖𝑑𝑒𝑑 𝑏𝑦 2 ⇒ 𝐷𝑖𝑙𝑎𝑡𝑖𝑜𝑛 𝑏𝑦 𝑠𝑐𝑎𝑙𝑒 𝑓𝑎𝑐𝑡𝑜𝑟 𝑜𝑓
2

Example 1.62
2
Line segment 𝐴𝐵 has endpoints 𝐴 = (3 , 𝑥) , 𝐵 = (2𝑝, 3𝑎 + 7). It is dilated with respect to the origin to get line
3
segment 𝑋𝑌 with endpoints 𝑋 = (2 , 𝑦 ) , 𝑌 = (3𝑞, 2𝑎 + 5) .
A. What is the scale factor used for the dilation?
B. What is the ratio 𝑥: 𝑦?
C. What is the ratio 𝑝: 𝑞?
D. Find the value of 𝑎.

Part A
Let the scale factor for the dilation be 𝑠.
2 3 9
∴ 𝑠= ⇒𝑠=
3 2 4
Part B
𝑥 1 1 4
𝑥𝑠 = 𝑦 ⇒ = = = ⇒ 𝑥: 𝑦 = 4: 9
𝑦 𝑠 9 9
4
Part C

P a g e 21 | 95
Get all the files at: https://bit.ly/azizhandouts
Aziz Manva ([email protected])

𝑝 3 3 3 2 2
2𝑝𝑠 = 3𝑞 ⇒ = = = = 3 × = ⇒ 𝑝: 𝑞 = 2: 3
𝑞 2𝑠 2 (9) (9) 9 3
4 2
Part D
(3𝑎 + 7)(𝑠) = 2𝑎 + 5
9
(3𝑎 + 7) ( ) = 2𝑎 + 5
4
27𝑎 + 63 = 8𝑎 + 20
19𝑎 = −43
43
𝑎=−
19

Example 1.63
A dilation of the plane—that is, a size transformation with a positive scale factor—sends the circle of radius 2
centered at 𝐴(2,2) to the circle of radius 3 centered at 𝐴’(5,6). What distance does the origin 𝑂(0,0), move
under this transformation? (AMC 10B 2016/20)

1.5 Rotations
A. Rotations
➢ Full Circle = 360°
1
➢ Half a Circle = 2 × 360° = 180°
1
➢ One fourth of a Circle = × 360° = 90°
4
1
➢ One fifth of a Circle = 4 × 360° = 72°

Example 1.64
The word “stop” starts in the position shown in the diagram to the right. It is then
rotated 180° clockwise about the origin, O, and this result is then reflected in the x-
axis. Which of the following represents the final image? (CEMC Gauss 7 2002/20,
CEMC Gauss 8 2002/19)

Example 1.65
A clockwise rotation around point Z (that is, a rotation in the direction of the arrow)
transforms the shaded quadrilateral to the unshaded quadrilateral. The angle of
rotation is approximately (CEMC Gauss 7 2014/9)

Example 1.66
Rotate the following points

𝟗𝟎° counter-clockwise:
A. (1,2)

P a g e 22 | 95
Get all the files at: https://bit.ly/azizhandouts
Aziz Manva ([email protected])

B. (3,7)
C. (𝑥, 𝑦)
𝟗𝟎° clockwise:
D. (1,2)
E. (3,7)
F. (𝑥, 𝑦)
𝟏𝟖𝟎° clockwise:
G. (1,2)
H. (3,7)
I. (𝑥, 𝑦)

𝟗𝟎° counter-clockwise:
(1,2) → (−2,1)
(3,7) → (−7,3)
(𝑥, 𝑦) → (−𝑦, 𝑥)

Note that, in general:


➢ The 𝑥 coordinate is the negative of the 𝑦
coordinate before rotation.
➢ The 𝑦 coordinate is the 𝑥 coordinate
before rotation.

𝟗𝟎° clockwise:
(1,2) → (2, −1)
(3,7) → (7, −3)
(𝑥, 𝑦) → (𝑦, −𝑥)

Note that, in general:


➢ The 𝑥 coordinate is 𝑦 coordinate before rotation.
➢ The 𝑦 coordinate is the negative of the 𝑥 coordinate
before rotation.

𝟏𝟖𝟎° clockwise:
(1,2) → (−1, −2)
(3,7) → (−3, −7)
(𝑥, 𝑦) → (−𝑥, −𝑦)

In general:
➢ Both the 𝑥 and the 𝑦 coordinate reverse sign.

P a g e 23 | 95
Get all the files at: https://bit.ly/azizhandouts
Aziz Manva ([email protected])

Example 1.67
Triangle ABC has vertices (2,3), (−1,2) and (1, −2). Find the coordinates of the vertices of
the triangle after it has been rotated 90°:
A. Counter-clockwise
B. Clockwise

Part A
(2,3) → (−3,2)
(−1,2) → (−2, −1)
(1, −2) → (2,1)

Part B
We can get the answer to Part B by simply reflecting the answer to Part A across the origin (which has the
effect of rotating it a further 180°.

(2,3) → (3, −2)


(−1,2) → (2,1)
(1, −2) → (−2, −1)

Example 1.68
Which of the following represents the result when the figure shown below is rotated clockwise
120∘ around its center? (AMC 8 1994/4)

Example 1.69
In the pattern below, the cat moves clockwise through the four squares and the mouse moves counterclockwise
through the eight exterior segments of the four squares. (AMC 8 2003/23)

P a g e 24 | 95
Get all the files at: https://bit.ly/azizhandouts
Aziz Manva ([email protected])

If the pattern is continued, where would the cat and mouse be after the 247th move?

Example 1.70
Rectangle 𝑃𝑄𝑅𝑆 lies in a plane with 𝑃𝑄 = 𝑅𝑆 = 2 and 𝑄𝑅 = 𝑆𝑃 = 6. The rectangle is rotated 90∘ clockwise
about 𝑅, then rotated 90∘ clockwise about the point 𝑆 moved to after the first rotation. What is the length of the
path traveled by point 𝑃? (AMC 10A 2008/19)

Example 1.71
A unit square is rotated 45∘ about its center. What is the area of the region swept out by the interior of the
square? (AMC 10A 2013/20)

Example 1.72
The diagram below shows the circular face of a clock with radius 20 cm and a circular disk with radius 10 cm
externally tangent to the clock face at 12 o'clock. The disk has an arrow painted on it, initially pointing in the
upward vertical direction. Let the disk roll clockwise around the clock face. At what point on the clock face will
the disk be tangent when the arrow is next pointing in the upward vertical direction? (AMC 10A 2014/14)

Example 1.73
Triangle 𝑂𝐴𝐵 has 𝑂 = (0,0), 𝐵 = (5,0), and 𝐴 in the first quadrant. In addition, ∠𝐴𝐵𝑂 = 90∘ and ∠𝐴𝑂𝐵 = 30∘ .
Suppose that 𝑂𝐴 is rotated 90∘ counterclockwise about 𝑂. What are the coordinates of the image of 𝐴? (AMC
10B 2008/14)

P a g e 25 | 95
Get all the files at: https://bit.ly/azizhandouts
Aziz Manva ([email protected])

Example 1.74
As shown below, convex pentagon 𝐴𝐵𝐶𝐷𝐸 has sides 𝐴𝐵 = 3, 𝐵𝐶 = 4, 𝐶𝐷 = 6, 𝐷𝐸 = 3, and 𝐸𝐴 = 7. The
pentagon is originally positioned in the plane with vertex 𝐴 at the origin and vertex 𝐵 on the positive 𝑥-axis.
The pentagon is then rolled clockwise to the right along the 𝑥-axis. Which side will touch the point 𝑥 = 2009 on
the 𝑥-axis? (AMC 10B 2009/13)

1.6 Symmetry and Combining Transformations


A. Line Symmetry

Example 1.75
The letter A has a vertical line of symmetry and the letter B does not. How many of the letters H L O R X D P E
have a vertical line of symmetry? (CEMC Gauss 7 2020/12)

𝐻, 𝑂, 𝑋 ⇒ 3 𝐿𝑒𝑡𝑡𝑒𝑟𝑠

Example 1.76
In the right-angled triangle 𝑃𝑄𝑅, 𝑃𝑄 = 𝑄𝑅. The segments 𝑄𝑆, 𝑇𝑈 and 𝑉𝑊 are
perpendicular to 𝑃𝑅, and the segments 𝑆𝑇 and 𝑈𝑉 are perpendicular to 𝑄𝑅, as shown.
What fraction of 𝑃𝑄𝑅 is shaded? (CEMC Gauss 7, 2013/23)

Since 𝑄𝑆 is a line of symmetry


1
Δ𝑆𝑄𝑅 = 𝑜𝑓 Δ𝑃𝑄𝑅
2
Since 𝑆𝑇 is a line of symmetry,
1
Δ𝑆𝑇𝑅 = 1/2 𝑜𝑓 Δ𝑆𝑄𝑅 = 𝑜𝑓 Δ𝑃𝑄𝑅
4
Since 𝑈𝑇 is a line of symmetry,
1 1 1
Δ𝑆𝑇𝑈 𝑖𝑠 𝑜𝑓 𝑇𝑟𝑖𝑎𝑛𝑔𝑙𝑒 𝑆𝑇𝑅 = 𝑜𝑓 Δ𝑆𝑄𝑅 = 𝑜𝑓 Δ𝑃𝑄𝑅
2 4 8

Similarly,
1
Δ𝑈𝑇𝑅 = 𝑜𝑓 Δ𝑃𝑄𝑅
16
1
Δ𝑈𝑉𝑊 = 𝑜𝑓 Δ𝑃𝑄𝑅
32

P a g e 26 | 95
Get all the files at: https://bit.ly/azizhandouts
Aziz Manva ([email protected])

Hence, the shading that we are looking for is


1 1 4 1 5
+ = + =
8 32 32 32 32

Example 1.77
Which of the five "T-like shapes" would be symmetric to the one shown with respect to the dashed line? (AMC 8
1989/11)

Example 1.78
What is the minimum number of small squares that must be colored black so that a line of symmetry lies on the
diagonal 𝐵𝐷 of square 𝐴𝐵𝐶𝐷? (AMC 8 2005/3)

Example 1.79
Which of the following figures has the greatest number of lines of symmetry? (AMC 8 2010/6)
(𝐴) equilateral triangle (𝐵) non-square rhombus (𝐶) non-square rectangle (𝐷) isosceles trapezoid (𝐸) square

How many of the twelve pentominoes pictured below have at least one line of symmetry? (AMC 10 2001/5)

Six

B. Rotational Symmetry

C. Combining Transformations

P a g e 27 | 95
Get all the files at: https://bit.ly/azizhandouts
Aziz Manva ([email protected])

Example 1.80
Making Transformations
Determining Transformations

Example 1.81
The letter F shown below is rotated 90∘ clockwise around the origin, then reflected in the 𝑦-axis, and then
rotated a half turn around the origin. What is the final image? (AMC 10B 2015/8)

Example 1.82
Using two transformations, the letter R is changed as shown:

Using the same two transformations, the letter L is changed as shown:

Using the same two transformations, the letter G is changed to (CEMC Gauss 2008/15)

1.7 Distance Formula


A. Revision: Radicals
To simplify √8 use the following procedure:
➢ Find the prime factorization of the number inside the radical sign.
√8 = √2 × 2 × 2
➢ Separate out perfect squares
√8 = √4 × 2
➢ Break the square root into two separate roots
√8 = √4 × √2
➢ Take the square root of the perfect squares
√8 = 2 × √2 = 2√2

Example 1.83: Simplifying Square Roots


Simplify the following square roots.
A. √50
B. √32
C. √72
D. √125
E. √27

√50 = √25 × 2 = √25 × √2 = 5 × √2 = 5√2


√32 = √16 × √2 = 4√2
√72 = √36 × √2 = 6√2
√125 = √25 × √5 = 5√5

P a g e 28 | 95
Get all the files at: https://bit.ly/azizhandouts
Aziz Manva ([email protected])

√27 = √9 × √3 = 3√3

B. Distance Formula

Example 1.84: Pythagorean Theorem to Find Distance


A. Find the distance between (2,3) and (4,7).
B. Find the distance between (2,1) and (7,5).
C. Find the distance between (3,5) 𝑎𝑛𝑑 (7,1).

Part A
The line that goes from A to B is slanted. It’s easier to find the straight-line distance. Plot
point 𝐶(4,3), which has the same vertical value as A, and the same horizontal value as B.

The horizontal distance between (2,3) and (4,7) is given by the difference in their 𝑥-coordinates, which is:
𝐴𝐶 = 𝑥2 − 𝑥1 = 4 − 2 = 2

The vertical distance between (2,3) and (4,7) is given by the difference in
the 𝑦-coordinates, which is:
𝐵𝐶 = 𝑦2 − 𝑦1 = 7 − 3 = 4

Apply the Pythagorean Theorem:


𝐴𝐵2 = 𝐴𝐶 2 + 𝐵𝐶 2 = 22 + 42 = 4 + 16 = 20
Take square roots:
𝐴𝐵 = √20 = √4 × √5 = 2√5

Part B
By the Pythagorean Theorem:
𝐷 = √(7 − 2)2 + (5 − 1)2 = √52 + 42 = √25 + 16 = √41

Part C
By the Pythagorean Theorem:
𝐷 = √(7 − 3)2 + (5 − 1)2 = √42 + 42 = √32

1.85: Distance Formula in the 2D Plane


The distance between two points on the coordinate plane is given by the square
root of the sum of the squares of the difference between the respective 𝒙
coordinates, and the respective 𝒚 coordinates.
𝑨𝑩 = √(𝒙2 − 𝒙1 )𝟐 + (𝒚2 − 𝒚1 )𝟐

Given: 𝐴 = (𝑥1 , 𝑦1 ) and 𝐵 = (𝑥2 , 𝑦2 )


Proof: Introduce Point 𝐶(𝑥2 , 𝑦1 ), such that Δ𝐴𝐵𝐶 is right-angled

We want the distance AB. Hence, apply Pythagoras Theorem in right Δ𝐴𝐵𝐶:
𝐴𝐵2 = 𝐴𝐶 2 + 𝐶𝐵2 = (𝑥2 − 𝑥1 )2 + (𝑦2 − 𝑦1 )2
Take the square root of both sides:
𝐴𝐵 = √(𝑥2 − 𝑥1 )2 + (𝑦2 − 𝑦1 )2

P a g e 29 | 95
Get all the files at: https://bit.ly/azizhandouts
Aziz Manva ([email protected])

Property
The order of the points does not matter, since the square of a number is the same as the square of its negation:
𝐴𝐵
= √(𝑥2 − 𝑥1 )2 + (𝑦2 − 𝑦1 )2
= √[−(𝑥2 − 𝑥1 )]2 + [−(𝑦2 − 𝑦1 )]2
= √(𝑥1 − 𝑥2 )2 + (𝑦1 − 𝑦2 )2

1.86: Distance Formula in 3D


The distance between two points 𝐴 = (𝑥1 , 𝑦1 , 𝑧1 ) and 𝐵 = (𝑥2 , 𝑦2 , 𝑧2 ) in three-dimensional space is given by:
𝐴𝐵 = √(𝑥2 − 𝑥1 )2 + (𝑦2 − 𝑦1 )2 + (𝑧2 − 𝑧1 )2

➢ Let 𝐴 and 𝐵 be at opposite corners of a cuboid in 3D coordinate space.


➢ Find the length of the base diagonal
➢ Find the length of the long diagonal using two consecutive applications of Pythagoras Theorem.

Example 1.87: Using the Distance Formula


Find the distance between:

A. (−5
⏟,⏟5 ) and ( ⏟
5 , −5
⏟)
𝒙𝟏 𝒚𝟏 𝒙𝟐 𝒚𝟐
B. (−1,4) and (2, −7)
C. (3, −1) and (0,4)
1 1 2 3
D. ( , ) and ( , )
2 3 3 4
E. (2,3,7) and (1, −2,5)

Part A
2
𝐷 = √(5 − (−5)) + (−5 − 5)2 = √100 + 100 = √200 = √100 × √2 = 10√2
Part B
2
𝐷 = √(2 − (−1)) + (−7 − 4)2 = √32 + (−11)2 = √9 + 121 = √130
Part C
2
𝐷 = √(0 − 3)2 + (4 − (−1)) = √(−3)2 + 52 = √9 + 25 = √34
Part D
2 1 2 3 1 2 1 2 5 2 1 25 29 √29
√ √
𝐷 = ( − ) +( − ) = ( ) +( ) = √ + =√ =
3 2 4 3 6 12 36 144 144 12
Part E
2
𝐷 = √(2 − 1)2 + (3 − (−2)) + (7 − 5)2 = √12 + 52 + 22 = √1 + 25 + 4 = √30

Example 1.88: Basic Word Problems


A. Find the distance between the points (2,2) and (−1, −1). (MathCounts 1992 Warm-Up 6)
B. What is the number of units in the distance between (2,5) and (−6, −1)? (MathCounts 2000 Warm-Up
17)

P a g e 30 | 95
Get all the files at: https://bit.ly/azizhandouts
Aziz Manva ([email protected])

1.89: Distance From the Origin


2𝐷 𝐺𝑒𝑜𝑚𝑒𝑡𝑟𝑦: √𝑥 2 + 𝑦 2
3𝐷 𝐺𝑒𝑜𝑚𝑒𝑡𝑟𝑦: √𝑥 2 + 𝑦 2 + 𝑧 2

When we want to find the distance of a point from the origin, the origin has coordinates 𝑂 = (0,0). Therefore,
the distance of 𝐴 = (𝑥2 , 𝑦2 ) from the origin becomes
√(𝑥2 − 𝑥1 )2 + (𝑦2 − 𝑦1 )2 = √(𝑥2 − 0)2 + (𝑦2 − 0)2 = √(𝑥2 )2 + (𝑦2 )2 = √𝑥 2 + 𝑦 2
Don’t bother memorizing the formula. Keep the concept in mind, and use the main formula only.

Example 1.90: Pythagorean Triplets


Find the distance that each point lies from the origin.
A. (3,4)
B. (5,12)
C. (7,24)
D. (8,15)
E. (9,40)

𝐴. √32 + 42 = √9 + 16 = √25 = 5
𝐵. √52 + 122 = √25 + 144 = √169 = 13
𝐶. √72 + 242 = √49 + 576 = √625 = 25
𝐷. √82 + 152 = √64 + 225 = √289 = 17
𝐸. √92 + 402 = √81 + 1600 = √1681 = 41

Example 1.91
A. The point 𝑃 has the same 𝑥 coordinate and the same 𝑦 coordinate. Find the coordinates of point 𝑃 if it is
at a distance of √8 units from the origin on the coordinate plane.
B. A mouse at point 𝑂 moves 𝑛 units in the 𝑥 direction, and 𝑛 units in the 𝑦 direction to reach point 𝑃.
Then, he comes back in a straight line to 𝑂, and travels √5 units. Find 𝑛.
C. A mouse at point 𝑂 moves 𝑛 units in the 𝑥 direction, and 𝑛 units in the 𝑦 direction to reach point 𝑃.
Then, he comes back in a straight line to 𝑂. Find the difference between the distance travelled when
going, and the distance travelled when coming back.

Part A
Let 𝑃 = (𝑎, 𝑎). By the distance formula, the distance from the origin (0,0):
√(𝑎 − 0)2 + (𝑎 − 0)2 = √8
√2𝑎2 = √8 ⇒ 2𝑎2 = 8 ⇒ 𝑎2 = 4 ⇒ 𝑎 = ±2
Part B
Introduce an origin at point 𝑂. Then the coordinates of point 𝑃 are:
(𝑛, 𝑛)
By the distance formula:
√(𝑛 − 0)2 + (𝑛 − 0)2 = √5
5 5
√2𝑛2 = √5 ⇒ 2𝑛2 = 5 ⇒ 𝑛2 = ⇒ 𝑛 = ±√
2 2
Part C
The distance when going is:

P a g e 31 | 95
Get all the files at: https://bit.ly/azizhandouts
Aziz Manva ([email protected])

𝑛 + 𝑛 = 2𝑛
The distance when coming back is:
√(𝑛 − 0)2 + (𝑛 − 0)2 = √2𝑛2 = √2𝑛
Difference
= 2𝑛 − √2𝑛 = 𝑛(2 − √2)

Example 1.92
𝑎𝑛 = 2𝑛 , 𝑛 = 1,2,3, …
A. Find the distance of the point (𝑎1 , 𝑎2, 𝑎3 ) from the origin.
B. 𝑃𝑛 = (𝑎𝑛 , 𝑎𝑛+1, 𝑎𝑛+2 ). The distance between 𝑃𝑛 and the origin is given by 𝐷𝑛 . Find 𝐷3 − 𝐷2.

Part A
(𝑎1 , 𝑎2 , 𝑎3 ) = (21 , 22 , 23 ) = (2,4,8)
𝐷𝑖𝑠𝑡𝑎𝑛𝑐𝑒 = √22 + 42 + 82 = √4 + 16 + 64 = √84 = 2√21
Part B
𝐷2 = √(22 )2 + (23 )2 + (24 )2 = √24 + 26 + 28 = √16 + 64 + 256 = √336 = 4√21
𝐷3 = √(23 )2 + (24 )2 + (25 )2 = √26 + 28 + 210 = √64 + 256 + 1024 = √1344 = 8√21
𝐷3 − 𝐷2 = 8√21 − 4√21 = 4√21

Example 1.93
𝑓(𝑥) = 𝑎𝑛3 + 𝑏𝑛2 + 𝑐𝑛 + 𝑑 = 0, 𝑎≠0
Point 𝑃 is a point in 3𝐷 space. Find the distance from the origin of 𝑃 = (𝑥, 𝑦, 𝑧) if
➢ 𝑥 is equal to the sum of the roots of the above equation.
➢ 𝑦 is equal to the product of the zeroes of 𝑓.
➢ 𝑧 is equal to the sum of the pair-wise products of the roots of the above equation.

𝑏 𝑑 𝑐
𝑥=− , 𝑦=− , 𝑧=
𝑎 𝑎 𝑎

𝑏 2 𝑑 2 𝑐 2 𝑏2 + 𝑐 2 + 𝑑2 √𝑏 2 + 𝑐 2 + 𝑑2

𝐷𝑖𝑠𝑡𝑎𝑛𝑐𝑒 = (− ) + (− ) + ( ) = √ =
𝑎 𝑎 𝑎 𝑎2 𝑎

Example 1.94
𝑥−3
𝑓(𝑥) =
(𝑥 2
− 𝑥 − 6)(𝑥 2 + 7𝑥 + 10)
The domain of 𝑓(𝑥) is 𝐷𝑓 = ℝ − {𝑥, 𝑦, 𝑧}. Find the distance of the point 𝑃 = {𝑥, 𝑦, 𝑧} from the origin.

𝑥−3
𝑓(𝑥) =
(𝑥 − 3)(𝑥 + 2)2 (𝑥 + 5)

𝐷𝑓 = ℝ − {−5, −2,3}

𝐷𝑖𝑠𝑡𝑎𝑛𝑐𝑒 = √(−5)2 + (−2)2 + 32 = √25 + 4 + 9 = √38

Example 1.95
The function used for the Collatz conjecture is defined piece-wise:

P a g e 32 | 95
Get all the files at: https://bit.ly/azizhandouts
Aziz Manva ([email protected])

𝑛
, 𝑖𝑓 𝑛 𝑖𝑠 𝑒𝑣𝑒𝑛
𝑓(𝑛) = { 2
3𝑛 + 1, 𝑖𝑓 𝑛 𝑖𝑠 𝑜𝑑𝑑
Recall that composition means that the function is applied twice.
𝑓 2 (𝑛) → 𝑓 𝑎𝑝𝑝𝑙𝑖𝑒𝑑 𝑡𝑤𝑖𝑐𝑒
𝑚 (𝑛)
𝑓 → 𝑓 𝑎𝑝𝑝𝑙𝑖𝑒𝑑 𝑚 𝑡𝑖𝑚𝑒𝑠
For some 𝑦:
𝑓 𝑥 (19) = 𝑓 𝑥+3 (19), 𝑓 𝑥+1 (19) = 𝑓 𝑥+4 (19), 𝑓 𝑥+2 (19) = 𝑓 𝑥+5 (19)
Find the distance from the origin of the point:
(𝑓 𝑥 (19), 𝑓 𝑥+1 (19), 𝑓 𝑥+2 (19))

19,58,29,88,44,22,11,34,17,52,26,13,40,20,10,5,16,8, 4,2,1
⏟ , 4,2,1

𝐶𝑦𝑐𝑙𝑒 𝐶𝑦𝑐𝑙𝑒

𝐷𝑖𝑠𝑡𝑎𝑛𝑐𝑒 = √42 + 22 + 12 = √16 + 4 + 1 = √21

Example 1.96
How many lattice points are exactly 1 unit away from the origin:
A. on the coordinate plane.
B. in 3D coordinate space.
C. in 𝑛 −dimensional coordinate space.

Part A
(±1,0), (0, ±1) ⇒ 4 𝑃𝑜𝑖𝑛𝑡𝑠
Part B
(±1,0,0), (0, ±1,0)(0,0, ±1) ⇒ 6 𝑃𝑜𝑖𝑛𝑡𝑠
Part C
(𝑎1 , 𝑎2 , … , 𝑎𝑛 )
Where exactly one out of 𝑎1 , 𝑎2 , … , 𝑎𝑛 is non-zero, others are zero.
And the non-zero variable can ±1.

𝑛 ×⏟ 2 = 2𝑛
𝐶ℎ𝑜𝑖𝑐𝑒 ±1
𝑜𝑓 𝐷𝑖𝑟𝑒𝑐𝑡𝑖𝑜𝑛

Example 1.97
How many lattice points are exactly 3 units away from the origin:
A. on the coordinate plane.
B. in 3D coordinate space.
C. in 4D dimensional coordinate space.
D. in 5D dimensional coordinate space.
E. in 𝑛-dimensional coordinate space.

Part A
(0, ±3), (±3,0) ⇒ 4 𝑃𝑜𝑖𝑛𝑡𝑠
Part B
Case I
(±3,0,0), (0, ±3,0)(0,0, ±3) ⇒ 6 𝑃𝑜𝑖𝑛𝑡𝑠

Case II

P a g e 33 | 95
Get all the files at: https://bit.ly/azizhandouts
Aziz Manva ([email protected])

Let the coordinates of a point satisfying this condition be 𝑃 = (𝑥, 𝑦, 𝑧), 𝑥, 𝑦, 𝑧 ∈ ℤ


√𝑥 2 + 𝑦 2 + 𝑧 2 = 3 ⇒ 𝑥 2 + 𝑦 2 + 𝑧 2 = 9
If we take
𝑥 = 3 ⇒ 𝑥 2 = 9 ⇒ 𝑦 2 = 𝑧 2 = 0 ⇒ 𝑁𝑜𝑡 𝑉𝑎𝑙𝑖𝑑
𝑥 = 1 ⇒ 𝑦 2 + 𝑧 2 = 8 ⇒ 𝑦 = 𝑧 = 2 𝑊𝑜𝑟𝑘𝑠
(1,2,2,3) 𝑖𝑠 𝑎 𝑃𝑦𝑡ℎ𝑎𝑔𝑜𝑟𝑒𝑎𝑛 𝑄𝑢𝑎𝑑𝑟𝑢𝑝𝑙𝑒

The points which will work are:


(1,2,2), (2,1,2), (2,2,1)

And each of the above can be positive or negative.



2 × ⏟
2 × ⏟
2 =8
𝑥 𝑐𝑜𝑜𝑟𝑑𝑖𝑛𝑎𝑡𝑒 𝑦 𝑐𝑜𝑜𝑟𝑑𝑖𝑛𝑎𝑡𝑒 𝑧 𝑐𝑜𝑜𝑟𝑑𝑖𝑛𝑎𝑡𝑒
Total from this case
= 3 × 8 = 24

Total for this Part


= 6 + 24 = 30
Part D
Case I
(±3,0,0,0), (0, ±3,0,0)(0,0, ±3,0), (0,0,0, ±3) ⇒ 8 𝑃𝑜𝑖𝑛𝑡𝑠

Case II
Let the coordinates of a point satisfying this condition be 𝑃 = (𝑥, 𝑦, 𝑧), 𝑥, 𝑦, 𝑧 ∈ ℤ
√𝑥 2 + 𝑦 2 + 𝑧 2 + 𝑎2 = 3 ⇒ 𝑥 2 + 𝑦 2 + 𝑧 2 + 𝑎2 = 9
If we take
𝑥 = 3 ⇒ 𝑥 2 = 9 ⇒ 𝑦 2 = 𝑧 2 = 0 ⇒ 𝑁𝑜𝑡 𝑉𝑎𝑙𝑖𝑑
𝑥 = 1 ⇒ 𝑦 2 + 𝑧 2 = 8 ⇒ 𝑦 = 𝑧 = 2 𝑊𝑜𝑟𝑘𝑠

The points which will work are a distinct rearrangement of:


(0,1,2,2)
There are

4 × ⏟
3 × ⏟
8 = 96
𝐶ℎ𝑜𝑖𝑐𝑒𝑠 𝑊𝑎𝑦 𝑡𝑜 𝑃𝑜𝑠𝑖𝑡𝑖𝑣𝑒
𝑓𝑜𝑟 𝑍𝑒𝑟𝑜 𝐴𝑟𝑟𝑎𝑛𝑔𝑒 (1,2,2) 𝑎𝑛𝑑 𝑁𝑒𝑔𝑎𝑡𝑖𝑣𝑒 𝑉𝑎𝑙𝑢𝑒𝑠

Total for this Part


= 6 + 96 = 102
Part E
Points on the coordinate axes:
2𝑛 𝑃𝑜𝑖𝑛𝑡𝑠

Case (1,2,2)
𝑛−1 (𝑛 − 1)(𝑛 − 2)

𝑛 × ( ) × ⏟3
2 =𝑛∙ ∙ 8 = 4𝑛(𝑛 − 1)(𝑛 − 2)
⏟ 2 2
𝑃𝑜𝑠𝑖𝑡𝑖𝑜𝑛 𝑜𝑓 1 𝑃𝑜𝑠𝑖𝑡𝑖𝑣𝑒
𝑃𝑜𝑠𝑖𝑡𝑖𝑜𝑛 𝑜𝑓 2 𝑎𝑛𝑑 𝑁𝑒𝑔𝑎𝑡𝑖𝑣𝑒 𝑉𝑎𝑙𝑢𝑒𝑠
Case (2,1,1,1,1,1)
𝑛−1 𝑛−1

𝑛 × ( ) × ⏟6
2 = 26 𝑛 ( )
⏟ 5 5
𝑃𝑜𝑠𝑖𝑡𝑖𝑜𝑛 𝑜𝑓 2 𝑃𝑜𝑠𝑖𝑡𝑖𝑣𝑒
𝑃𝑜𝑠𝑖𝑡𝑖𝑜𝑛 𝑜𝑓 1′ 𝑠 𝑎𝑛𝑑 𝑁𝑒𝑔𝑎𝑡𝑖𝑣𝑒 𝑉𝑎𝑙𝑢𝑒𝑠

P a g e 34 | 95
Get all the files at: https://bit.ly/azizhandouts
Aziz Manva ([email protected])

Case (1,1,1,1,1,1,1,1,1)
𝑛 𝑛
(⏟) × ⏟9
2 = 29 ( )
9 9
𝑃𝑜𝑠𝑖𝑡𝑖𝑣𝑒
𝐶ℎ𝑜𝑜𝑠𝑒 𝑡ℎ𝑒 𝑎𝑛𝑑 𝑁𝑒𝑔𝑎𝑡𝑖𝑣𝑒 𝑉𝑎𝑙𝑢𝑒𝑠
9 1′ 𝑠

Challenge 1.98
𝐴 = (𝑎, 𝑏) and 𝐵 = (𝑎, 𝑏, 𝑐), 𝑎𝑏𝑐 ≠ 0 are lattice points in the 𝑥𝑦 coordinate plane, and 3D coordinate space,
respectively. 𝐴 is an integral number of units from the origin. 𝐵 is 13 units from the origin. The 𝑛 distinct
ordered triples satisfying these conditions can be written (𝑎1 , 𝑏1 , 𝑐1 ), (𝑎2 , 𝑏2 , 𝑐2 ), … , (𝑎𝑛 , 𝑏𝑛 , 𝑐𝑛 ). Find 𝑥 + 𝑦 given
that:
𝑛

∏ 𝑎𝑖 𝑏𝑖 𝑐𝑖 = (𝑎1 𝑏1 𝑐1 ) × (𝑎2 𝑏2 𝑐2 ) × … × (𝑎𝑛 𝑏𝑛 𝑐𝑛 ) = 2𝑥 3𝑦


𝑖=1

A is a lattice point, and also it is an integer number of units from the origin:
∴ (𝑎, 𝑏, √𝑎2 + 𝑏 2 ) 𝑖𝑠 𝑎 𝑃𝑦𝑡ℎ𝑎𝑔𝑜𝑟𝑒𝑎𝑛 𝑇𝑟𝑖𝑝𝑙𝑒𝑡

Introduce a cuboid with 𝑂 = (0,0) at the origin, 𝐴 = (𝑎, 𝑏) on the base, diagonally opposite to O.
Point B is on the longest diagonal of the cuboid.
𝐵𝑎𝑠𝑒 𝑑𝑖𝑎𝑔𝑜𝑛𝑎𝑙 𝑜𝑓 𝑡ℎ𝑒 𝑐𝑢𝑏𝑜𝑖𝑑 = √𝑎2 + 𝑏 2 ⇒ 𝐼𝑛𝑡𝑒𝑔𝑒𝑟
𝐻𝑒𝑖𝑔ℎ𝑡 𝑜𝑓 𝑡ℎ𝑒 𝑐𝑢𝑏𝑜𝑖𝑑 = 𝑐 ⇒ 𝐼𝑛𝑡𝑒𝑔𝑒𝑟
(√𝑎2 + 𝑏 2 , 𝑐, 13) 𝑖𝑠 𝑎 𝑃𝑦𝑡ℎ𝑎𝑔𝑜𝑟𝑒𝑎𝑛 𝑇𝑟𝑖𝑝𝑙𝑒𝑡
Hence, we must have
(5,12,13) 𝑜𝑟 (12,5,13)

12 does not work, since 12 is not the hypotenuse of any Pythagorean Triplet.
However, 5 is the hypotenuse of
(3,4,5)

(𝑎, 𝑏, 𝑐) = (3,4,12)
We have a total of
2 𝐶ℎ𝑜𝑖𝑐𝑒𝑠 × ⏟
⏟ 2 𝐶ℎ𝑜𝑖𝑐𝑒𝑠 × ⏟
2 𝐶ℎ𝑜𝑖𝑐𝑒𝑠
±3 ±4 ±12

(3,4,12), (−3,4,12), (3, −4,12), (3,4, −12), (−3, −4,12), (−3,4, −12), (3, −4, −12), (−3, −4, −12)

Now we need to take the product of the above:

(3 × 4 × 12)8 = (31 × 22 × 3 × 22 )8 = (32 × 24 )8 = 316 × 232


𝑎 + 𝑏 = 16 + 32 = 48

Example 1.99: Radicals in coordinate points


Recall the following property from exponents and radicals”
2 1 2 1
×2
(√𝑥) = (𝑥 2 ) = 𝑥 2 = 𝑥 1 = 𝑥 ⇒ 𝑇ℎ𝑒 𝑠𝑞𝑢𝑎𝑟𝑒 𝑎𝑛𝑑 𝑡ℎ𝑒 𝑠𝑞𝑢𝑎𝑟𝑒 𝑟𝑜𝑜𝑡 𝑐𝑎𝑛𝑐𝑒𝑙

Find the distance that each point lies from the origin.
A. (√40, √41)

P a g e 35 | 95
Get all the files at: https://bit.ly/azizhandouts
Aziz Manva ([email protected])

B. (√3, √22)
C. (−√3, √5)
D. (√7, −√11)

2 2
𝐴. √(√40) + (√41) = √40 + 41 = √81 = 9
2 2
𝐵. √(√3) + (√22) = √3 + 22 = √25 = 5
2 2
𝐶. √(−√3) + (√5) = √3 + 5 = √8 = √4 × √2 = 2√2
2 2
𝐷. √(√7) + (−√11) = √7 + 11 = √18 = √9 × √2 = 3√2

Example 1.100
What is the minimum distance between two points in 3D coordinate space?
A. Without any further condition
B. If they are both lattice points
C. If they are both distinct lattice points

Parts A and B
𝑀𝑖𝑛𝑖𝑚𝑢𝑚 𝐷𝑖𝑠𝑡𝑎𝑛𝑐𝑒 = 0
Part C
𝑀𝑖𝑛𝑖𝑚𝑢𝑚 𝐷𝑖𝑠𝑡𝑎𝑛𝑐𝑒 = 1
C. Algebraic Applications
In the prior section, we looked at direct applications of the distance formula, which let us calculate the distance
from
➢ One point to another
✓ Remember that distance is never negative. Hence, the smallest value for distance is zero.
➢ A point to the origin
✓ This is just a special case of one point to another, but it is quite useful.

In this section, we look at calculations using the distance formula, where:


➢ some values are missing
➢ OR special conditions are given

These missing values or special conditions lead to the formation of equations. Solving these equations gives us
the values that we need to find the answer to the question.

Example 1.101: Distance from the origin


Sherlock finds that the distance of a well from a tree on an alfalfa farm is 23 units. He plots a coordinate
system, and puts the well at the origin. What is the sum of the squares of the coordinates of the tree?

𝐷 ( 𝑇𝑟𝑒𝑒 ⏟ ) = √(𝑥2 − 𝑥1 )2 + (𝑦2 − 𝑦1 )2 = √(𝑥2 − 0)2 + (𝑦2 − 0)2 = √(𝑥


⏟ , 𝑊𝑒𝑙𝑙 ⏟ 2 )2 + (𝑦2 )2 = 23
(𝒙𝟐 ,𝒚𝟐 ) (𝟎,𝟎) 𝑬𝒒𝒖𝒂𝒕𝒊𝒐𝒏 𝑰
Square both sides of Equation I:

P a g e 36 | 95
Get all the files at: https://bit.ly/azizhandouts
Aziz Manva ([email protected])

(𝑥2 )2 + (𝑦2 )2 = 529



𝑺𝒖𝒎 𝒐𝒇 𝒕𝒉𝒆 𝒔𝒒𝒖𝒂𝒓𝒆𝒔 𝒐𝒇 𝒕𝒉𝒆
𝒄𝒐𝒐𝒓𝒅𝒊𝒏𝒂𝒕𝒆𝒔 𝒐𝒇 𝒕𝒉𝒆 𝒕𝒓𝒆𝒆

Example 1.102: Equidistant points


A. Find the point 𝐴 on the 𝑦-axis that is equidistant from the points 𝑋(3, −3) and 𝑌(−2,2).
B. Find the point 𝐴 on the 𝑦-axis that is equidistant from the points 𝑀(−1,5) and 𝑁(4, −7).
C. Point 𝐴 lies on the 𝑥-axis at the same distance from 𝑃(−2, 0) and 𝑄(0,4). Find the coordinates of 𝐴.
D. Point 𝑃 is the point on the 𝑥-axis equidistant from 𝐴(4, −2) and 𝐵(1,5). Find the coordinates of 𝑃.
E. Point 𝑃(𝑎 − 3, 𝑎 + 2) is equidistant from the points (4,1) and (5,2). Find the coordinates of P.

Part A form (𝑥, 0):


Any point on the 𝑦 −axis must have coordinates 𝑃𝐴
⏟ = 𝑄𝐴

(0, 𝑎). 𝑫𝒊𝒔𝒕𝒂𝒏𝒄𝒆 𝑫𝒊𝒔𝒕𝒂𝒏𝒄𝒆
𝒇𝒓𝒐𝒎 𝑷 𝒕𝒐 𝑨 𝒇𝒓𝒐𝒎 𝑸 𝒕𝒐 𝑨
Since point A is equidistant from 𝑋 and 𝑌:
Substitute 𝑃(−2, 0) in the LHS, and 𝑄(0,4) in the
𝐴𝑋
⏟ = 𝐴𝑌

𝑫𝒊𝒔𝒕𝒂𝒏𝒄𝒆 𝑫𝒊𝒔𝒕𝒂𝒏𝒄𝒆
RHS and use the distance formula:
𝒇𝒓𝒐𝒎 𝑨 𝒕𝒐 𝑿 𝒇𝒓𝒐𝒎 𝑨 𝒕𝒐 𝒀 √(𝑥 + 2)2 + (0 − 0)2 = √(𝑥 − 0)2 + (0 − 4)2
Use the distance formula: Simplify, and square both sides:
√(0 − 3)2 + (𝑎 + 3)2 = √(0 + 2)2 + (𝑎 − 2)2 𝒙𝟐 + 4𝑥 + 4 = 𝒙𝟐 + 16
Square both sides and simplify: Solve for 𝑥:
9 + 𝑎2 + 6𝑎 + 9 = 4 + 𝑎2 − 2𝑎 + 4 4𝑥 = 12 ⇒ 𝑥 = 3 ⇒ 𝐴 = (3,0)
Solve for 𝑎: Part D
10 5 Since the point is on the 𝑥-axis, it must be of the
8𝑎 + 18 = 8 ⇒ 8𝑎 = −10 ⇒ 𝑎 = − =−
8 4 form (𝑥, 0):
Part B
Any point on the 𝑦 −axis must have coordinates
(0, 𝑎). Part E
Since point 𝐴 is equidistant from 𝑀(−1,5) and 𝑃𝑄 2 = (𝑎 − 3 − 4)2 + (𝑎 + 2 − 1)2
𝑁(4, −3): = (𝑎 − 7)2 + (𝑎 + 1)2
𝐴𝑀
⏟ = 𝐴𝑁
⏟ = 𝑎 − 14𝑎 + 49 + 𝑎2 + 2𝑎 + 1
2
𝑫𝒊𝒔𝒕𝒂𝒏𝒄𝒆 𝑫𝒊𝒔𝒕𝒂𝒏𝒄𝒆
𝒇𝒓𝒐𝒎 𝑨 𝒕𝒐 𝑴 𝒇𝒓𝒐𝒎 𝑨 𝒕𝒐 𝑵 = 2𝑎2 − 12𝑎 + 50
Use the distance formula:
√(0 + 1)2 + (𝑎 − 5)2 = √(0 − 4)2 + (𝑎 + 7)2 𝑃𝑅 2 = (𝑎 − 3 − 5)2 + (𝑎 + 2 − 2)2
Square both sides and simplify: = (𝑎 − 8)2 + 𝑎2
1 + 𝑎2 − 10𝑎 + 25 = 16 + 𝑎2 + 14𝑎 + 49 = 2𝑎2 − 16𝑎 + 64
Solve for 𝑎:
39 ⏟ 2 − 12𝑎 + 50 = ⏟
2𝑎 2𝑎2 − 16𝑎 + 64
−10𝑎 + 26 = 14𝑎 + 65 ⇒ 4𝑎 = 39 ⇒ 𝑎 = 𝑷𝑸𝟐 𝑷𝑹𝟐
4
Part C 14 7
4𝑎 = 14 ⇒ 𝑎 = =
Since the point is on the 𝑥-axis, it must be of the 4 2

Example 1.103: Finding the coordinates of a point


1
The distance between (𝑥, 4) and (7, 2 ) is 6. Find the sum of the possible values of 𝑥.

Substitute the known values in the distance formula √(𝑥2 − 𝑥1 )2 + (𝑦2 − 𝑦1 )2 :

P a g e 37 | 95
Get all the files at: https://bit.ly/azizhandouts
Aziz Manva ([email protected])

1 2
√(𝑥 − 7)2 + (4 − ) = 6
2
Square both sides:
49
(𝑥 − 7)2 + = 36
4
Isolate (𝑥 − 7)2 on the left-hand side:
49 95
(𝑥 − 7)2 = 36 − ⇒ (𝑥 − 7)2 =
4 4
Take the square root both sides, and solve for 𝑥:
95 √95
𝑥 − 7 = ±√ ⇒ 𝑥 = 7 ±
4 2
Find the sum:
√95 √95
(7 + ) + (7 − ) = 14
2 2

Example 1.104: Finding the coordinates of a point


The distance between (4,5) and (6, 𝑦) is 12. Find the product of the possible values of 𝑦.

Substitute the known values in the distance formula √(𝑥2 − 𝑥1 )2 + (𝑦2 − 𝑦1 )2 :


√(6 − 4)2 + (𝑦 − 5)2 = 12
Square both sides, and simplify within the square root:
(2)2 + (𝑦 − 5)2 = 144
Isolate the (𝑦 − 5)2 term, and take square roots both sides:
(𝑦 − 5)2 = 140 ⇒ 𝑦 − 5 = ±√140
Simplify the right-hand side, and then solve for 𝑦:
𝑦 − 5 = ±(√4 × √35) ⇒ 𝑦 − 5 = ±2√35 ⇒ 𝑦 = 5 ± 2√35
𝑃𝑟𝑜𝑑𝑢𝑐𝑡 = (5 + 2√35)(5 − 2√35)
This is of the form (𝑎 + 𝑏)(𝑎 − 𝑏) = 𝑎2 − 𝑏 2 :
2
52 − (2√35) = 25 − 4 × 35 = 25 − 140 = −115

Example 1.105: Finding the coordinates of a point


The points (𝑎, 2,3) and (2,4,5) have a distance of 5 between them. Find the possible values of 𝑎.

√(𝑎 − 2)2 + (2 − 4)2 + (3 − 5)2 = 5


(𝑎 − 2)2 + 4 + 4 = 25
(𝑎 − 2)2 = 17
𝑎 − 2 = ±√17
𝑎 = 2 ± √17

Example 1.106: Finding the coordinates of a point


A. Find the coordinates of the point 𝑋 on the 𝑧 axis equidistant from 𝑃(3,1,4) and 𝑄(2,7,1).
B. Find the coordinates of the point on the 𝑥 axis is twice as far from (2,4, −3) as it is from (1,6,3).

Any point on the 𝑧 axis must be of the form (0,0, 𝑧):


𝑋𝑃 = 𝑋𝑄

P a g e 38 | 95
Get all the files at: https://bit.ly/azizhandouts
Aziz Manva ([email protected])

√(3 − 0)2 + (1 − 0)2 + (4 − 𝑧)2 = √(2 − 0)2 + (7 − 0)2 + (1 − 𝑧)2


9 + 1 + (𝑧 2 − 8𝑧 + 16) = 4 + 49 + (𝑧 2 − 2𝑧 + 1)
−8𝑧 + 26 = −2𝑧 + 44
−6𝑧 = 28
18
𝑧=− = −3
6

Challenge 1.107: Finding the coordinates of a point


The points (𝑎, −3,2) and (4, 𝑏, 5) have a distance of 7 between them. If 𝑏 is twice the absolute value of 𝑎, find
the sum of all possible values of 𝑎.

𝑏 = 2|𝑎|
Case I: 𝒂 ≥ 𝟎 ⇒ 𝒃 = 𝟐𝒂
√(4 − 𝑎)2 + (2𝑎 + 3)2 + (5 − 2)2 = 7
(𝑎2
− 8𝑎 + 16) + (4𝑎2 + 12𝑎 + 9) + 9 = 49
5𝑎2 + 4𝑎 − 15 = 0
Solve the above quadratic using 𝑎 = 5, 𝑏 = 4, 𝑐 = −15:
−4 ± √16 − (4)(5)(−15) −4 ± √316 −4 ± 2√79 −2 ± √79
𝑎= = = =
2(5) 10 10 5
−2 + √79
⇒ +𝒗𝒆
5
−2 − √19 ⇒ −𝒗𝒆 ⇒ 𝑪𝒐𝒏𝒕𝒓𝒂𝒅𝒊𝒄𝒕𝒊𝒐𝒏
𝑏 4
𝑆𝑢𝑚 𝑜𝑓 𝑉𝑎𝑙𝑢𝑒𝑠 = − −
𝑎 5
Case II: 𝒂 < 𝟎 ⇒ 𝒃 = −𝟐𝒂

√(4 − 𝑎)2 + (−2𝑎 + 3)2 + (5 − 2)2 = 7


(𝑎2 − 8𝑎 + 16) + (4𝑎2 − 12𝑎 + 9) + 9 = 49
5𝑎2 − 20𝑎 − 15 = 0
𝑎2 − 4𝑎 − 3 = 0
Solve the above quadratic using 𝑎 = 1, 𝑏 = −4, 𝑐 = −3:
4 ± √16 − (4)(1)(−3) 4 ± √28 4 ± 2√7
𝑎= = = = 2 ± √7
2(1) 2 2
2 + √7 > 0 ⇒ +𝒗𝒆 ⇒ 𝑪𝒐𝒏𝒕𝒓𝒂𝒅𝒊𝒄𝒕𝒊𝒐𝒏
2 − √4 = 2 − 2 = 0 ⇒ 2 − √7 < 0 ⇒ −𝒗𝒆

The final answer is:


−2 + √79 8 + √79 − 5√7
+ 2 − √7 =
5 5

Challenge 1.108: Finding the coordinates of a point


Recall that the floor function 𝑓(𝑥) = ⌊𝑥⌋ gives the largest value of 𝑥 that is the left of 𝑥 on the real number line.
Find the value of 𝑥 given that
A. the points (⌊𝑥⌋, 1,6) and (2, ⌊𝑥⌋, 4) have a distance of √21 between them.
B. the points (⌊𝑥⌋, 1,6) and (2, ⌊𝑥⌋, 4) have a distance of √17 between them.

Part A

P a g e 39 | 95
Get all the files at: https://bit.ly/azizhandouts
Aziz Manva ([email protected])

Use a change of variable. Let 𝑦 = ⌊𝑥⌋:


√(𝑦 − 2)2 + (1 − 𝑦)2 + (6 − 4)2 = √21
(𝑦 2 − 4𝑦 + 4) + (𝑦 2 − 2𝑦 + 1) + 4 = 21
2𝑦 2 − 6𝑦 − 12 = 0
𝑦 2 − 3𝑦 − 6 = 0
Solve the above quadratic using 𝑎 = 1, 𝑏 = −3, 𝑐 = −6:
3 ± √9 − (4)(1)(−6) 3 ± √33
𝑦= =
2(1) 2
3 ± √33
⌊𝑥⌋
⏟ = ⇒ 𝑪𝒐𝒏𝒕𝒓𝒂𝒅𝒊𝒄𝒕𝒊𝒐𝒏 ⇒ 𝑵𝒐 𝑺𝒐𝒍𝒖𝒕𝒊𝒐𝒏𝒔
⏟ 2
𝐼𝑛𝑡𝑒𝑔𝑒𝑟
𝑁𝑜𝑡 𝑎𝑛 𝐼𝑛𝑡𝑒𝑔𝑒𝑟
Part B
√(𝑦 − 2)2 + (1 − 𝑦)2 + (6 − 4)2 = √17
(𝑦 2 − 4𝑦 + 4) + (𝑦 2 − 2𝑦 + 1) + 4 = 17
2𝑦 2 − 6𝑦 − 8 = 0
𝑦 2 − 3𝑦 − 4 = 0
(𝑦 − 4)(𝑦 + 1) = 0

𝑦 = 4 ⇒ ⌊𝑥⌋ = 4 ⇒ 4 ≤ 𝑥 < 5 ⇒ 𝑥 ∈ [4,5)


𝑦 = −1 ⇒ ⌊𝑥⌋ = −1 ⇒ −1 ≤ 𝑥 < 0 ⇒ 𝑥 ∈ [−1,0)
Hence, the final answer is:
𝑥 ∈ [−1,0) ∪ [4,5)
D. Geometrical Applications

Example 1.109
Wanda is trying to locate the Fermat point 𝑃 of Δ𝐴𝐵𝐶, where 𝐴 is at the origin, 𝐵 is at (8, −1), and 𝐶 is at (5,4)
(the Fermat point is the point such that the sum of its distances from the vertices of a triangle is minimized).
She guesses that the point is at 𝑃 = (4,2), and computes the sum of the distances from 𝑃 to the vertices of
Δ𝐴𝐵𝐶. If she obtains 𝑚 + 𝑛√5, where 𝑚 and 𝑛 are integers, what is 𝑚 + 𝑛? (AOPS Alcumus, Algebra, Distance in
the Plane)

𝑃𝐴 = √42 + 22 = √20 = 2√5


𝑃𝐵 = √(4 − 8)2 + (2 + 1)2 = √16 + 9 = √25 = 5
𝑃𝐶 = √(4 − 5)2 + (2 − 4)2 = √1 + 22 = √5

𝑃𝐴 + 𝑃𝐵 + 𝑃𝐶 = 5 + 3√5 ⇒ 𝑚 + 𝑛 = 5 + 3 = 8

Example 1.110
Consider
𝛥𝑃𝑄𝑅 𝑤𝑖𝑡ℎ 𝑃 = (3, −4), 𝑄 = (−2, −5), 𝑅 = (2,1)
A. The perimeter of the triangle can be written as 𝑎(√𝑏 + √𝑐), where a is a natural number, and b and c
are natural numbers with no perfect square factor. Find 𝑎 + 𝑏 + 𝑐.
B. Classify the triangle. (If more than one classification fits, use them all).

P a g e 40 | 95
Get all the files at: https://bit.ly/azizhandouts
Aziz Manva ([email protected])

𝑃𝑄 = √(3 + 2)2 + (−4 + 5)2 = √25 + 1 = √26


𝑄𝑅 = √(−2 − 2)2 + (−5 − 1)2 = √16 + 36 = √52 = 2√13
𝑃𝑅 = √(3 − 2)2 + (−4 − 1)2 = √1 + 25 = √26

Part A
𝑃𝑒𝑟𝑖𝑚𝑒𝑡𝑒𝑟 = 2(√13 + √26)
Part B
Note that:
2 2 2
𝑃𝑄 2 + 𝑃𝑅 2 = (√26) + (√26) = 26 + 26 = 52 = (√52) = 𝑄𝑅 2
Since 𝑃𝑄 2 + 𝑃𝑅 2 = 𝑄𝑅 2
∴ Δ𝑃𝑄𝑅 𝑖𝑠 𝑟𝑖𝑔ℎ𝑡 − 𝑎𝑛𝑔𝑙𝑒𝑑 𝑏𝑦 𝐶𝑜𝑛𝑣𝑒𝑟𝑠𝑒 𝑜𝑓 𝑃𝑦𝑡ℎ𝑎𝑔𝑜𝑟𝑒𝑎𝑠 𝑇ℎ𝑒𝑜𝑟𝑒𝑚

Draw a diagram. From the diagram:


∠𝑃 𝑖𝑠 𝑡ℎ𝑒 𝑟𝑖𝑔ℎ𝑡 − 𝑎𝑛𝑔𝑙𝑒

Also, the triangle is isosceles.


∴ 𝛥𝑃𝑄𝑅 𝑖𝑠 𝑎 45 − 45 − 90 𝑡𝑟𝑖𝑎𝑛𝑔𝑙𝑒

Example 1.111
What is the second smallest number of sides whose length must be changed (and by how much), to make the
triangle Δ𝑃𝑄𝑅 with 𝑃 = (√2, 0), 𝑄 = (−√2, 0), 𝑅 = (0, −√5) equilateral?

2 2 2
𝑃𝑄 = √[√2 − (−√2)] + (0 − 0)2 = √[√2 + √2] = √[2√2] = 2√2
2 2 2 2
𝑄𝑅 = √(−√2 − 0) + [0 − (−√5)] = √(−√2) + (√5) = √2 + 5 = √7
2 2 2 2
𝑃𝑅 = √(√2 − 0) + [0 − (−√5)] + √(√2) + (√5) = √2 + 5 = √7

𝑄𝑅 = 𝑃𝑅 ≠ 𝑃𝑄 ⇒ Δ𝑃𝑄𝑅 𝑖𝑠 𝑖𝑠𝑜𝑠𝑐𝑒𝑙𝑒𝑠

The smallest number of sides that must be changed is:


1

Second smallest number of sides that must be changed is:


2

Decrease by
2√2 − √7

Example 1.112
If 𝐴𝐶 = 𝑝 and 𝐵𝐷 = 𝑞, find the area of quadrilateral 𝐴𝐵𝐶𝐷 with 𝐴(𝑥1 , 𝑦1 ), 𝐵(𝑥2 , 𝑦2 ), 𝐶(𝑥3 , 𝑦3 ), 𝐷(𝑥4 , 𝑦4 ) and
𝐴𝐵 = 𝐵𝐶 = 𝐶𝐷 = 𝐷𝐴 = 𝑑

Since all sides of the quadrilateral are equal, it must be a rhombus.


𝑃𝑟𝑜𝑑𝑢𝑐𝑡 𝑜𝑓 𝐷𝑖𝑎𝑔𝑜𝑛𝑎𝑙𝑠 𝑝𝑞
𝐴𝑟𝑒𝑎 𝑜𝑓 𝑅ℎ𝑜𝑚𝑏𝑢𝑠 = =
2 2
P a g e 41 | 95
Get all the files at: https://bit.ly/azizhandouts
Aziz Manva ([email protected])

E. Conceptual Applications
Along with other things, conceptual applications focus on visualization of possible locations of points that meet
criteria for geometrical shapes given in the question.

Example 1.113
Points 𝑃(5, −4) and 𝑄(9, −4) are two vertices of rectangle 𝑃𝑄𝑅𝑆 whose area is 20 square units. What are the
possible values of the co-ordinates of 𝑅 and 𝑆?

𝐴(𝑃𝑄𝑅𝑆) 20
𝑃𝑄 = 9 − 4 = 5 ⇒ 𝑃𝑅 = = =4
𝑃𝑄 5
𝑆 has to be either 4 units above or below 𝑃 ⇒ 𝑆 = {(5,0), (5, −8)}
𝑅 has to be either 4 units above or below 𝑃 ⇒ 𝑅 = {(9,0), (9, −8)}

Example 1.114
A. The center of a circle has coordinates 𝑃(5,7). The point 𝑄(2,1) lies on the
boundary of the circle. Find the area of the circle in terms of 𝜋.
B. 𝐴(3,6) and 𝐵(−2,4) are the endpoints of the chord of maximum length of a
circle. Find the area of the circle.

Part A
The radius is the distance between the center of the circle, and the boundary of the circle:
𝑅𝑎𝑑𝑖𝑢𝑠 = 𝑃𝑄 = √(5 − 2)2 + (7 − 1)2 = √32 + 62 = √9 + 36 = √45
2
𝐴 = 𝜋𝑟 2 = 𝜋(√45) = 45𝜋
Part B
Diameter = 𝐴𝐵 = √(3 + 2)2 + (6 − 4)2 = √25 + 4 = √29
2
2 √29 29
Area = 𝜋𝑟 = 𝜋 ( ) = 𝜋
2 4

Example 1.115: Area of a Square


𝐴(2,3) and 𝐵(4,5) are vertices of a square. Find the difference in the possible areas of the square.

First, find the distance between the two points using the distance formula:
𝐴𝐵 = √(2 − 4)2 + (3 − 5)2 = √4 + 4 = √8

Now, there are two cases to consider:


Case I: AB is the side of the square:
2
𝐴𝑟𝑒𝑎 = 𝐴𝐵2 = (√8) = 8

Case II: AB is the diagonal of the square:


2
𝐴𝐵2 (√8) 8
𝐴𝑟𝑒𝑎 = = = =4
2 2 2
Difference = 𝐴(Side)
⏟ − 𝐴(Diagonal)
⏟ =8−4 = 4
𝑨𝒅𝒋𝒂𝒄𝒆𝒏𝒕 𝑽𝒆𝒓𝒕𝒊𝒄𝒆𝒔 𝑵𝒐𝒏−𝑨𝒅𝒋𝒂𝒄𝒆𝒏𝒕 𝑽𝒆𝒓𝒕𝒊𝒄𝒆𝒔

Example 1.116: Area of a Kite

P a g e 42 | 95
Get all the files at: https://bit.ly/azizhandouts
Aziz Manva ([email protected])

1.117: Minimum sum of distances to vertices in a convex


quadrilateral
In a convex quadrilateral, the point of intersection of the diagonals minimizes
the sum of the distance to its vertices.
Hence, the minimum distance is given by the sum of the length of the diagonals.

In Quadrilateral 𝐴𝐵𝐶𝐷, let the point of intersection of the diagonals be 𝑂, and 𝑃


be any point not 𝑂.

By the triangle inequality:


𝐴𝐶 < 𝑃𝐴 + 𝑃𝐶 ,
⏟ 𝐵𝐷 < 𝑃𝐵 + 𝑃𝐷

𝑰𝒏𝒆𝒒𝒖𝒂𝒍𝒊𝒕𝒚 𝑰 𝑰𝒏𝒆𝒒𝒖𝒂𝒍𝒊𝒕𝒚 𝑰𝑰
Add Inequality I and II:
𝐴𝐶 + 𝐵𝐷 < 𝑃𝐴 + 𝑃𝐵 + 𝑃𝐶 + 𝑃𝐷
𝑂𝐴 + 𝑂𝐵 + 𝑂𝐶 + 𝑂𝐷 < 𝑃𝐴 + 𝑃𝐵 + 𝑃𝐶 + 𝑃𝐷
Hence proved

Example 1.118: Point equidistant from Vertices


Points 𝐴, 𝐵, 𝐶, and 𝐷 are in a Cartesian plane such that 𝐴 = (2048, 2058), 𝐵 = (2018, 2018),
𝐶 = (2036, 2046), and 𝐷 = (2060, 2039). Given that point 𝑃 is in the same plane, the minimum
possible value of 𝑃𝐴 + 𝑃𝐵 + 𝑃𝐶 + 𝑃𝐷 is closest to which integer? (AOPS/Stormersyle Mock AMC 8 2018/20)

The underlying concept is simple, but the question is high quality because:
➢ The values of the coordinates are large, leading to information overload
➢ The minimum value is actually an integer, but the question does not give this away.
Change of Origin
We introduce a new pair of coordinate axes at the location (2018,2018) to get “nicer” numbers:
𝐴 = (2048 − 2018, 2058 − 2018) = (30,40)
𝐵 = (2018, 2018) = (0,0)
𝐶 = (2036, 2046) = (18,28)
𝐷 = (2060, 2039) = (42,21)
Form a convex quadrilateral
The points 𝐵𝐶𝐴𝐷, in that order, form a convex quadrilateral. This can be checked since BA lies on the line
4
𝑦= 𝑥
3
And point C lies above the line, whereas point D lies below the line.
Find the minimum distance
𝐵𝐴 + 𝐶𝐷 = √(30 − 0)2 − (40 − 0)2 + √(42 − 18)2 + (21 − 28)2 = 50 + 25 = 75
Shortcut:
The last step can be done much faster (and possibly more accurately) if you know your Pythagorean Triplets:
(30,40, 𝑙1 ) = 10(3,4, 𝑥) = 10(3,4,5) = (30,40,50)
(42 − 18,28 − 21, 𝑙2 ) = (24,7, 𝑙2 ) = (24,7,25)
And finally
50 + 25 = 75

P a g e 43 | 95
Get all the files at: https://bit.ly/azizhandouts
Aziz Manva ([email protected])

1.8 Midpoint and Section Formula


A. Midpoint on the Number Line

1.119: Midpoint on the Number Line


The midpoint of the line segment connecting two points 𝑥1 and 𝑥2 on the number line is given by:
𝑥1 + 𝑥2
𝑀𝐴𝐵 =
2

Example 1.120
Find the midpoint of the following numbers on the real number line:
A. 3 𝑎𝑛𝑑 4
B. −2 𝑎𝑛𝑑 2
C. −3 𝑎𝑛𝑑 7
D. 0.3 and 0.01
1 1
E. 2
𝑎𝑛𝑑 3

3+4 7
=
2 2
−2 + 2 0
= =0
2 2
−3 + 7 4
= =2
2 2

Example 1.121
Points P,Q and R are on a number line. Q is halfway between P and R. If P is at -6 and Q is at -1, then R is at
(CEMC Gauss 7 2016/10)

Example 1.122
Suzy's 5m long ribbon has shaded and unshaded sections
of equal length, as shown. Points A,B,C,D,E are equally
spaced along the ribbon. If Suzy wants a ribbon that is
11/15 of the size of this ribbon, at which point could she make a single vertical cut? (CEMC Gauss 7 2016/19)

B. Midpoint on the Coordinate Plane

1.123: Midpoint on the Coordinate Plane


The midpoint of the line segment connecting two points 𝐴(𝑥1 , 𝑦1 ) and 𝐵(𝑥2 , 𝑦2 ) is given by:
𝑥1 + 𝑥2 𝑦1 + 𝑦2
𝑀𝐴𝐵 = ( , )
2 2

The 𝑥 coordinate of the midpoint will be given by 𝑥1 added to half the distance of the 𝑥 coordinates of the two
points:
𝑥2 − 𝑥1 2𝑥1 + 𝑥2 − 𝑥1 𝑥1 + 𝑥2
𝑀𝑥 = 𝑥1 + = =
2 2 2
Similarly:

P a g e 44 | 95
Get all the files at: https://bit.ly/azizhandouts
Aziz Manva ([email protected])

𝑦1 + 𝑦2
𝑀𝑦 =
2

Example 1.124
For each part below, find the midpoint of the line segment connecting the points. Also, state the quadrant in
which the midpoint lies.
A. (2,3) and (6,9)
B. (−5,7) and (12, −8)
1 2 7 5
C. (3 , 5) and (9 , 7)
3 3 3 2
D. (4 , − 7) and (− 7 , 11)

2+6 3+9 8 12
(, ) = ( , ) = (4,6) ⇒ 𝐿𝑖𝑒𝑠 𝑖𝑛 𝑄 − 𝐼
2 2 2 2
−5 + 12 7 − 8 7 −1
( , ) = ( , ) ⇒ 𝐿𝑖𝑒𝑠 𝑖𝑛 𝑄 − 𝐼𝑉
2 2 2 2
1 7 2 5 10 39
+ + 10 1 39 1 5 39
(3 9 , 5 7) = ( 9 , 35 ) = ( × , × ) = ( , ) ⇒ 𝐿𝑖𝑒𝑠 𝑖𝑛 𝑄 − 𝐼
2 2 2 2 9 2 35 2 9 70
3 3 3 2 9 −19
− − + 28 9 19
( 4 7 , 7 11 ) = ( , 77 ) = ( , − )
2 2 2 2 56 154

Example 1.125
Find 𝑋 + 𝑌 given that:
➢ 𝑋 = Sum of the coordinates of the midpoint of the line segment connecting (3, 4) and (7, 12).
3 4 1 2
➢ 𝑌= Product of the coordinates of the midpoint of the line segment connecting (− 2 , 5) and (3 , − 7).

3 + 7 4 + 12
( , ) = (5,8) ⇒ 𝑋 = 5 + 8 = 13
2 2
3 1 4 2 7 18
−2 + 3 5 − 7 − 6 35 7 9 7 9 3
( , )=( , ) = (− , ) ⇒ 𝑌 = − × =−
2 2 2 2 12 35 12 35 20
3 17
𝑋 + 𝑌 = 13 − = 12
20 20

Example 1.126: Back-Calculations


The midpoint of 𝐴(𝑥, 𝑦) and 𝐵(2,3) is (1, 4). Find the co-ordinates of Point 𝐴.
𝑥+2
=1⇒𝑥+2=2⇒𝑥 =0
2
𝑦+3
= 4 ⇒ 𝑦 + 3 = 8 ⇒ 𝑦 = 5 ⇒ 𝐴 = (0,5)
2

C. Geometrical Applications

Example 1.127: Circle

P a g e 45 | 95
Get all the files at: https://bit.ly/azizhandouts
Aziz Manva ([email protected])

The points (2, 3) and (7,10) lie on the endpoints of the diameter of a circle. Find the positive difference of the
coordinates of the 𝑐𝑒𝑛𝑡𝑟𝑒
⏟ of the circle.
𝑪

Recall that the center of a circle is the midpoint of the diameter of the circle.
2 + 7 3 + 10 9 13
𝐶=( , ) = ( , ) = (4.5,6.5) ⇒ Difference = 6.5 − 4.5 = 2
2 2 2 2

Example 1.128: Triangle


Δ𝐴𝐵𝐶 has vertices 𝐴(1,2), 𝐵(−2, −1) and 𝐶(−3,4). Find the co-ordinates of the vertices of the triangle formed
by the midpoints of Δ𝐴𝐵𝐶.

1 1 −5 3
(− , ) , ( , ) , (−1,3)

⏟ 2 2 ⏟2 2
𝑀(𝐶𝐴)
𝑀(𝐴𝐵) 𝑀(𝐵𝐶)

Example 1.129
In Δ𝐴𝐵𝐶 from the previous example, find the midpoint of the median from the vertex 𝐴.

Draw median AD, intersecting 𝐵𝐶 at D.


−5 3
𝐷 = 𝑀𝑖𝑑𝑝𝑜𝑖𝑛𝑡 𝑜𝑓 𝐵𝐶 = ( , )
2 2

Now, we can find the midpoint of the median as:


3 7
𝑀𝑖𝑑𝑝𝑜𝑖𝑛𝑡(𝐴𝐷) = (− , )
4 4

Example 1.130
Find the point of intersection of the diagonals of the square 𝐴𝐵𝐶𝐷, which has vertices 𝐴( ), 𝐵( ), 𝐶( ) and
𝐷( ).

Example 1.131
Prove that the midpoint of the line segment connecting two points 𝐴(𝑥1 , 𝑦1 , 𝑧1 ) and 𝐵(𝑥2 , 𝑦2 , 𝑧2 ) is given by:
𝑥1 + 𝑥2 𝑦1 + 𝑦2 𝑧1 + 𝑧2
Midpoint = 𝑀 = ( , , )
2 2 2

D. Algebraic Applications

E. Section Formula: Internal Division


Background
The midpoint of a line segment divides the line segment into two equal parts. In other words, the midpoint
divides a line segment in the ratio 1: 1.
This concept of a midpoint can be generalised to have a point that divides a line segment in the ratio 𝑚: 𝑛.
Definition
1
Let 𝑨𝑩 = 𝟔. Let point 𝑀 be 3rd of the way between 𝐴 and 𝐵. ⇒ 𝑨𝑴 = 𝟐, 𝑴𝑩 = 𝟒
A 2 units 𝑀 4 units B

Point 𝑴 divides 𝑨𝑩 internally in the ratio 𝑨𝑴: 𝑴𝑩 = 𝒎: 𝒏 = 𝟐: 𝟒 = ⏟


𝟏 : ⏟
𝟐 .
𝒎=𝟏 𝒏=𝟐

P a g e 46 | 95
Get all the files at: https://bit.ly/azizhandouts
Aziz Manva ([email protected])

1.132: Section Formula for Internal Division


Prove that the coordinates of the point 𝑀 that divides the points 𝐴(𝑥1 , 𝑦1 ) and 𝐵(𝑥2 , 𝑦2 ) internally in the ratio
𝑚: 𝑛 are given by:
𝑚𝑥2 + 𝑛𝑥1 𝑚𝑦2 + 𝑛𝑦1
𝑀=( , )
𝑚+𝑛 𝑚+𝑛
Proof
Consider the right-angled triangles formed by dropping perpendiculars from point 𝑀, and point 𝐵 to lines
parallel to the 𝑥-axis drawn from point 𝐴 and 𝑀, respectively.

These right-angled triangles are similar.

𝑚
Point 𝑀 is times the distance from 𝐴 to 𝐵.
𝑚+𝑛
𝑚 (𝑚 + 𝑛)(𝑥1 ) + (𝑚)(𝑥2 − 𝑥1 ) 𝑚𝑥2 + 𝑛𝑥1
∴ 𝑥 = 𝑥1 + (𝑥2 − 𝑥1 ) = =
𝑚+𝑛 𝑚+𝑛 𝑚+𝑛
Similarly, for 𝑦.

Example 1.133: Internal Division


2
Point 𝐶 is 3 rd of the way between 𝐴(6,9) and 𝐵(4,0). Find 𝐶.

Method I: Using the Section Formula


Point 𝐶 divides 𝐴𝐵 in the ratio 2: 1.
Substitute 𝑚 = 2, 𝑛 = 1 in the Section Formula:
𝑚𝑥2 + 𝑛𝑥1 𝑚𝑦2 + 𝑛𝑦1 2×4+1×6 2×0+1×9 8+6 0+9 14
𝐶=( , )=( , )=( , ) = ( , 3)
𝑚+𝑛 𝑚+𝑛 2+1 2+1 3 3 3

Method II: Using Logic


Start from x coordinate 6.

Distance between 6 and 4 is


−2

2
3
𝑟𝑑 of this distance is
2 4
−2 × =−
3 3

Hence, we need to add the above distance to the starting point:


4 18 4 14
6 + (− ) = − =
3 3 3 3

And we can do the same thing for the y coordinate


2
9 + (−9 × ( )) = 9 − 6 = 3
3

Example 1.134: Internal Division


Point 𝑋 divides 𝑃(−3,4), and 𝑄(2, −7) in the ratio 3: 5. Find 𝑋.

P a g e 47 | 95
Get all the files at: https://bit.ly/azizhandouts
Aziz Manva ([email protected])

𝑚𝑥2 + 𝑛𝑥1 𝑚𝑦2 + 𝑛𝑦1 3 × 2 + 5 × (−3) 3 × (−7) + 5 × 4 −9 −1


𝑋=( , )=( , )=( , )
𝑚+𝑛 𝑚+𝑛 3+5 2+1 8 8

Example 1.135: Back Calculations


Point 𝑃(2,3) divides 𝑀(1,2), and 𝑄 in the ratio 5: 7. Find 𝑄.

5𝑥2 + 7 × 1 5𝑦2 + 7 × 2
S1: (2,3) = ( , )
5+7 5+7
5𝑥2 + 7 17
2= ⇒ 24 = 5𝑥2 + 7 ⇒ 𝑥2 =
12 5
5𝑦2 + 14 22
3= ⇒ 36 = 5𝑦2 + 14 ⇒ 𝑦2 =
12 5

F. Section Formula: External Division


Writing Assignment 1.1
Prove that the coordinates of the point that divides the points 𝐴(𝑥1 , 𝑦1 ) and 𝐵(𝑥2 , 𝑦2 ) externally in the ratio
𝑚: 𝑛 are given by:
𝑚𝑥1 − 𝑛𝑥2 𝑚𝑦1 − 𝑛𝑦2
𝑀=( , )
𝑚+𝑛 𝑚+𝑛

Example 1.136: External Division


Perimeter of a Polygon
Q1: F
S2: S

1.9 Triangles
A. Centroid
Revision
1. The median is the line joining the vertex of a triangle to the midpoint of its opposite side. Every triangle
has three midpoints.
2. If three or more lines intersect at a point, the lines are called concurrent.

Definition
The point of concurrency (intersection) of the three medians of a triangle is its centroid.

Property
The centroid of a triangle divides the median in the ratio 2: 1

1.137: Centroid of a Triangle

P a g e 48 | 95
Get all the files at: https://bit.ly/azizhandouts
Aziz Manva ([email protected])

A triangle has vertices 𝐴(𝑥1 , 𝑦1 ), 𝐵(𝑥2 , 𝑦2 ) and 𝐶(𝑥3 , 𝑦3 ). Show that the:
𝑥1 +𝑥2 𝑦1 +𝑦2
A. Midpoint of 𝐴𝐵 = 𝑀 = ( 2
, 2 )
𝑥 +𝑥 +𝑥 𝑦 +𝑦 +𝑦
B. Centroid = 𝐺 = ( 1 22 3 , 1 22 3 )
Part A
𝑥1 + 𝑥2 𝑦1 + 𝑦2
Midpoint of 𝐴𝐵 = 𝑀 = ( , )
2 2
Part B
Centroid divides C𝑀 in the ratio 2: 1. Hence, using the
section formula with 𝑚 = 2, 𝑛 = 1:
𝑥 + 𝑥2 𝑦 +𝑦
2[ 1
2 ] + 𝑥3 2 [ 1 2 2 ] + 𝑦3
Centroid = 𝐺 = ( , )
2+1 2+1
𝑥1 + 𝑥2 + 𝑥3 𝑦1 + 𝑦2 + 𝑦3
=( , )
3 3

Example 1.138: Centroid of a Triangle


Perimeter of a Polygon
Q2: F
S3: S

B. Incentre
Writing Assignment 1.2 (Formula for Incentre of a Triangle)
Show that the incentre of a triangle with vertices 𝐴(𝑥1 , 𝑦1 ), 𝐵(𝑥2 , 𝑦2 ) and 𝐶(𝑥2 , 𝑦2 ) is:
𝑎𝑥1 + 𝑏𝑥2 + 𝑐𝑥3 𝑎𝑦1 + 𝑏𝑦2 + 𝑐𝑦3
𝐺=( , )
𝑎+𝑏+𝑐 𝑎+𝑏+𝑐

C. Circumcenter

1.139: Circumcenter of a Triangle


Circumcenter of a triangle is the point of concurrency of the perpendicular bisectors of the sides

Find the midpoint of a side.


Find the line perpendicular to the side.
Same way, find the line perpendicular to second side.
Finally find the point of intersection of the two lines.

Example 1.140: Incentre of a Triangle


Perimeter of a Polygon
Q3: F
S4: S

P a g e 49 | 95
Get all the files at: https://bit.ly/azizhandouts
Aziz Manva ([email protected])

1.10 Area
A. Area Basics

Example 1.141
The coordinates of the vertices of rectangle PQRS are given in the diagram. The
area of rectangle PQRS is 120. The value of p is (CEMC Gauss 8 2000/13)

Example 1.142
In the diagram, point F has coordinates (6,6). Points P and Q are two vertices of a
triangle. Which of the following poin ts can be joined to P and Q to create a triangle with
an area of 6? (CEMC Gauss 8 2019/9)

Example 1.143
A line segment is drawn joining the points (0,6) and (4,0), as shown. The area of the shaded
triangle is (CEMC Gauss 8 2020/5)

Example 1.144
Triangle ABC has its vertices at A(2, 0), B(6, 0) and C(6, 3). The area of the triangle, in square
units, is (CEMC Gauss 7 2003/16)

Example 1.145
Points P(15, 55), Q(26, 55) and R(26, 35) are three vertices of rectangle PQRS. The
area of this rectangle is(CEMC Gauss 7 2020/13)

Example 1.146
Four vertices of a quadrilateral are located at (7,6),(-5,1),(-2,-3), and (10, 2). The area of the quadrilateral in
square units is (CEMC Gauss 7 2017/24)

B. More Area

Example 1.147
In the 10 by 10 grid of squares shown, point P can be at any of the 41 points of
intersection of pairs of gridlines inside (and not on) 4FGH. For each possible location
of P, exactly three triangles are formed: 4FPG, 4GPH, 4HPF. How many of these 123
triangles have an area that is exactly half of the area of 4FGH? (CEMC Gauss 7
2020/24)

P a g e 50 | 95
Get all the files at: https://bit.ly/azizhandouts
Aziz Manva ([email protected])

Example 1.148
In the 6 by 6 grid shown, two lines are drawn through point P, dividing the grid into
three regions of equal area. These lines will pass through the points (CEMC Gauss 8
2000/20)

C. Shoelace Theorem

Example 1.149
Rectangle 𝐷𝐸𝐹𝐴 below is a 3 × 4 rectangle with 𝐷𝐶 = 𝐶𝐵 = 𝐵𝐴 = 1. The area of the "bat
wings" (shaded area) is (AMC 8 2016/22)

D. Pick’s Theorem

1.150: Pick’s Theorem


If a polygon has lattice points for all of its vertices, then:
𝑏
𝐴𝑟𝑒𝑎 𝑜𝑓 𝑃𝑜𝑙𝑦𝑔𝑜𝑛 = 𝐴 = 𝑖 + − 1
2
where
𝑖 = 𝑁𝑢𝑚𝑏𝑒𝑟 𝑜𝑓 𝑙𝑎𝑡𝑡𝑖𝑐𝑒 𝑝𝑜𝑖𝑛𝑡𝑠 𝑡ℎ𝑎𝑡 𝑎𝑟𝑒 𝑖𝑛𝑡𝑒𝑟𝑖𝑜𝑟 𝑡𝑜 𝑡ℎ𝑒 𝑝𝑜𝑙𝑦𝑔𝑜𝑛
𝑏 = 𝑁𝑢𝑚𝑏𝑒𝑟 𝑜𝑓 𝑙𝑎𝑡𝑡𝑖𝑐𝑒 𝑝𝑜𝑖𝑛𝑡𝑠 𝑜𝑛 𝑡ℎ𝑒 𝑝𝑒𝑟𝑖𝑚𝑒𝑡𝑒𝑟 𝑜𝑓 𝑡ℎ𝑒 𝑝𝑜𝑙𝑦𝑔𝑜𝑛

Example 1.151
Pi using Pick’s Theorem

https://www.geogebra.org/m/y2nuDV37

E. Area of a triangle using determinants

P a g e 51 | 95
Get all the files at: https://bit.ly/azizhandouts
Aziz Manva ([email protected])

2. LINES
2.1 Basics
A. Horizontal and Vertical Lines
Vertical Line
A vertical line cutting the 𝑥-axis at 𝑎 has equation
𝑥= ⏟
𝑎 , 𝑎 = 𝑥 − 𝑖𝑛𝑡𝑒𝑟𝑐𝑒𝑝𝑡
𝑨𝒏𝒚 𝑪𝒐𝒏𝒔𝒕𝒂𝒏𝒕
This is true because, for a vertical line, any point on the line has the 𝑥 coordinate 𝑎.
The place where a line cuts the 𝑥-axis is called its 𝑥-intercept.

Horizontal Line
A horizontal line cutting the 𝑦-axis at 𝑎 has equation
𝑦= ⏟
𝑎 , 𝑎 = 𝑦 − 𝑖𝑛𝑡𝑒𝑟𝑐𝑒𝑝𝑡
𝑨𝒏𝒚 𝑪𝒐𝒏𝒔𝒕𝒂𝒏𝒕

This is true because, for a horizontal line, any point on the line has the 𝑦 coordinate 𝑎.
The place where a line cuts the 𝑦-axis is called its 𝑦-intercept.

Summary
Equation Nature of Line Example
𝑥= ⏟
𝑎 Vertical Line cutting the 𝑥-axis at 𝑎 𝑥=5
𝑨𝒏𝒚 𝑪𝒐𝒏𝒔𝒕𝒂𝒏𝒕
𝑦= ⏟
𝑎 Horizontal Line cutting the 𝑦-axis at 𝑎 𝑦=4
𝑨𝒏𝒚 𝑪𝒐𝒏𝒔𝒕𝒂𝒏𝒕

Example 2.1
Identify the following lines as horizontal or vertical. Also, state their 𝑥-intercept, or 𝑦-intercept, as the case may
be.
𝑥
Basics E. 4 = 2
A. 𝑥 = 4 1
F. =2
B. 𝑦 = 3 𝑦
1 1
Fractions G. 𝑥 = 3
4
C. 𝑥 = 3 Decimals
5 𝑥
D. 𝑦 = − 7 H. 0.2
= 0.7
0.3 0.7
Equations I. =
𝑦 0.03

Basics
𝑥 = 4 ⇒ 𝑉𝑒𝑟𝑡𝑖𝑐𝑎𝑙 𝐿𝑖𝑛𝑒, 𝑥 − 𝑖𝑛𝑡𝑒𝑟𝑐𝑒𝑝𝑡: 4
𝑦 = 4 ⇒ 𝐻𝑜𝑟𝑖𝑧𝑜𝑛𝑡𝑎𝑙 𝐿𝑖𝑛𝑒, 𝑦 − 𝑖𝑛𝑡𝑒𝑟𝑐𝑒𝑝𝑡: 3
Fractions
4 4
𝑥= ⇒ 𝑉𝑒𝑟𝑡𝑖𝑐𝑎𝑙 𝐿𝑖𝑛𝑒, 𝑥 − 𝑖𝑛𝑡𝑒𝑟𝑐𝑒𝑝𝑡:
3 3
5 5
𝑦 = − ⇒ 𝐻𝑜𝑟𝑖𝑧𝑜𝑛𝑡𝑎𝑙 𝐿𝑖𝑛𝑒, 𝑦 − 𝑖𝑛𝑡𝑒𝑟𝑐𝑒𝑝𝑡: −
7 7
Equations
𝑥
= 2 ⇒ 𝑥 = 8 ⇒ 𝑉𝑒𝑟𝑡𝑖𝑐𝑎𝑙 𝐿𝑖𝑛𝑒, 𝑥 − 𝑖𝑛𝑡𝑒𝑟𝑐𝑒𝑝𝑡: 8
4
P a g e 52 | 95
Get all the files at: https://bit.ly/azizhandouts
Aziz Manva ([email protected])

1 1 1
= 2 ⇒ 𝑦 = ⇒ 𝐻𝑜𝑟𝑖𝑧𝑜𝑛𝑡𝑎𝑙 𝐿𝑖𝑛𝑒, 𝑦 − 𝑖𝑛𝑡𝑒𝑟𝑐𝑒𝑝𝑡:
𝑦 2 2
Decimals
𝑥 7 2 14
= 0.7 ⇒ 𝑥 = 0.7 × 0.2 = × = = 0.14 ⇒ 𝑉𝑒𝑟𝑡𝑖𝑐𝑎𝑙 𝐿𝑖𝑛𝑒, 𝑥 − 𝑖𝑛𝑡𝑒𝑟𝑐𝑒𝑝𝑡: 0.14
0.2 10 10 100
0.3 0.7 𝑦 0.03 0.009 0.009 0.09
= ⇒ = ⇒𝑦= ⇒ 𝐻𝑜𝑟𝑖𝑧𝑜𝑛𝑡𝑎𝑙 𝐿𝑖𝑛𝑒, 𝑦 − 𝑖𝑛𝑡𝑒𝑟𝑐𝑒𝑝𝑡: =
𝑦 0.03 0.3 0.7 0.7 0.7 7

B. Slope
Slope refers to how much the 𝑦 coordinate changes when the 𝑥 coordinate changes.
➢ In a horizontal line, the 𝑦 coordinate does not change at all. Hence, the slope of a horizontal line is zero.
➢ In a horizontal line, the 𝑥 coordinate does not change at all. Hence, a vertical line is said to have no slope
✓ The technical reason for not having slope will become clear in the next section when we give the
formula for slope.

Example 2.2
State the slope of the lines below:
A. 𝑥 = 12
B. 𝑦 = 7
4
C. 𝑥 =
9
6
D. 𝑦 =
11

𝑥 = 12 ⇒ 𝑉𝑒𝑟𝑡𝑖𝑐𝑎𝑙 𝐿𝑖𝑛𝑒 = 𝑁𝑜 𝑆𝑙𝑜𝑝𝑒


𝑦 = 7 ⇒ 𝐻𝑜𝑟𝑖𝑧𝑜𝑛𝑡𝑎𝑙 𝐿𝑖𝑛𝑒 = 𝑍𝑒𝑟𝑜 𝑆𝑙𝑜𝑝𝑒 ⇒ 𝑆𝑙𝑜𝑝𝑒 = 0

Example 2.3: Review


For the following lines, state whether they are horizontal or vertical. State their x-intercept, or y-intercept, as
the case may be. Also, state their slope.

C. Slope from Points


Background
➢ Slope is a numerical measure of the steepness of a line.
➢ Any two points on a line will always have the same slope.
Slope between two points
There are a number of alternate definitions of slope. Each of them represents a different way of conceptualizing
slope, and all of them give the same answer.
However, questions can be solved faster if you know the right choice of definition to use.
Rise over Run
The slope between (𝑥1 , 𝑦1 ) and (𝑥2 , 𝑦2 ) is given by:
Rise
𝑚=

Run
𝑪𝒐𝒏𝒄𝒆𝒑𝒕
➢ 𝑚 is the variable which is generally used for slope
➢ Rise is the change in the 𝑦 coordinate.
➢ Run is the change in the 𝑥 coordinate.

Change in 𝒙 over Change in 𝒚

P a g e 53 | 95
Get all the files at: https://bit.ly/azizhandouts
Aziz Manva ([email protected])

Recall that
➢ Rise is the change in the 𝑦 coordinate
= Δ𝑦
➢ Run is the change in the 𝑥 coordinate
= Δ𝑥

The symbol Δ is the Capital Greek Letter Delta, which is often used whenever a change is to be measured.
Using this notation, we can write the formula for slope as:
Rise Δ𝑦
𝑚= =

Run ⏟
Δ𝑥
𝑪𝒐𝒏𝒄𝒆𝒑𝒕 𝑪𝒉𝒂𝒏𝒈𝒆 𝒊𝒏 𝒚
𝑪𝒉𝒂𝒏𝒈𝒆 𝒊𝒏 𝒙

Formula for slope


We can convert the above expression into a formula that is directly applicable to the coordinates of the points
that we get.
Δ𝑦 = 𝐶ℎ𝑎𝑛𝑔𝑒 𝑖𝑛 𝑦 𝑐𝑜𝑜𝑟𝑑𝑖𝑛𝑎𝑡𝑒 = 𝑦2 − 𝑦1
Δ𝑥 = 𝐶ℎ𝑎𝑛𝑔𝑒 𝑖𝑛 𝑥 𝑐𝑜𝑜𝑟𝑑𝑖𝑛𝑎𝑡𝑒 = 𝑥2 − 𝑥1

Using this, we can now write our formula for slope as:
Rise Δ𝑦 𝑦2 − 𝑦1
𝑚= = =

Run ⏟
Δ𝑥 𝑥2 − 𝑥1
𝑪𝒐𝒏𝒄𝒆𝒑𝒕 𝑪𝒉𝒂𝒏𝒈𝒆 𝒊𝒏 𝒚
𝑪𝒉𝒂𝒏𝒈𝒆 𝒊𝒏 𝒙
Alternate Formula for slope
The above formula can be used to calculate slope. But we sometimes want to put coordinates of the first point
first. Hence, we find an alternate version of the formula above:
𝑦2 − 𝑦1 𝑦2 − 𝑦1 −1 −𝑦2 + 𝑦1 𝑦1 − 𝑦2
= × = =
𝑥2 − 𝑥1 𝑥2 − 𝑥1 −1 −𝑥2 + 𝑥1 𝑥1 − 𝑥2
Summary
We can summarise the different form of the slope formula that we have done so far in the expression below.
Rise Δ𝑦 𝑦2 − 𝑦1 𝑦1 − 𝑦2
𝑚= = = =

Run ⏟
Δ𝑥 𝑥2 − 𝑥1 𝑥1 − 𝑥2

𝑪𝒐𝒏𝒄𝒆𝒑𝒕 𝑪𝒉𝒂𝒏𝒈𝒆 𝒊𝒏 𝒚 −𝟏
𝑪𝒉𝒂𝒏𝒈𝒆 𝒊𝒏 𝒙 𝑴𝒖𝒍𝒕𝒊𝒑𝒍𝒚 𝒃𝒚
−𝟏

Example 2.4: Slope from Points


Find the slope between

A. ( ⏟
2,⏟
4 ) and ( ⏟
1,⏟
3)
𝑥1 𝑦1 𝑥2 𝑦2
B. ( ⏟
2,⏟
5 ) and (−3
⏟ , −1
⏟)
𝒙𝟏 𝒚𝟏 𝒙𝟐 𝒚𝟐

Part A
𝑦1 − 𝑦2 4 − 3 1
𝑚= = = =1
𝑥1 − 𝑥2 2 − 1 1
Part B
𝑦1 − 𝑦2 5 − (−1) 6
Slope = 𝑚 = = =
𝑥1 − 𝑥2 2 − (−3) 5

P a g e 54 | 95
Get all the files at: https://bit.ly/azizhandouts
Aziz Manva ([email protected])

We can also calculate the slope using the other version of the formula:
𝑦2 − 𝑦1 −1 − 5 −6 6
𝑚= = = =
𝑥2 − 𝑥1 ⏟ −3 − 2 −5 5
𝑴𝒊𝒏𝒖𝒔 𝒔𝒊𝒈𝒏𝒔 𝒄𝒂𝒏𝒄𝒆𝒍
𝑺𝒍𝒐𝒑𝒆 𝒊𝒔 𝒔𝒂𝒎𝒆

Example 2.5: Slope from Graph


Identify the slope of the line graphed alongside.

Observe that the points (0,1) and (1,2) lie on the graph.
Substitute

Example 2.6: Slope from Table

D. Zero Slope and No Slope


We now consider some edge cases with respect to calculation of slope.
Vertical Lines do not have Slope
If the denominator is zero, recall that division by zero is not defined
➢ Hence, there is no slope.
➢ The value of the numerator is not important in this case. It can be anything (including zero).
Horizontal Lines have Zero Slope
If the numerator is zero, then there is no change in the 𝑦 coordinate and hence the slope is zero.
Summary Table
𝑁𝑢𝑚𝑒𝑟𝑎𝑡𝑜𝑟 = 𝑛 Denominator Slope
𝑛≠0 𝑛=0 Not Defined
𝑛=0 𝑛=0 Not Defined
𝑛=0 Any non-zero number 0

Example 2.7: Zero Slope and No Slope


The information gives six pairs of points. For each pair of points:
A. Find the slope between the points.
B. Hence, identify whether the lines connecting the points are horizontal or vertical.

I. (⏟
6,⏟
9 ) and ( ⏟
4,⏟
9)
𝑥1 𝑦1 𝑥2 𝑦2
II. (3,4) and (3,7)
III. (𝑎, 5) and (𝑎, 7)
IV. (𝑏, 8) and (𝑐, 8) where 𝑏 ≠ 𝑐
V. (𝑥, 𝑦) and (𝑥, 𝑧) where 𝑦 ≠ 𝑧
VI. (𝑝, 𝑞) and (𝑟, 𝑞) where 𝑝 ≠ 𝑟
𝑦1 − 𝑦2 9 − 9 0
𝑚= = = =0
𝑥1 − 𝑥2 6 − 4 2
𝑦1 − 𝑦2 4 − 7 −3
𝑚= = = ⇒ 𝑁𝑜𝑡 𝐷𝑒𝑓𝑖𝑛𝑒𝑑 ⇒ 𝑁𝑜 𝑆𝑙𝑜𝑝𝑒
𝑥1 − 𝑥2 3 − 3 0
𝑦1 − 𝑦2 5 − 7 2
𝑚= = = − ⇒ 𝑁𝑜𝑡 𝐷𝑒𝑓𝑖𝑛𝑒𝑑 ⇒ 𝑁𝑜 𝑆𝑙𝑜𝑝𝑒
𝑥1 − 𝑥2 𝑎 − 𝑎 0

P a g e 55 | 95
Get all the files at: https://bit.ly/azizhandouts
Aziz Manva ([email protected])

𝑦1 − 𝑦2 8 − 8 0
𝑚= = = =0
𝑥1 − 𝑥2 𝑏 − 𝑐 𝑏 − 𝑐
𝑦1 − 𝑦2 𝑦 − 𝑧 𝑦 − 𝑧
𝑚= = = ⇒ 𝑁𝑜𝑡 𝐷𝑒𝑓𝑖𝑛𝑒𝑑 ⇒ 𝑁𝑜 𝑆𝑙𝑜𝑝𝑒
𝑥1 − 𝑥2 𝑥 − 𝑥 0
𝑦1 − 𝑦2 𝑞 − 𝑞 0
𝑚= = = =0
𝑥1 − 𝑥2 𝑝 − 𝑟 𝑝 − 𝑟

E. Parallel Lines and Perpendicular Lines


We learn from Euclidean Geometry that any two points define a line.
Similarly, in coordinate geometry also, any two points will define a line

2.8: Parallel Lines

There are two cases


Case I: Both lines have slopes
If two lines are parallel, they have the same slope.
If two lines have the same slope, then they are parallel.
Case II: One or both lines do not have slopes
If both lines do not have slopes, both are vertical, and parallel.

Two lines are parallel if both have:


➢ the same slope
➢ No Slope

2.9: Perpendicular Lines

There are two cases


Case I: Both lines have slopes
If two lines are perpendicular, the product of their slopes is −1.
1
𝑚1 𝑚2 = −1 ⇒ 𝑚2 = −
𝑚1
From the above, we can conclude that if two lines are perpendicular, they will have slopes
1 1
𝑚 𝑎𝑛𝑑 − ⇒ (𝑚) (− ) = −1
𝑚 𝑚
If the product of the slopes of two lines is −1, then the two lines are perpendicular.
Case II: One or more of the lines do not have slopes
Additionally, a line which is no slope is perpendicular to a line which has zero slope.

Example 2.10: Identifying Lines


The information below is on nine lines, named Lines 𝐴 − 𝐼. Identify pairs of lines which are
A. Parallel
B. Perpendicular
1 1
𝑆𝑙𝑜𝑝𝑒𝑠: ⏟ 𝑁𝑜 𝑆𝑙𝑜𝑝𝑒 , ⏟0 , ⏟ 1 , ⏟ 2 , , − , ⏟ 1

2 ⏟ 2 𝑳𝒊𝒏𝒆 𝑮
𝑳𝒊𝒏𝒆 𝑨𝑳𝒊𝒏𝒆 𝑩 𝑳𝒊𝒏𝒆 𝑪 𝑳𝒊𝒏𝒆 𝑫
𝑳𝒊𝒏𝒆 𝑬 𝑳𝒊𝒏𝒆 𝑭
𝑁𝑎𝑡𝑢𝑟𝑒 𝑜𝑓 𝐿𝑖𝑛𝑒: 𝐻𝑜𝑟𝑖𝑧𝑜𝑛𝑡𝑎𝑙
⏟ ⏟
𝑉𝑒𝑟𝑡𝑖𝑐𝑎𝑙
𝑳𝒊𝒏𝒆 𝑯 𝑳𝒊𝒏𝒆 𝑰

Perpendicular Lines

P a g e 56 | 95
Get all the files at: https://bit.ly/azizhandouts
Aziz Manva ([email protected])

First Pair
⏟ 𝐴 (𝑛𝑜 𝑠𝑙𝑜𝑝𝑒) is perpendicular to the line 𝐵
𝐿𝑖𝑛𝑒 ⏟ (𝑧𝑒𝑟𝑜 𝑠𝑙𝑜𝑝𝑒)
𝐻𝑜𝑟𝑖𝑧𝑜𝑛𝑡𝑎𝑙 𝐿𝑖𝑛𝑒 𝑉𝑒𝑟𝑡𝑖𝑐𝑎𝑙 𝐿𝑖𝑛𝑒

Second Pair
1
2 × − = −1 ⇒ 𝐿𝑖𝑛𝑒𝑠 𝑎𝑟𝑒 𝑝𝑒𝑟𝑝𝑒𝑛𝑑𝑖𝑐𝑢𝑙𝑎𝑟
2
Third and Fourth Pair
Line H is horizontal. Line A is vertical. They are perpendicular
Line H is horizontal. Line I is vertical. They are perpendicular
Fifth and Sixth Pair
Line I is vertical. Line B is horizontal. They are perpendicular
Line A is vertical. Line B is horizontal. They are perpendicular
Parallel Lines
First Pair
Both Line C and Line G have slope 1. Line C parallel to Line G.
Second Pair
Line H is horizontal. Line B is also horizontal. They are parallel.
Third Pair
Line I is vertical. Line A is also vertical. They are parallel.

Example 2.11: Identifying Lines


Identify parallel and perpendicular lines from the below
A. Line A passes through points Type equation here. And Type equation here.

Example 2.12: Finding Slope


1 1
𝑆𝑙𝑜𝑝𝑒𝑠: ⏟
𝑁𝑜 𝑆𝑙𝑜𝑝𝑒 , ⏟0 , ⏟ 3 , ⏟ 7 , , − , ⏟1

4 ⏟9 𝐿𝑖𝑛𝑒 𝐺
𝐿𝑖𝑛𝑒 𝐴 𝐿𝑖𝑛𝑒 𝐵 𝐿𝑖𝑛𝑒 𝐶 𝐿𝑖𝑛𝑒 𝐷
𝐿𝑖𝑛𝑒 𝐸 𝐿𝑖𝑛𝑒 𝐹
Use the information given above on the slopes of various lines to find the slope of:
A. A line parallel to line D
B. A line perpendicular to Line C
C. A line parallel to Line A
D. A line perpendicular to line A
E. A line perpendicular to Line F
F. A line perpendicular to Line B

𝑹𝒊𝒔𝒆
F. Slope as 𝑹𝒖𝒏
Rise
Interpretation of slope is best done using the Run concept.
𝑅𝑖𝑠𝑒 = 𝐶ℎ𝑎𝑛𝑔𝑒 𝑖𝑛 𝑦 − 𝑣𝑎𝑟𝑖𝑎𝑏𝑙𝑒
𝑅𝑢𝑛 = 𝐶ℎ𝑎𝑛𝑔𝑒 𝑖𝑛 𝑥 − 𝑣𝑎𝑟𝑖𝑎𝑏𝑙𝑒
When Considered from Left to Right (that is, when the 𝒙 coordinate is increasing):
➢ Positive Slope means that the line is going up
➢ Negative slope means that the line is going down
When Considered from Right to Left (that is, when the 𝒙 coordinate is decreasing):
➢ Positive Slope means that the line is going down
➢ Negative slope means that the line is going up

P a g e 57 | 95
Get all the files at: https://bit.ly/azizhandouts
Aziz Manva ([email protected])

Some slopes are the same in whichever direction you consider the 𝒙 coordinate:
➢ Zero slope means that the line is going neither up nor down (Horizontal Line)
➢ No slope means that the line is vertical

Example 2.13: Interpreting Slope


6
The slope of a line is given to be 5. Interpret the slope.
6 Δ𝑦 𝐶ℎ𝑎𝑛𝑔𝑒 𝑖𝑛 𝑦 − 𝑣𝑎𝑟𝑖𝑎𝑏𝑙𝑒
= =
5 Δ𝑥 𝐶ℎ𝑎𝑛𝑔𝑒 𝑖𝑛 𝑥 − 𝑣𝑎𝑟𝑖𝑎𝑏𝑙𝑒
6
A slope of means that a change of:
5
➢ 5 units in the 𝑥-direction ⇔ 6 units in the 𝑦-direction
✓ Since the slope is positive, the line is going up when the x coordinate increases
➢ −5 units in the 𝑥-direction, there will be a change of −6 units in the 𝑦-direction
✓ Since the slope is positive, the line is going down when the 𝑥 coordinate decreases

G. Calculating Points on the Line

Example 2.14: Line Passing Through the Origin


3
The slope of a line is given to be 5. It is known that the origin(𝑂 = (0,0)) lies on the line.
A. Find five points to the right of the origin that lie on the line.
B. Find five points to the left of the origin that lie on the line
C. Hence, find a pattern for these points.
D. Also, write a real-world interpretation of the slope.

Parts A, B and C

𝑥 −25 −20 −15 −10 −5 0 5 10 15 20 25


𝑦 −15 −12 −9 −6 −3 0 3 6 9 12 15
Left of Origin Origin Right of Origin

Pattern
The x coordinate is a multiple of 5.
The y coordinate is a multiple of 3.
Real World Interpretation
Suppose that five chocolates cost $3. Then, the cost of one chocolate is given by:
𝐶𝑜𝑠𝑡 𝑜𝑓 𝐶ℎ𝑜𝑐𝑜𝑙𝑎𝑡𝑒𝑠 3 𝑑𝑜𝑙𝑙𝑎𝑟𝑠
=
𝑁𝑜. 𝑜𝑓 𝐶ℎ𝑜𝑐𝑜𝑙𝑎𝑡𝑒𝑠 5 𝑐ℎ𝑜𝑐𝑜𝑙𝑎𝑡𝑒𝑠
Hence, the cost of ten chocolates will be
3 𝑑𝑜𝑙𝑙𝑎𝑟𝑠 2 6 𝑑𝑜𝑙𝑙𝑎𝑟𝑠
× =
5 𝑐ℎ𝑜𝑐𝑜𝑙𝑎𝑡𝑒𝑠 2 10 𝑐ℎ𝑜𝑐𝑜𝑙𝑎𝑡𝑒𝑠

Example 2.15: General Line


The slope of a line is given to be 2. It is known that the point (0,8) lies on the line.
A. Find five points to the right of (0,8) that lie on the line.
B. Find five points to the left of (0,8) that lie on the line
C. Hence, find a pattern for these points.
D. Also, write a real-world interpretation of the slope.

P a g e 58 | 95
Get all the files at: https://bit.ly/azizhandouts
Aziz Manva ([email protected])

Parts A, B and C

2
𝑆𝑙𝑜𝑝𝑒 = 2 =
1

𝑥 -5 -4 -3 -2 -1 0 1 2 3 4 5
𝑦 -2 0 2 4 6 8 10 12 14 16 18
Left of (0,8) (0,8) Right of (0,8)

Pattern
The y coordinate is given by
𝑦 = 2𝑥 + 8

Example 2.16: Calculating change using given slope


4
The slope of a line is given to be 7.
E. Find the change in the 𝑦 coordinate when the 𝑥 coordinate increases by 7
F. Find the change in the 𝑦 coordinate when the 𝑥 coordinate decreases by 7
G. Find the change in the 𝑥 coordinate when the 𝑦 coordinate increases by 4
H. Find the change in the 𝑥 coordinate when the 𝑦 coordinate decreases by 4

Slope
4 Δ𝑦 𝐶ℎ𝑎𝑛𝑔𝑒 𝑖𝑛 𝑦 − 𝑣𝑎𝑟𝑖𝑎𝑏𝑙𝑒
𝑆𝑙𝑜𝑝𝑒 = = =
7 Δ𝑥 𝐶ℎ𝑎𝑛𝑔𝑒 𝑖𝑛 𝑥 − 𝑣𝑎𝑟𝑖𝑎𝑏𝑙𝑒
Part A
The 𝑥 coordinate has increased by seven. This means that
𝑥2 − 𝑥1 = Δ𝑥 = 7
But, from the formula for slope, we know that:
Δ𝑦 4
=
Δ𝑥 7
Substitute Δ𝑥 = 7 in the above:
Δ𝑦 4
= ⇒ Δ𝑦 = 4
7 7
Part B
Substitute Δ𝑥 = −7 in the formula for slope:
Δ𝑦 4 Δ𝑦 4
= ⇒ = ⇒ Δ𝑦 = −4
Δ𝑥 7 −7 7

Part C
Substitute Δ𝑦 = 4 in the formula for slope:
Δ𝑦 4 4 4 Δ𝑥 7
= ⇒ = ⇒ = ⇒ Δ𝑥 = 7
Δ𝑥 7 Δ𝑥 7 4 4

Part D
Substitute Δ𝑦 = −4 in the formula for slope:
Δ𝑦 4 −4 4 −Δ𝑥 7
= ⇒ = ⇒ = ⇒ Δ𝑥 = −7
Δ𝑥 7 Δ𝑥 7 4 4

Example 2.17: Real World Interpretation

P a g e 59 | 95
Get all the files at: https://bit.ly/azizhandouts
Aziz Manva ([email protected])

The cost of 𝑥 hours of internet time in a subscription plan is 3𝑥 dollars.


A. Tabulate the relation between internet time and the cost of that time
B. Frame this as a linear equation.
C. Plot the equation on a graph.
D. Calculate the slope of the graph that you plotted in part B.
E. Interpret the slope that you calculated in part C
I. Algebraically
II. Graphically
III. In the real world
S1:
A. We can tabulate below
Internet Time (Hours) Cost (Dollars)
1 3
2 6
5 15

B. 𝑦
⏟ = 3𝑥

𝑪𝒐𝒔𝒕 𝒐𝒇 𝒙 𝒉𝒐𝒖𝒓𝒔 𝒙=𝑵𝒐.𝒐𝒇 𝑯𝒐𝒖𝒓𝒔 𝒐𝒇 𝑰𝒏𝒕𝒆𝒓𝒏𝒆𝒕 𝒖𝒔𝒆𝒅
C. Since this is a linear equation, we know that its graph is a line. Two points are enough to define a line.
Substitute simple values of 𝑥 in the equation:
𝑥 = 0 ⇒ 𝑦 = 0, 𝑥 = 1 ⇒ 𝑦 = 3
This tells us that (0,0), (1, 3)
⏟ are two points that lie on the graph of the line.
𝑃𝑙𝑜𝑡 𝑡ℎ𝑒 𝑝𝑜𝑖𝑛𝑡𝑠
𝐽𝑜𝑖𝑛 𝑡ℎ𝑒𝑚 𝑢𝑠𝑖𝑛𝑔 𝑎 𝑟𝑢𝑙𝑒𝑟
3−0 3 0−3 −3 3
D. 𝑚 = = = 3, = = = 3 𝑎𝑙𝑠𝑜 𝑤𝑜𝑟𝑘𝑠
1−0 1 0−1 −1 1

E. Interpretation
Algebraic For every increase in 𝑥 by one, there will be a change of 3 units in 𝑦.
Graphical For every movement of one unit in the 𝑥-direction, there is a change of 3 units in
the 𝑦-direction.
Real World For every additional hour that you use the internet, you will need to pay the ISP
(Internet Service Provider), three dollars

H. Back Calculations
Given the slope of two points, it is possible to identify a missing value of one of the points.

Example 2.18: Back Calculations


Find 𝑥 if the slope of the line connecting (4,3) and (𝑥, 2) is 3.
𝑦2 − 𝑦1 3−2 11
𝑚= ⇒ 3= ⇒ 12 − 3𝑥 = 1 ⇒ 12 = 3𝑥 ⇒ 𝑥 =
𝑥2 − 𝑥1 ⏟ 4−𝑥 3
𝑺𝒖𝒃𝒔𝒕𝒊𝒕𝒖𝒕𝒆 𝑲𝒏𝒐𝒘𝒏 𝑽𝒂𝒍𝒖𝒆𝒔

I. Counting

Example 2.19
A lattice point is a point (x, y), with x and y both integers. For example, (2, 3) is a lattice point but (4, 1/3) is
not. In the diagram, how many lattice points lie on the perimeter of the triangle? (CEMC Gauss 8 2007/24)

P a g e 60 | 95
Get all the files at: https://bit.ly/azizhandouts
Aziz Manva ([email protected])

2.2 Slope Intercept Form


A. Types of Lines and Their Slope
A linear equation in 𝑥 and 𝑦 will give a line when you plot it on the coordinate plane.
Converse: The graph of a line on the coordinate plane is a linear equation.

Nature of Line 𝒚𝟐 −𝒚𝟏 Value of Slope Example


Nature of Slope ( )
𝒙𝟐 −𝒙𝟏
Parallel to 𝒙-axis No change in 𝑦. ∴ Slope = 0. 𝑦=3
(Horizontal Line) The numerator of the formula is Slope = 0
always zero.
Parallel to 𝒚-axis No change in 𝑥 ∴ No slope (not 𝑥=4
(Vertical Line) ∴The denominator is always zero defined). Slope is not defined
Division by zero is not defined.
Going upward (from Numerator will be positive for ∴ +𝑣𝑒 Slope 𝑦 = 2𝑥 + 3
left to right) positive change in 𝑦. Slope = 2 (+𝑣𝑒)
Going downward Numerator will be negative for ∴ -𝑣𝑒 Slope 𝑦 = −3𝑥 + 4
(from left to right) positive change in 𝑦. Slope = −3 (−𝑣𝑒)

2.20: Slope-Intercept Form


𝑦 = 𝑚𝑥 + 𝑐
𝑚 = 𝑠𝑙𝑜𝑝𝑒
𝑐 = 𝑦 − 𝑖𝑛𝑡𝑒𝑟𝑐𝑒𝑝𝑡

Derivation
Let the 𝑦-intercept of a line be
(0, 𝑐)
Substitute the 𝑦-intercept in the definition of slope
𝑦2 − 𝑦1 𝑦2 − 𝑐
𝑚= ⇒𝑚= ⇒ 𝑚𝑥2 = 𝑦2 − 𝑐 ⇒ 𝑦2 = 𝑚𝑥2 + 𝑐
𝑥2 − 𝑥1 𝑥2 − 0

But, we now have only one 𝑥 variable and one variable in the equation. So, we do not need the subscript.
𝐿𝑒𝑡 𝑥 = 𝑥2 , 𝑦 = 𝑦2 ⇒ 𝑦 = 𝑚𝑥 + 𝑐

Identifying Slope in the Slope-Intercept Form


For a line in the form
𝑦 = 𝑚𝑥 + 𝑐
The slope is given by
𝑆𝑙𝑜𝑝𝑒 = 𝑚 = 𝐶𝑜𝑒𝑓𝑓𝑖𝑐𝑖𝑒𝑛𝑡 𝑜𝑓 𝑥
And the y-intercept is given by
𝑐 = 𝑐𝑜𝑛𝑠𝑡𝑎𝑛𝑡 𝑡𝑒𝑟𝑚

B. Classifying Lines based on Slope

Example 2.21
Classify the following lines based on the table above:
{𝑥 = 4, 𝑦 = −7, 𝑥 = 0, 𝑦 = 3𝑥 + 4, 𝑦 = 0, 𝑦 = −2𝑥 + 4}

P a g e 61 | 95
Get all the files at: https://bit.ly/azizhandouts
Aziz Manva ([email protected])

𝑥
⏟= 4 , 𝑦 = −7 ,
⏟ 𝑥
⏟= 0 , 𝑦 = 3𝑥 + 4 ,
⏟ 𝑦
⏟= 0 , 𝑦 = −2𝑥 + 4

𝑽𝒆𝒓𝒕𝒊𝒄𝒂𝒍 𝑯𝒐𝒓𝒊𝒛𝒐𝒏𝒕𝒂𝒍 𝒚−𝒂𝒙𝒊𝒔 +𝒗𝒆 𝒔𝒍𝒐𝒑𝒆 𝒙−𝒂𝒙𝒊𝒔 −𝒗𝒆 𝒔𝒍𝒐𝒑𝒆
𝑵𝒐 𝑺𝒍𝒐𝒑𝒆 𝒁𝒆𝒓𝒐 𝑺𝒍𝒐𝒑𝒆 𝑵𝒐 𝑺𝒍𝒐𝒑𝒆 𝒎=𝟑 𝒁𝒆𝒓𝒐 𝒔𝒍𝒐𝒑𝒆 𝒎=−𝟐

C. Algebraic Form of a Line

Example 2.22
Verify that the table given below satisfies the equation
𝑦 = 2𝑥 + 3

𝑥 -1 0 1 2 3 4
𝑦 1 3 5 7 9 11

𝑥 -1 0 1 2 3 4
𝑦 −1 × 2 3 5 7 9 11
+3=1

Example 2.23
The cost to enter a theme park is three dollars. For every ride that you sit in the park, you need to pay a further
two dollars.
A. Find the cost of:
I. Entering the theme park, but not sitting in any ride.
II. Sitting in one ride
III. Sitting in two rides
IV. Sitting in three rides
V.
VI. Sitting in four rides
B. Plot the values obtained in part A on the coordinate plane using the x-axis for the number of rides, and
the y-axis for the cost of those rides.
C. Find an equation that connects the number of rides with the cost of those rides.
D. Does the value 𝑥 = −1 make sense in the context of the theme park?

A B C D E
Number 0 1 2 3 4
of Rides
𝐶𝑜𝑠𝑡 3 5 7 9 11

Let the number of rides be given by 𝑥.


Let the cost of 𝑥 rides be 𝑦.

The equation connecting the two is


𝑦 = 2𝑥 + 3

D. x-intercept and y-intercept


At the x-intercept, the y coordinate is zero.
At the y-intercept, the x coordinate is zero.

P a g e 62 | 95
Get all the files at: https://bit.ly/azizhandouts
Aziz Manva ([email protected])

Example 2.24
Sheetal has a hundred dollars in her bank account currently, which is Month Zero. She has been donating ten
dollars every month to a charity since the three months
A. Tabulate the amount of money in her bank account using a table.
B. Plot the point on the table using the coordinate plane. Plot the month number on the x-axis, and the
money on the y-axis.
C. Use the table to write an equation connecting x and y.

Month 𝑥 𝑥1 = 0 𝑥2 = 1 2 3 4
Money 𝑦 𝑦1 = 100 𝑦2 = 90 80 70 60

Month -1 -2 -3
Money 110 120 130

Slope
𝑦2 − 𝑦1 100 − 90 10
𝑆𝑙𝑜𝑝𝑒 = 𝑚 = = =− = −10
𝑥2 − 𝑥1 0−1 1
𝑦2 − 𝑦1 90 − 100 −10
𝑆𝑙𝑜𝑝𝑒 = 𝑚 = = = = −10
𝑥2 − 𝑥1 1−0 1

y-intercept
The y-intercept is the point on a line where it cuts the y-axis.
Recall that any point on the y-axis will have x coordinate zero.

Hence, the value of the y-intercept is the point where


𝑥=0

Equation
𝑦= 𝑚
⏟ 𝑥+ ⏟
𝑐 = −10𝑥 + 100
𝑆𝑙𝑜𝑝𝑒 𝑦−𝑖𝑛𝑡𝑒𝑟𝑐𝑒𝑝𝑡

Example 2.25
A. The slope of a line is 3, and the y-intercept of the line is 7. Find the equation of the line.
3 2
B. A line with y-intercept 4 has slope − 3. Find the equation of the line.

𝑦 = 𝑚𝑥 + 𝑐 ⇒ 𝑦 = 3𝑥 + 7
2 3
𝑦 = 𝑚𝑥 + 𝑐 ⇒ 𝑦 = − 𝑥 +
3 4

Example 2.26: Number Theory


The slope of a line has the same numerical value as the smallest prime number. The 𝑥-intercept of the line has
the same numerical value as the smallest odd prime number. The 𝑦-intercept of the line has a numerical value
double of the 𝑥-intercept. Find the equation of the line.

𝑦 = 𝑚𝑥 + 𝑐 = 2𝑥 + 6

Example 2.27
P a g e 63 | 95
Get all the files at: https://bit.ly/azizhandouts
Aziz Manva ([email protected])

A line has the points (0,4) and (5,0) on it. Find the slope-intercept form of the equation of the line.

Strategy
𝑦 = 𝑚𝑥 + 𝑐
In this, we need to find two things:
𝑆𝑙𝑜𝑝𝑒 = 𝑚
𝑦 − 𝑖𝑛𝑡𝑒𝑟𝑐𝑒𝑝𝑡 = 𝑐
y and x will not have values. They will remain as variables.

Finding Slope and y-intercept


𝑦2 − 𝑦1 0 − 4 4
𝑆𝑙𝑜𝑝𝑒 = = =−
𝑥2 − 𝑥1 5 − 0 5

The y-intercept is the point where the x coordinate is zero.


(0,4) ⇒ 𝑦 − 𝑖𝑛𝑡𝑒𝑟𝑐𝑒𝑝𝑡 = 𝑐 = 4

Find the equation


4
𝑦 = 𝑚𝑥 + 𝑐 ⇒ 𝑦 = − 𝑥 + 4
5

Example 2.28
Find the equation of the line connecting the points (2,1) and (5, −8)

First, find the slope


−8 − 1 −9
𝑚= = = −3
5−2 3
Now, find the y-intercept.
𝑚 = −3 ⇒ 𝐼𝑛𝑐𝑟𝑒𝑎𝑠𝑒 𝑜𝑓 𝑥 − 𝑣𝑎𝑙𝑢𝑒 𝑏𝑦 1 ⇒ 𝐷𝑒𝑐𝑟𝑒𝑎𝑠𝑒 𝑜𝑓 𝑦 − 𝑣𝑎𝑙𝑢𝑒 𝑏𝑦 3
⇒ 𝐷𝑒𝑐𝑟𝑒𝑎𝑠𝑒 𝑜𝑓 𝑥 − 𝑣𝑎𝑙𝑢𝑒 𝑏𝑦 1 ⇒ 𝐼𝑛𝑐𝑟𝑒𝑎𝑠𝑒 𝑜𝑓 𝑦 − 𝑣𝑎𝑙𝑢𝑒 𝑏𝑦 3

Hence,
𝑐 = 𝑦 − 𝑖𝑛𝑡𝑒𝑟𝑐𝑒𝑝𝑡 = 1 + 2(3) = 1 + 6 = 7

Hence
𝐸𝑞𝑢𝑎𝑡𝑖𝑜𝑛: 𝑦 = −3𝑥 + 7

Example 2.29
The point (5,7) lies on a line with 𝑦-intercept 4. Find the equation of the line.

𝑦 − 𝑖𝑛𝑡𝑒𝑟𝑐𝑒𝑝𝑡 = 𝑐 = 4 = (0,4)

𝑦2 − 𝑦1 7 − 4 3
𝑆𝑙𝑜𝑝𝑒 = 𝑚 = = =
𝑥2 − 𝑥1 5 − 0 5

Substitute the values that we found of 𝑚 and 𝑐 into the equation of a line:
3
𝑦 = 𝑚𝑥 + 𝑐 = 𝑥 + 4
5

P a g e 64 | 95
Get all the files at: https://bit.ly/azizhandouts
Aziz Manva ([email protected])

E. Checking if Points lie on a Line


Given a point, we can check if the point lies on the line by substituting it into the equation.
If both sides of the equation have the same value, then the equation is satisfied, and the point lies on the line.

Example 2.30
Does the point (3,13) lie on the line 𝑦 = 3𝑥 + 4.

𝐿𝐻𝑆 = 𝑦 = 13
𝑅𝐻𝑆 = 3𝑥 + 4 = 3(3) + 4 = 9 + 4 = 13 = 𝐿𝐻𝑆 ⇒ (3,13) 𝑙𝑖𝑒𝑠 𝑜𝑛 𝑡ℎ𝑒 𝑙𝑖𝑛𝑒

Example 2.31
The equation of a line is 𝑦 = 6𝑥 + 5.
A. Does the point (3,23) lie on the line?
B. Does the point (2,21) lie on the line?

𝑦 = 6𝑥 + 5 ⇒ 23 = 6(3) + 5 ⇒ 23 = 23 ⇒ 𝑉𝑎𝑙𝑖𝑑
Since the two sides of the equation match, the point (3,23) lies on the line.

𝑦 = 6𝑥 + 5 ⇒ 21 = 6(2) + 5 ⇒ 21 = 17 ⇒ 𝑁𝑜𝑡 𝑉𝑎𝑙𝑖𝑑


Since the two sides of the equation do not match, the point (2,21) does not lie on the line.
F. Finding coordinates

Example 2.32
The equation of a line is 𝑦 = 6𝑥 + 5.
A. Find the point on the line that has 𝑥 coordinate 5.
B. If a point on the line has 𝑦 coordinate 8, what is its 𝑥 coordinate?

Substitute 𝑥 = 5 in the equation of the line:


𝑦 = 6𝑥 + 5 = 6(5) + 5 = 30 + 5 = 35 ⇒ (𝑥, 𝑦) = (5,35)

Substitute 𝑦 = 5 in the equation of the line:


1 1
8 = 6𝑥 + 5 ⇒ 3 = 6𝑥 ⇒ 𝑥 = ⇒ (𝑥, 𝑦) = ( , 8)
2 2

G. Function Notation1
Instead of writing the 𝑦 coordinate, we can also write
𝑓(𝑥)
⏟ = 6𝑥 + 5
𝑭𝒖𝒏𝒄𝒕𝒊𝒐𝒏
Notice that the only change is that on the left-hand side, we have replaced 𝑦 with 𝑓(𝑥).

Earlier, we used to write the coordinates of points as:


(𝑥, 𝑦)

Now, since we replaced y with 𝑓(𝑥), we will do that for the coordinates of points as well, and write
(𝑥, 𝑓(𝑥))

1
It is preferable if you have done some functions before you do this part.

P a g e 65 | 95
Get all the files at: https://bit.ly/azizhandouts
Aziz Manva ([email protected])

This looks different, but works the same way as the earlier (𝑥, 𝑦) notation. It is a different notation that is used
frequently in higher grades.

Example 2.33
A function is given by
𝑓(𝑥) = 6𝑥 + 5
A. Identify the nature of the function
B. If 𝑥 = 5, find 𝑓(𝑥). In other words, find the point (5, 𝑓(5)).
C. If 𝑓(𝑥) = 8, what is 𝑥? In other words, find (𝑥, 8)?

The highest power of 𝑥 is 1. Hence, it is a linear function.

Substitute 𝑥 = 5 in the equation of the line:


𝑓(𝑥) = 6𝑥 + 5 = 6(5) + 5 = 30 + 5 = 35 ⇒ (𝑥, 𝑓(𝑥)) = (5,35)

Substitute 𝑦 = 5 in the equation of the line:


1 1
8 = 6𝑥 + 5 ⇒ 3 = 6𝑥 ⇒ 𝑥 = ⇒ (𝑥, 𝑦) = ( , 8)
2 2
H. 𝒙- and 𝒚-intercepts
At the 𝑥-intercept, the value of 𝑦 will be zero.
To get the 𝑥 − intercept, substitute 𝑦 = 0 in the equation of a line

At the y-intercept, the value of 𝑥 will be zero.


To get the 𝑦 − intercept. substitute 𝑥 = 0 in the equation of a line

Example 2.34: Finding 𝒙 and 𝒚 intercepts


Find the 𝑥 and the 𝑦 intercepts of the following lines:
A. 𝑦 = −2𝑥 + 4
3
B. 𝑦 = 5 𝑥 + 2
C. 3𝑦 + 2𝑥 = 4
Part A
Substitute 𝑥 = 0 to find the 𝑦-intercept:
𝑦 = −2𝑥 + 4 ⇒ 𝑦 = −2(0) + 4 = 4 ⇒ (𝟎, 𝟒)

𝒚−𝒊𝒏𝒕𝒆𝒓𝒄𝒆𝒑𝒕
Substitute 𝑦 = 0 to find the 𝑥-intercept:
0 = −2𝑥 + 4 ⇒ 2𝑥 = 4 ⇒ 𝑥 = 2 ⇒ (𝟐, 𝟎)

𝒙−𝒊𝒏𝒕𝒆𝒓𝒄𝒆𝒑𝒕

Part B
Substitute 𝑥 = 0 to find the 𝑦-intercept:
3
𝑦 = (0) + 2 = 0 + 2 = 2 ⇒ (𝟎,
⏟ 𝟐)
5
⏟ 𝒚−𝒊𝒏𝒕𝒆𝒓𝒄𝒆𝒑𝒕

Substitute 𝑦 = 0 to find the 𝑥-intercept:


3 3 5 10 𝟏𝟎
0 = 𝑥 + 2 ⇒ −2 = 𝑥 ⇒ −2 × = 𝑥 ⇒ 𝑥 = − ⇒ (− , 𝟎)
5 5 3 3 ⏟ 𝟑
𝒙−𝒊𝒏𝒕𝒆𝒓𝒄𝒆𝒑𝒕

Part C

P a g e 66 | 95
Get all the files at: https://bit.ly/azizhandouts
Aziz Manva ([email protected])

Substitute 𝑥 = 0 to find the 𝑦-intercept:


4 4
3𝑦 + 2𝑥 = 4 ⇒ 3𝑦 + 2(0) = 4 ⇒ 3𝑦 = 4 ⇒ 𝑦 = ⇒ 𝑦 − 𝑖𝑛𝑡𝑒𝑟𝑐𝑒𝑝𝑡 = (0, )
3 3
Substitute 𝑦 = 0 to find the 𝑥-intercept:
3𝑦 + 2𝑥 = 4 ⇒ 3(0) + 2𝑥 = 4 ⇒ 2𝑥 = 4 ⇒ 𝑥 = 2 ⇒ 𝑥 − 𝑖𝑛𝑡𝑒𝑟𝑐𝑒𝑝𝑡 = (2,0)

I. 𝒙- and 𝒚-intercepts from co-ordinates of points

Example 2.35: 𝒙 and 𝒚 intercepts from a table


Find the x-intercept and the y-intercept if the following points lie on a line.

𝑥 4 5 6 7
𝑦 5 7 9 11

Extrapolate the table back

𝑥 0 1 2 3 4 5 6 7
𝑦 -3 -1 1 3 5 7 9 11
y-intercept

𝑦 = 0 lies between the points in the table above where 𝑦 = 1, and 𝑦 = −1

𝑥 1 1.5 2
𝑦 -1 0 1
x-intercept

Example 2.36: 𝒙 and 𝒚 intercepts from a table

Subtracting 13 𝑥 y Adding 15
22 = 35 − 13 22 0 0 = −15 + 15
35 = 48 − 13 35 -15 −15 = −30 + 15
48 = 61 − 𝟏𝟑 48 -30 −30 = −45 + 𝟏𝟓
61 = 74 − 𝟏𝟑 61 -45 −45 = −60 + 𝟏𝟓
74 -60

J. Converting word problems to equations

Example 2.37: 𝒙 and 𝒚 intercepts from a table


A gymnasium charges a flat fee of $500 for membership for the year. Any sessions with a personal trainer are
charged at a cost of $30 per session.
A. Identify the dependent and the independent variable
B. Identify the 𝑦 − 𝑖𝑛𝑡𝑒𝑟𝑐𝑒𝑝𝑡
C. Identify the slope
D. Hence, write the equation of the line.

𝐷𝑒𝑝𝑒𝑛𝑑𝑒𝑛𝑡 𝑉𝑎𝑟𝑖𝑎𝑏𝑙𝑒 = 𝑦 = 𝐶𝑜𝑠𝑡 𝑜𝑓 𝐺𝑦𝑚


𝐼𝑛𝑑𝑒𝑝𝑒𝑛𝑑𝑒𝑛𝑡 𝑉𝑎𝑟𝑖𝑎𝑏𝑙𝑒 = 𝑥 = 𝑁𝑜. 𝑜𝑓 𝑆𝑒𝑠𝑠𝑖𝑜𝑛𝑠

P a g e 67 | 95
Get all the files at: https://bit.ly/azizhandouts
Aziz Manva ([email protected])

𝑦-intercept is the value when 𝑥, which is number of sessions is zero:


𝑦 − 𝑖𝑛𝑡𝑒𝑟𝑐𝑒𝑝𝑡 = 500

Slope is the increase in the cost due to additional session


𝑆𝑙𝑜𝑝𝑒 = 30

The equation of a line in slope-intercept form is


𝑦 = 𝑚𝑥 + 𝑐
We know that y-intercept = 500, and that slope =30. Substitute:
𝑦 = 30𝑥 + 500

K. Collinearity of Points
Three points 𝐴, 𝐵 and 𝐶 are collinear if
Slope 𝐴𝐵 = Slope 𝐵𝐶

Example 2.38
Check if the points 𝐴(2,3), 𝐵(3,5) and 𝐶(4,7) are collinear.

5−3 2 7−5 2
𝑚𝐴𝐵 = = = 2, 𝑚𝐵𝐶 = = = 2 ⇒ 𝑚𝐴𝐵 = 𝑚𝐵𝐶 ⇒ Points are collinear
3−2 1 4−3 1

Example 2.39
If points 𝐴(1,5), 𝐵(3, 𝑦) and 𝐶(7,10) are collinear, find 𝐵.

𝑦 − 5 10 − 𝑦 𝑦 − 5 10 − 𝑦 20
𝑚𝐴𝐵 = 𝑚𝐵𝐶 ⇒ = ⇒ = ⇒ 2𝑦 − 10 = 10 − 𝑦 ⇒ 3𝑦 = 20 ⇒ 𝑦 =
3−1 7−3 2 4 3

L. Parallel Lines
Parallel lines have the same slope but different intercepts.

Example 2.40: Checking for Parallel Lines


Determine which of the following lines, if any, parallel. If they are parallel, find the distance between them.
𝑦 = 3𝑥 + 2, 𝑦 = 7𝑥 + 5, 𝑦 = 3𝑥 + 7
𝑦 = 3𝑥 + 2 ,
⏟ 𝑦 = 7𝑥 + 5 ,
⏟ 𝑦 = 3𝑥 + 7 ⇒ 𝑇ℎ𝑒 𝑙𝑖𝑛𝑒𝑠 𝑤𝑖𝑡ℎ 𝑠𝑙𝑜𝑝𝑒 3 𝑎𝑟𝑒 𝑝𝑎𝑟𝑎𝑙𝑙𝑒𝑙

𝑆𝑙𝑜𝑝𝑒=3 𝑆𝑙𝑜𝑝𝑒=7 𝑆𝑙𝑜𝑝𝑒=3
𝐷𝑖𝑠𝑡𝑎𝑛𝑐𝑒 = 7 − 2 = 5

M. Converting from Other Forms

2.41:
Rewrite the equation
𝐴𝑥 + 𝐵𝑦 = 𝐶
In the slope-intercept form
𝑦 = 𝑚𝑥 + 𝑐

P a g e 68 | 95
Get all the files at: https://bit.ly/azizhandouts
Aziz Manva ([email protected])

𝐴 𝐶
𝐴𝑥 + 𝐵𝑦 = 𝐶 ⇒ 𝐵𝑦 = −𝐴𝑥 + 𝐶 ⇒ 𝑦 = − 𝑥+
⏟ 𝐵 ⏟
𝐵
𝑚=𝑆𝑙𝑜𝑝𝑒 𝑦−𝑖𝑛𝑡𝑒𝑟𝑐𝑒𝑝𝑡

2.3 Other Forms


A. Forms of Equation of a Line
The equation of a line can be represented in different forms.
Each form has its own applications, and is useful in different contexts. Many exam questions come down to
receiving information in one form, and converting it to another form.
Command over the different forms, their uses, and their interpretation is very important.

Form Equation Features When to use


Special Cases
Horizontal Line 𝑦=𝑐 Given a line with zero slope
Vertical Line 𝑥=𝑐 Given a line with no slope

General Cases
Slope-Intercept 𝑦 = 𝑚𝑥 + 𝑐 𝑚 = slope Given the slope, and the 𝑦-intercept
𝑐 = 𝑦 − intercept

Slope-Point 𝑦 − 𝑦1 = 𝑚(𝑥 − 𝑥1 ) 𝑚 = slope Given the slope, and a point.


(𝑥1 , 𝑦1 ) is a point Can also be used with two points, by
calculating slope first

Other Forms
General 𝐴𝑥 + 𝐵𝑦 = 𝐶 Useful in Linear Algebra
Two-point 𝑦2 − 𝑦1 𝑦2 − 𝑦1 Use when given two points
𝑦 − 𝑦1 = ( ) (𝑥 − 𝑥1 ) = slope
𝑥2 − 𝑥1 𝑥2 − 𝑥1

2.42: Slope-Point Form


The slope-point form of the equation of a line is given by:
𝑦 − 𝑦1 = 𝑚(𝑥 − 𝑥1 )

➢ We can derive this equation from the formula for slope:


𝑦2 − 𝑦1
𝑚= ⇒ ⏟ 𝑦2 − 𝑦1 = 𝑚(𝑥2 − 𝑥1 ) ⇒ ⏟ 𝑦 − 𝑦1 = 𝑚(𝑥 − 𝑥1 )
⏟ 𝑥2 − 𝑥1 𝑴𝒖𝒍𝒕𝒊𝒑𝒍𝒚 𝒃𝒐𝒕𝒉 𝒔𝒊𝒅𝒆𝒔 𝒃𝒚 𝒙𝟐 −𝒙𝟏 𝑳𝒆𝒕 𝒚=𝒚𝟐 ,𝒙=𝒙𝟐
𝑫𝒆𝒇𝒊𝒏𝒊𝒕𝒊𝒐𝒏 𝒐𝒇 𝑺𝒍𝒐𝒑𝒆
➢ This equation is useful for determining the equation of a line from two points.

Example 2.43: Lines with no and zero slope


Find the equation of the line with
A. slope zero, and 𝑦-intercept 3.
B. no slope, and 𝑥-intercept 7. Can you find it by substituting it in the slope-point form of the equation of a
line?
A line with zero slope must be horizontal. Since, it has 𝑦-intercept 3, the equation of the line must be
𝑦=3
A line with no slope must be vertical. Since, it has 𝑥-intercept 7, the equation of the line must be
𝑥=7
If a line does not have slope, you cannot find the equation of the line by substituting it in the slope-point form.

P a g e 69 | 95
Get all the files at: https://bit.ly/azizhandouts
Aziz Manva ([email protected])

Example 2.44: Determining the Equation of a line


Find the equation of the following lines:
A. Slope 5 and 𝑦-intercept 3
B. Slope 1.2 and (7,12) lies on the line
C. The points (2,3) and (7,12) lie on the line.
D. The points (-8,8) and (1,-10) lie on the line.

Part A
Substitute 𝑚 = 5, and 𝑐 = 3 in the slope intercept form to get:
𝑦 = 5𝑥 + 3
Part B
Substitute 𝑚 = 1.2, 𝑥1 = 7, 𝑦1 = 12 in the slope point form 𝑦 − 𝑦1 = 𝑚(𝑥 − 𝑥1 ) to get:
𝑦 − 12 = 1.2(𝑥 − 7) ⇒ 𝑦 − 12 = 1.2𝑥 − 8.4 ⇒ ⏟ 𝑦 = 1.2𝑥 + 3.6
𝑺𝒍𝒐𝒑𝒆−𝑰𝒏𝒕𝒆𝒓𝒄𝒆𝒑𝒕 𝑭𝒐𝒓𝒎
𝑶𝒇𝒕𝒆𝒏 𝒎𝒐𝒔𝒕 𝒖𝒔𝒆𝒇𝒖𝒍

Part C
Find the slope:
12 − 3 9
𝑚= =
7−2 5
9
Substitute 𝑚 = , 𝑥1 = 2, 𝑦1 = 3 in the slope point form 𝑦 − 𝑦1 = 𝑚(𝑥 − 𝑥1 ) to get:
5
9 9 18 9 17
𝑦 − 3 = (𝑥 − 2) ⇒ 𝑦 − 3 = 𝑥 − ⇒ 𝑦= 𝑥+
5 5 5 ⏟ 5 5
𝑺𝒍𝒐𝒑𝒆−𝑰𝒏𝒕𝒆𝒓𝒄𝒆𝒑𝒕 𝑭𝒐𝒓𝒎

Part D
−10 − 8 18
𝑚= =− = −2
1 − (−8) 9
Substitute 𝑚 = −2, 𝑥1 = −8, 𝑦1 = 8 in the slope-point form 𝑦 − 𝑦1 = 𝑚(𝑥 − 𝑥1 ):
𝑦 − 8 = −2(𝑥 − (−8))
𝑦 − 8 = −2𝑥 − 16
𝑦 = −2𝑥 − 8

Example 2.45: Converting to slope-intercept form

Example 2.46: Calculating the slope from points

B. Perpendicular Lines
Case I: Horizontal and Vertical Lines
A line with zero slope(horizontal) is perpendicular to a line with no slope (vertical).
Case II: Everything Else
Two lines are perpendicular if their slopes are negative reciprocals of each other, or equivalently if the product
of their slopes is negative unity.
1 1
𝑙⏟1 ⊥
⏟ 𝑙⏟2 ⇔ 𝑚1 𝑚2 = −1 ⇔ 𝑚1 = − ⇔ 𝑚2 = −
𝑃𝑒𝑟𝑝𝑒𝑛𝑑𝑖𝑐𝑢𝑙𝑎𝑟
𝑚2 𝑚1
𝐿𝑖𝑛𝑒 1 𝐿𝑖𝑛𝑒 2

P a g e 70 | 95
Get all the files at: https://bit.ly/azizhandouts
Aziz Manva ([email protected])

Example 2.47: Slopes of Perpendicular Lines


2 7
Find if the lines 𝑦 = 3 𝑥 + 6, and 𝑦 = 4 𝑥 + 4 are perpendicular.
2 7
𝑚1 = , 𝑚2 =
3 4
You don’t need to do the multiplication here because both slopes are positive, and the product of two positive
numbers will never be negative.
2 7 7
Product of slopes = 𝑚1 𝑚2 = 3 × 4 = 6

In order for two lines to be perpendicular, their slopes must be opposite in sign.

Example 2.48: Slopes of Perpendicular Lines


Q1: Line 𝑙1 has positive slope. Line 𝑙2 has negative slope. Line 𝑙3 has zero slope. Line 𝑙4 has no slope. Line 𝑙5 has
positive slope.
A. Which pair of lines must be perpendicular?
B. Which pair of lines can be perpendicular? (You can have more than one pair)
C. Which pair of lines cannot be perpendicular?
S2:
A. 𝑙3 ⊥ 𝑙4
B. 𝑙1 ⊥ 𝑙2 is possible. 𝑙2 ⊥ 𝑙5 is possible.
C. 𝑙5 cannot be perpendicular to 𝑙1 , 𝑙3 cannot be perpendicular to any line except 𝑙4 ,

C. Graphing Lines

Example 2.49
Graph 𝑦 = 3𝑥 − 1

We have been given the slope-intercept form, so make use of it:


𝑆𝑙𝑜𝑝𝑒 = 𝑚 = 3
𝑦 𝑖𝑛𝑡𝑒𝑟𝑐𝑒𝑝𝑡 = −1

We can substitute 𝑥 = 1 to get:


𝑦 = 3(1) − 1 = 3 − 1 = 2 ⇒ (1,2) 𝑙𝑖𝑒𝑠 𝑜𝑛 𝑡ℎ𝑒 𝑙𝑖𝑛𝑒

Example 2.50
Graph the line that goes through the points
(1,2) 𝑎𝑛𝑑 (2,3)

Plot the points on the coordinate plane and join them.

P a g e 71 | 95
Get all the files at: https://bit.ly/azizhandouts
Aziz Manva ([email protected])

Example 2.51
Graph the line that goes through the point (1,2) and has slope 3

D. Back Calculations

Example 2.52
Find the value of 𝑎 + 𝑏 if the line given in the graph satisfies
3𝑥 + 𝑎𝑦 = 𝑏

Method I
Identify the intercepts:
(0,6) 𝑎𝑛𝑑 (2,0)

Method II
Two points on the line are:
(0,6) 𝑎𝑛𝑑 (2,0)
𝑆𝑙𝑜𝑝𝑒 = −3
𝑦 − 𝑖𝑛𝑡𝑒𝑟𝑐𝑒𝑝𝑡 = 6

Substitute 𝑚 = −3, 𝑐 = 6
𝑦 = 𝑚𝑥 + 𝑐 ⇒ 𝑦 = −3𝑥 + 6 ⇒ 3𝑥 + 𝑦 = 6

Example 2.53
Two points (1,3) and (4,5) lie on the line 𝑎𝑥 + 4𝑦 = 𝑐. Find the values of 𝑎 and 𝑐.

Substitute (1,3) in the equation of the line:


𝑎 + 12 = 𝑐

Substitute (4,5) in the equation of the line:


4𝑎 + 20 = 𝑐

8
𝑎 + 12 = 4𝑎 + 20 ⇒ 3𝑎 = −8 ⇒ 𝑎 = −
3

8 −8 + 36 28
𝑐 = 𝑎 + 12 = − + 12 = =
3 3 3

Example 2.54
A line with 𝑥-intercept 4 and 𝑦-intercept 7 has equation 𝑏𝑦 + 2𝑥 + 𝑐 = 0. Find 𝑏𝑐.

8 + 𝑐 = 0 ⇒ 𝑐 = −8

8
7𝑏 + 𝑐 = 0 ⇒ 7𝑏 + (−8) = 0 ⇒ 7𝑏 = 8 ⇒ 𝑏 =
7

P a g e 72 | 95
Get all the files at: https://bit.ly/azizhandouts
Aziz Manva ([email protected])

8 64
𝑏𝑐 = (−8) ( ) = −
7 7

Example 2.55
A horizontal line passes though the point (3,4). Find the equation of the line.

𝑦=4

Example 2.56
2 4
A vertical line passes though the point ( , ). Find the equation of the line.
3 7

2
𝑥=
3

2.4 Multiple Lines


A. Quadrilateral Flowchart

Example 2.57: Quadrilaterals


Q2: Determine the nature of the quadrilateral in each of the below. Choose your answers from one or more of:
Parallelogram, Rhombus, Rectangle, Square, Kite, General.
ABCD
S3: U
Q3: Show that the diagonals of a rectangle bisect each other.
S4: U

B. Area

Example 2.58
Line 𝑙 has 𝑥 −intercept and y-intercept 3 units away from the origin. There are four different possibilities that
satisfy line 𝑙. These four possibilities together enclose a square. Find the area of the square.

Length of diagonal
= 𝑑 = 3 − (−3) = 6
Area of Square
1 1
= × 𝑑2 = × 62 = 18
2 2

Example 2.59
Find the area of the triangle enclosed by the 𝑥-axis, the 𝑦-axis, and the line with 𝑥-intercept 5, and 𝑦-intercept 4.

(5,0) 𝑎𝑛𝑑 (0,4)

1 1
𝐴𝑟𝑒𝑎 = ℎ𝑏 = × 5 × 4 = 10
2 2

P a g e 73 | 95
Get all the files at: https://bit.ly/azizhandouts
Aziz Manva ([email protected])

Example 2.60
Find the area of the triangle enclosed by the 𝑥-axis, the 𝑦-axis, and the line with equation 𝑦 = 3𝑥 + 7.

7
(0,7) 𝑎𝑛𝑑 (− , 0)
3

1 1 7 49
ℎ𝑏 = × 7 × =
2 2 3 6
C. Polygons

Example 2.61
Let the slope between points 𝐴 and 𝐵 be written 𝑚(𝐴𝐵). Vertex 𝑉𝑛 , 0 ≤ 𝑛 ≤ 4 of polygon 𝑃 with five vertices is
given by
𝑛(𝑛 + 1)
(𝑥, 𝑦) = (𝑛, )
2
A. What is the slope of the line segment of 𝑃 with maximum slope?
𝑚(𝑉0 ,𝑉1 )+𝑚(𝑉1 ,𝑉2 )+𝑚(𝑉2 ,𝑉3 )+ 𝑚(𝑉3 ,𝑉4)
B. Find 4
𝑡ℎ 𝑛(𝑛+1)
C. If the coordinates of the 𝑛 vertex are given by (𝑥, 𝑦) = (𝑛, (−1)𝑛 2
), then repeat Parts A and B.

Part A
𝑦1 − 𝑦0 1 − 0
𝑚(𝑉0 , 𝑉1 ) = = =1
𝑥1 − 𝑥0 1 − 0
𝑦2 − 𝑦1 3 − 1
𝑚(𝑉1 , 𝑉2 ) = = =2
𝑥2 − 𝑥1 2 − 1
𝑦3 − 𝑦2 6 − 3
𝑚(𝑉2 , 𝑉3 ) = = =2
𝑥3 − 𝑥2 2 − 1
𝑦4 − 𝑦3 10 − 6
𝑚(𝑉3 , 𝑉4 ) = = = 4 ⇒ 𝑀𝑎𝑥
𝑥4 − 𝑥3 4−3
Part B

𝑚(𝑉0 , 𝑉1 ) + 𝑚(𝑉1 , 𝑉2 ) + 𝑚(𝑉2 , 𝑉3 ) + 𝑚(𝑉3 , 𝑉4 ) 1 + 2 + 3 + 4 10


= = = 2.5
4 4 4

You can also calculate:


𝑦1 − 𝑦0 𝑦2 − 𝑦1 𝑦3 − 𝑦2 𝑦4 − 𝑦3 𝑦4 − 𝑦1
𝑥1 − 𝑥0 + 𝑥2 − 𝑥1 + 𝑥3 − 𝑥2 + 𝑥4 − 𝑥3 1 𝑦4 − 𝑦3 10 − 0 10
= = = 𝑚(𝑉0 , 𝑉4 ) = = = 2.5
4 4 𝑥4 − 𝑥1 4−0 4
Part C

P a g e 74 | 95
Get all the files at: https://bit.ly/azizhandouts
Aziz Manva ([email protected])

Example 2.62
𝑇ℎ𝑖𝑠 𝑞𝑢𝑒𝑠𝑡𝑖𝑜𝑛 𝑒𝑥𝑡𝑒𝑛𝑑𝑠 𝑡ℎ𝑒 𝑝𝑟𝑒𝑣𝑖𝑜𝑢𝑠 𝑒𝑥𝑎𝑚𝑝𝑙𝑒.
Let the slope between points 𝐴 and 𝐵 be written 𝑚(𝐴𝐵). Vertex 𝑉𝑛 , 0 ≤ 𝑛 ≤ 2020 of polygon 𝑃 with 2021
vertices is given by:
𝑛(𝑛 + 1)
(𝑥, 𝑦) = (−2021 + 𝑛, −2021 + (−1)𝑛 )
2
A. What is the slope of the line segment of 𝑃 with maximum slope?
B. Find 𝑚(𝑉0 , 𝑉1 ) + 𝑚(𝑉1 , 𝑉2 ) + ⋯ + 𝑚(𝑉2019 , 𝑉2020 )

P a g e 75 | 95
Get all the files at: https://bit.ly/azizhandouts
Aziz Manva ([email protected])

3. CONICS
3.1 Conic Sections and Circles
A. Conics from a Cone

3.1: Conic Sections


➢ A slice parallel to the base creates a circle.
➢ If you “tilt” the slice, you get an ellipse.
➢ If you “tilt” the slice further so that one end becomes
“open, you get a parabola.
➢ So far (circles, ellipses, and parabolas), only one part
of the cone is involved. If you cut both parts, then you
get a hyperbola.

Note: Do not treat these as definitions. We will take up more


technical (and precise) definitions based on 2D geometry.

B. Transformations of Conics

C. Circle

3.2: Locus of a Circle


A circle is defined as the locus of points that are the same distance from another point. The common distance is
called the radius. The other point is called the center of the circle.

3.3: Equation of a Circle with center at the origin


𝑥2 + 𝑦2 = 𝑟2
𝐶𝑒𝑛𝑡𝑒𝑟 𝑜𝑓 𝑐𝑖𝑟𝑐𝑙𝑒 𝑎𝑡 𝑜𝑟𝑖𝑔𝑖𝑛

Let the center of the circle have coordinates (ℎ, 𝑘). Let the radius of the circle be 𝑟.

Substitute coordinates of origin (0,0) in the distance formula 𝐷 = √(𝑥1 − 𝑥2 )2 + (𝑦1 − 𝑦2 )2 :


𝐷 = √(𝑥1 − 𝑥2 )2 + (𝑦1 − 𝑦2 )2 = √(𝑥1 )2 + (𝑦1 )2

Since we want all points that are a fix distance from the origin, we set 𝐷 = 𝑟:
𝑟 = √(𝑥1 )2 + (𝑦1 )2 ⇒ 𝑟 2 = 𝑥 2 + 𝑦 2

Example 3.4: Determining Center and Radius


Determine the center and radius of the following circles:
A. 𝑥 2 + 𝑦 2 = 1
B. 𝑥 2 + 𝑦 2 = 4
C. 𝑥 2 + 𝑦 2 = 9
D. 𝑥 2 + 𝑦 2 = 10

Center for all circles above is the origin.

P a g e 76 | 95
Get all the files at: https://bit.ly/azizhandouts
Aziz Manva ([email protected])

𝑥2 + 𝑦2 = 1 ⇒ 𝑟 = 1
𝑥2 + 𝑦2 = 4 ⇒ 𝑟 = 2
𝑥2 + 𝑦2 = 9 ⇒ 𝑟 = 3
𝑥 2 + 𝑦 2 = 10 ⇒ 𝑟 = √10

Example 3.5: Determining the Equation


A circle with center at the origin has radius √3. Find the equation of a circle and graph it.

𝑥2 + 𝑦2 = 3

3.6: Equation of a Circle


(𝒙 − 𝒉)𝟐 + (𝒚 − 𝒌)𝟐 = 𝒓𝟐
𝑪𝒆𝒏𝒕𝒓𝒆 = (𝒉, 𝒌), 𝑹𝒂𝒅𝒊𝒖𝒔 = 𝒓
Let the coordinates of a point lying on the circumference of the circle be (𝑥, 𝑦).

By the distance formula, the distance between (ℎ, 𝑘) and (𝑥, 𝑦) is given by
√(𝑥 − ℎ)2 + (𝑦 − 𝑘 2 ) = 𝑟
(𝑥 − ℎ)2 + (𝑦 − 𝑘 2 ) = 𝑟

Example 3.7: Graphing Circles


Graph the circles:
A. (𝑦 − 3)2 + (𝑥 − 2)2 = 16
B. (𝑦 + 2)2 + (𝑥 + 1)2 = 10

Part A
Compare the equation with (𝑥 − ℎ)2 + (𝑦 − 𝑘)2 = 𝑟 2
ℎ = 2, 𝑘 = 3 ⇒ centre is (2,3)
2
𝑟 = 16 ⇒ 𝑟 = √16 = 4

Hence, we want to graph a circle with centre at (2,3) and radius of 4.


Part B

Note that the equation we have has plus signs, while the standard form has minus signs. So, we need to create
the minus signs in the equation that we have:
2 2
(𝑦 − (−2)) + (𝑥 − (−1)) = 10

Now, compare the equation with (𝑥 − ℎ)2 + (𝑦 − 𝑘)2 = 𝑟 2


ℎ = −1, 𝑘 = −2 ⇒ centre is (−1, −2)
2
𝑟 = 10 ⇒ 𝑟 = √10
Hence, we want to graph a circle with
➢ Centre at (−1, −2)
➢ Radius of √10

P a g e 77 | 95
Get all the files at: https://bit.ly/azizhandouts
Aziz Manva ([email protected])

Example 3.8: Finding the Equation of a Circle-I


In each part below, find the equation of the circle, and graph it.
A. A circle with center (1,4) and radius 3.
B. A circle with center (−2, −5) and radius 4.
C. The circle on the right.
D. A circle in the third quadrant with radius 3 tangent to both the axes.
E. A circle in the second quadrant with radius 4 is tangent to the 𝑥-axis. The least
distance between the circle and the 𝑦-axis is 3. Graph the circle, and find its
equation.
F. The endpoints of the diameter of a circle are (1,2) 𝑎𝑛𝑑 (5,7). Find the equation of the circle.
Part A
Substitute ℎ = 1, 𝑘 = 4, 𝑟 = 3 in (𝑥 − ℎ)2 + (𝑦 − 𝑘)2 = 𝑟 2 to get:
(𝑥 − 1)2 + (𝑦 − 4)2 = 9
Part B
Substitute ℎ = −2, 𝑘 = −5, 𝑟 = 4 in (𝑥 − ℎ)2 + (𝑦 − 𝑘)2 = 𝑟 2 to get:
(𝑥 + 2)2 + (𝑦 + 5)2 = 16
Part C
Draw a horizontal diameter, and a vertical diameter. Then:
𝐶𝑒𝑛𝑡𝑟𝑒 𝑜𝑓 𝐶𝑖𝑟𝑐𝑙𝑒 = 𝐼𝑛𝑡𝑒𝑟𝑠𝑒𝑐𝑡𝑖𝑜𝑛 𝑜𝑓 𝐷𝑖𝑎𝑚𝑒𝑡𝑒𝑟𝑠 = (1,2)
𝑅𝑎𝑑𝑖𝑢𝑠 = 4
(𝑥 − 1)2 + (𝑦 − 2)2 = 16
Part D
The circle is at a distance of three units from the 𝑥-axis, and it lies in the third quadrant.
Hence, the 𝑥 coordinate of the center must be
−3

Similarly, the circle is at a distance of three units from the y-axis, and it lies in the third
quadrant. Hence, the 𝑥 coordinate of the center must be
−3
By
(𝑥 + 3)2 + (𝑦 + 3)2 = 9
Part E

(𝑥 + 7)2 + (𝑦 − 4)2 = 16
Part F
The center of the circle is also the midpoint of the diameter. Hence, by
the midpoint formula:
1+5 2+7
𝐶=( , ) = (3,4.5)
2 2
𝑟 2 = (5 − 1)2 + (7 − 2)2 = 16 + 25 = 41
(𝑥 − 3)2 + (𝑥 − 4.5) = 41

Example 3.9: Completing the square to get standard form


Attempt to write the equation below as a circle in standard form. If it is a circle determine its center and radius.
If it is not a circle, explain, why not.
A. 𝑥 2 + 6𝑥 + 𝑦 2 + 2𝑦 = 1

P a g e 78 | 95
Get all the files at: https://bit.ly/azizhandouts
Aziz Manva ([email protected])

Part A
(𝑥 + 3)2 + (𝑦 + 1)2 = 11
𝐶𝑒𝑛𝑡𝑒𝑟: (−3, −1), 𝑅𝑎𝑑𝑖𝑢𝑠: √11

Example 3.10: Distance between circles


Circles 𝐴 and 𝐵 are given below:
𝐶𝑖𝑟𝑐𝑙𝑒 𝐴: 𝑥 2 − 6𝑥 + 4 + 𝑦 2 − 8𝑥 + 5 = 0
𝐶𝑖𝑟𝑐𝑙𝑒 𝐵: 𝑥 2 + 6𝑥 + 4 + 𝑦 2 + 8𝑥 + 5 = 0
A. Find the center, radius, area and circumference of circles 𝐴 and 𝐵.
B. Find the distance between the centers of 𝐴 and 𝐵.
C. Find the shortest distance between two points, one on Circle A and the other on Circle B.
D. Similarly, find the greatest distance.

Part A
𝑥 2 − 6𝑥 + 4 + 𝟓 + ⏟
𝐶𝑖𝑟𝑐𝑙𝑒 𝐴: ⏟ 𝑦 2 − 8𝑥 + 5 + 𝟏𝟏 = +𝟓 + 𝟏𝟏
(𝒙−𝟑)𝟐 (𝒚−𝟒)𝟐
2
(𝑥 − 3) + (𝑦 − 4) = 16 2
⇒⏟
𝑪=(𝟑,𝟒),𝒓=𝟒,𝑨=𝟏𝟔𝝅,𝑪=𝟖𝝅

𝑥 2 + 6𝑥 + 4 + 𝟓 + ⏟
𝐶𝑖𝑟𝑐𝑙𝑒 𝐵: ⏟ 𝑦 2 + 8𝑥 + 5 + 𝟏𝟏 = +𝟓 + 𝟏𝟏
(𝒙+𝟑)𝟐 (𝒚+𝟒)𝟐
2
(𝑥 + 3) + (𝑦 + 4) = 16 2
⇒⏟
𝑪=(−𝟑,−𝟒),𝒓=𝟒,𝑨=𝟏𝟔𝝅,𝑪=𝟖𝝅

Part B
Distance between the centres = 𝐴𝐵 = √(3 + 3)2 + (4 + 4)2
= √36 + 64 = √100 = 10
Part C
The shortest distance will be of two points on the line connecting the centers of the two circles such that they
are on the boundaries of the circle closest to each other.
Shortest Distance between two points on the Circles = 𝐴𝐵 − 𝑟1 − 𝑟2 = 10 − 4 − 4 = 2
Part D
Greatest Distance between two points on the Circles = 𝐴𝐵 + 𝑟1 + 𝑟2 = 10 + 4 + 4 = 18

Example 3.11
Find the area of circle A below the line Type equation here. and to the right of the line Type equation here.
(Horizontal Line)

Example 3.12
Find the area of circle B to the right of line Type equation here. (Passing through the centre = Diameter) (Tilted
Line)

P a g e 79 | 95
Get all the files at: https://bit.ly/azizhandouts
Aziz Manva ([email protected])

Example 3.13
A machine-shop cutting tool has the shape of a notched circle, as shown. The radius
of the circle is √50 cm, the length of AB is 6 cm and that of BC is 2 cm. The angle
ABC is a right angle. Find the square of the distance (in centimeters) from B to the
center of the circle. (AIME 1983/4)

Shortcut
We want to find 𝑂𝐵2 = 𝑂𝐸 2 + 𝐸𝐵2 . By observation:
𝑥 = 𝑂𝐸 = 1, 𝑦 = 𝐸𝐵 = 5
Using Coordinate Geometry
Let the center of the circle be the origin. Then, the equation of the circle is:
𝑥 2 + 𝑦 2 = 50
Let point B have coordinates (𝑥, 𝑦). Then
𝐴 = (𝑥, 𝑦 + 6), 𝐵 = (𝑥 + 2, 𝑦)
Since both A and B lie on the circle:
𝑃𝑜𝑖𝑛𝑡 𝐴: 50 = 𝑥 2 + (𝑦 + 6)2 ⇒ 50 ⏟ = 𝑥 2 + 𝑦 2 + 12𝑦 + 36
𝑬𝒒𝒖𝒂𝒕𝒊𝒐𝒏 𝑰
𝑃𝑜𝑖𝑛𝑡 𝐵: 50 = (𝑥 + 2)2 + 𝑦 2 ⇒ ⏟
50 = 𝑥 2 + 4𝑥 + 4 + 𝑦 2
𝑬𝒒𝒖𝒂𝒕𝒊𝒐𝒏 𝑰𝑰

Since the LHS of both Eq. I and II is equal, equate the RHS of both equations:
𝑥 2 + 𝑦 2 + 12𝑦 + 36 = 𝑥 2 + 4𝑥 + 4 + 𝑦 2 ⇒ 12𝑦 + 36 = 4𝑥 + 4 ⇒ 3𝑦
⏟ +8 = 𝑥
𝑬𝒒𝒖𝒂𝒕𝒊𝒐𝒏 𝑰𝑰𝑰
Substitute the value of 𝑥 from Equation III in Equation I:
50 = (3𝑦 + 8)2 + 𝑦 2 + 12𝑦 + 36
50 = 9𝑦 2 + 48𝑦 + 64𝑦 2 + 12𝑦 + 36
0 = 10𝑦 2 + 60𝑦 + 50
0 = 𝑦 2 + 6𝑦 + 5
0 = (𝑦 + 6)(𝑦 + 1)
𝑦 ∈ {−1, −5}
By looking at the diagram, we take 𝑦 = −1 ⇒ 𝑥 = 5. Hence, the final answer is:
𝑥 2 + 𝑦 2 = 12 + 52 = 1 + 25 = 26

Example 3.14: Finding the Equation of a Circle-II


More complicated cases

D. Sphere
A sphere is the three-dimensional version of a circle. The equation of a sphere is very similar to the equation of
a circle.

3.15: Equation of a Sphere


(𝒙 − 𝒉)𝟐 + (𝒚 − 𝒌)𝟐 + (𝒛 − 𝒋)𝟐 = 𝒓𝟐
𝑪𝒆𝒏𝒕𝒓𝒆 = (𝒉, 𝒌, 𝒋), 𝑹𝒂𝒅𝒊𝒖𝒔 = 𝒓

Example 3.16
What is the largest possible distance between two points, one on the sphere of radius 19 with center
(−2, −10,5) and the other on the sphere of radius 87 with center (12,8, −16)? (AIME 1987/2)

P a g e 80 | 95
Get all the files at: https://bit.ly/azizhandouts
Aziz Manva ([email protected])

The distance between the two centers:


= 𝐷 = √[12 − (−2)]2 + [8 − (−10)]2 + (−16 − 5)2 = √142 + 182 + 212 = 31

Largest possible distance between two points on the spheres:


= 𝑟1 + 𝑟2 + 𝐷 = 19 + 87 + 31 = 137

3.2 Ellipse
A. Ellipse with center at Origin

3.17: Definition
➢ An ellipse is the set of all points in a plane, the sum of whose
distances from two foci is constant.
➢ The center of an ellipse is the midpoint of the line segment joining
the foci.

Example 3.18
𝐶ℎ𝑜𝑜𝑠𝑒 𝑎𝑙𝑙 𝑐𝑜𝑟𝑟𝑒𝑐𝑡 𝑜𝑝𝑡𝑖𝑜𝑛𝑠
An ellipse is drawn alongside. It looks like:
A. An egg
B. An oval
C. A squashed circle

𝑂𝑝𝑡𝑖𝑜𝑛𝑠 𝐴, 𝐵, 𝐶

Example 3.19: Vocabulary Test


An ellipse is drawn alongside.
A. Identify the foci and the center
B. Point 𝑃 is on the ellipse. What is 𝑄𝐹1 + 𝑄𝐹2 ?
C. Point 𝑄 is on the ellipse. What is 𝑄𝐹1 + 𝑄𝐹2 ?

𝐹𝑜𝑐𝑖 𝑎𝑟𝑒 𝐹1 𝑎𝑛𝑑 𝐹2


𝐶𝑒𝑛𝑡𝑒𝑟 𝑖𝑠 𝐶
𝑃𝐹1 + 𝑃𝐹2 = 𝑑1 + 𝑑2
𝑄𝐹1 + 𝑄𝐹2 = 𝑑1 + 𝑑2

3.20: Terminology
➢ Center
➢ Major Axis
➢ Vertices

➢ Minor Axis

3.21: Equation of an ellipse (Warm-up)


An ellipse has foci (0,4) and (0, −4). The sum of the distances from the foci to the boundary of the ellipse is 10.
Prove that the equation of the ellipse is:
𝑥2 𝑦2
+ =1
9 25
P a g e 81 | 95
Get all the files at: https://bit.ly/azizhandouts
Aziz Manva ([email protected])

Use the distance formula in the condition for an ellipse 𝑃𝐹1 + 𝑃𝐹2 = 10, 𝑃 = (𝑥, 𝑦), 𝐹1 =
(0,4), 𝐹2 = (0, −4):
√𝑥 2 + (𝑦 − 4)2 + √𝑥 2 + (𝑦 + 4)2 = 10
Isolate a radical on the LHS, square both sides, simplify, and isolate the remaining radical
on the RHS:
√𝑥 2 + (𝑦 − 4)2 = 10 − √𝑥 2 + (𝑦 + 4)2
𝑥 2 + (𝑦 − 4)2 = 100 − 20√𝑥 2 + (𝑦 + 4)2 + [𝑥 2 + (𝑦 + 4)2 ]
𝑥 2 + 𝑦 2 − 8𝑦 + 16 = 100 − 20√𝑥 2 + (𝑦 + 4)2 + [𝑥 2 + 𝑦 2 + 8𝑦 + 16]
−8𝑦 = 100 − 20√𝑥 2 + (𝑦 + 4)2 + 8𝑦
−16𝑦 − 100 = −20√𝑥 2 + (𝑦 + 4)2
16𝑦 + 100 = 20√𝑥 2 + (𝑦 + 4)2
4𝑦 + 25 = 5√𝑥 2 + (𝑦 + 4)2
We had two radicals. Now, we have only one. So, we have made progress. Again, square both sides to eliminate
the remaining radical, and simplify:
16𝑦 2 + 200𝑦 + 625 = 25(𝑥 2 + 𝑦 2 + 8𝑥 + 16)
16𝑦 2 + 200𝑦 + 625 = 25𝑥 2 + 25𝑦 2 + 200𝑦 + 400
225 = 25𝑥 2 + 9𝑦 2
Divide both sides by 225:
25𝑥 2 9𝑦 2 𝑥2 𝑦2
+ =1⇒ + =1
225 225 9 25

3.22: Standard Equation of an ellipse


An ellipse has foci (0, 𝑐) and (0, −𝑐). Prove that the equation of the ellipse is:
𝑥2 𝑦2
+ = 1, 𝑏 2 = 𝑎2 − 𝑐 2
𝑎2 𝑏 2

3.23: Eccentricity
𝑐
𝐸𝑐𝑐𝑒𝑛𝑡𝑟𝑖𝑐𝑖𝑡𝑦 = 𝑒 =
𝑎
0≤𝑒<1

A circle is a special case of an ellipse:


𝑒 = 0 ⇒ 𝑆ℎ𝑎𝑝𝑒 𝑖𝑠 𝑎 𝑐𝑖𝑟𝑐𝑙𝑒

If
𝑒 = 1 ⇒ 𝑆ℎ𝑎𝑝𝑒 𝑖𝑠 𝑎 𝑝𝑎𝑟𝑎𝑏𝑜𝑙𝑎

B. Ellipse with center at (𝒉, 𝒌)

Example 3.24: Find the center and radius


Completing the Square

P a g e 82 | 95
Get all the files at: https://bit.ly/azizhandouts
Aziz Manva ([email protected])

C. Directrix Definition

3.25: Directrix

3.3 Parabola
3.4 Hyperbola

P a g e 83 | 95
Get all the files at: https://bit.ly/azizhandouts
Aziz Manva ([email protected])

4. 3D GEOMETRY
4.1 Lines and Planes
A. Locus of Points

B. Lines

C. Planes

Example 4.1
Which point on the plane 2𝑥 + 3𝑦 + 6𝑧 = 98 is closest to the origin?

The point on the plane closest to any point lies on the line
perpendicular to that point. The vector of coefficients (2,3,6) is
perpendicular to the plane.
∴ Both the origin, and (2,3,6) lie on the line perpendicular to the plane.

The vector equation of a line is


𝑟(𝑡) = 𝑃 + 𝑡𝒗, where P = (0,0,0), 𝐯 = (2,3,6)
𝑟(𝑡) = (0,0,0) + 𝑡(2,3,6)
𝑟(𝑡) = 𝑡(2,3,6)
𝑥𝒊 + 𝑦𝒋 + 𝑧𝒌 = 2𝑡𝒊 + 3𝑡𝒋 + 6𝑡𝒌
Equating coefficients gives us:
𝑥 = 2𝑡, 𝑦 = 3𝑡, 𝑧 = 6𝑡
We find the intersection of the line and the plane. Substituting the
above in the equation of the plane 2𝑥 + 3𝑦 + 6𝑧 = 98:
2(2𝑡) + 3(3𝑡) + 6(6𝑡) = 98 ⇒ 49𝑡 = 98 ⇒ 𝑡 = 2
Hence,
𝑥 = 2𝑡 = 4, 𝑦 = 3𝑡 = 6, 𝑧 = 6𝑡 = 12 ⇒ (𝑥, 𝑦, 𝑧) = (4,9,12)
And this is the point on the plane closest to the origin.

4.2: Parallel Planes


Planes are parallel when:
A. Normal vector of one plane is a scalar multiple of the other
B. Cross product of the vectors is zero

4.3: Perpendicular Planes


Planes are perpendicular when their normal vectors are perpendicular:
A. Two vectors are perpendicular if their dot product is zero

Example 4.4
𝑃𝑙𝑎𝑛𝑒 𝑃: −4𝑥 + 5𝑦 − 2𝑧 = 37
A. Write equation for plane 𝑃1 parallel to plane 𝑃, and passing through (-2,8,5)
B. Write equation for plane 𝑃2 parallel to plane 𝑃, and passing through (-2,8,5)

Part A

P a g e 84 | 95
Get all the files at: https://bit.ly/azizhandouts
Aziz Manva ([email protected])

(𝑁𝑜𝑟𝑚𝑎𝑙 𝑉𝑒𝑐𝑡𝑜𝑟)𝑃 = (𝑁𝑜𝑟𝑚𝑎𝑙 𝑉𝑒𝑐𝑡𝑜𝑟)𝑃1 = [−4,5, −2]

Dot Product of Normal Vector and Plane is Zero:


[−4,5, −2][𝑥 + 2, 𝑦 − 8, 𝑧 − 5] = 0
(−4𝑥 − 8) + (5𝑦 − 40) + (−2𝑧 + 10) = 0
−4𝑥 + 5𝑦 − 2𝑧 = 38
Part B
Substitute 𝑎=1, 𝑏=1 in [−4,5, −2][𝑎, 𝑏, 𝑐] = 0:
1
[−4,5, −2][1,1, 𝑐] = 0 ⇒ −4 + 5 − 2𝑐 = 0 ⇒ 𝑐 =
2

1
[1,1, ] [𝑥 + 2, 𝑦 − 8, 𝑧 − 5] = 0
2
(𝑥 + 2) + (𝑦 − 8) + (0.5𝑧 − 2.5) = 0
2𝑥 + 2𝑦 + 𝑧 = 13

Example 4.5
Find the shortest distance from (−1.2, −12.5,6.1) to the plane 2𝑥 + 5𝑦 − 𝑧 = −8.

The point on the plane closest to any point lies on the line
perpendicular to that point. The vector of coefficients (2,5,-1) is
perpendicular to the plane.
∴ Both (−1.2, −12.5,6.1), and (2,5,-1) lie on the line perpendicular to
the plane.

The vector equation of a line is


𝑟(𝑡) = 𝑃 + 𝑡𝒗, where P = (−1.2, −12.5,6.1), 𝐯 = (2,5, −1)
𝑟(𝑡) = (−1.2, −12.5,6.1) + 𝑡(2,5, −1)
𝑟(𝑡) = (−1.2 + 2t, −12.5 + 5t, 6.1 − t))
Equating coefficients gives us:
𝑥 = −1.2 + 2𝑡, 𝑦 = −12.5 + 5𝑡, 𝑧 = 6.1 − 𝑡
We find the intersection of the line and the plane. Substituting the
above in the equation of the plane 2𝑥 + 3𝑦 + 6𝑧 = 98:
2(−1.2 + 2𝑡) + 5(−12.5 + 5𝑡) − 1(6.1 − 𝑡) = −8 ⇒ 𝑡 = 2.1
Hence,
𝑥 = −1.2 + 2𝑡 = 3
𝑦 = −12.5 + 5𝑡 = −2
𝑧 = 6.1 − 𝑡 = 4
(𝑥, 𝑦, 𝑧) = (3, −2,4)

Use the distance formula to find the distance between (−1.2, −12.5,6.1) and (3, −2,4)
𝑑 = √(−1.2 − 3)2 + (−12.5 + 2)2 + (6.1 − 4)2 = 11.502

P a g e 85 | 95
Get all the files at: https://bit.ly/azizhandouts
Aziz Manva ([email protected])

4.2 AMC Questions


A. Lines

Example 4.6
The lines with equations 𝑎𝑥 − 2𝑦 = 𝑐 and 2𝑥 + 𝑏𝑦 = −𝑐 are perpendicular and intersect at (1, −5). What is 𝑐?
(AMC 10B 2017/10)

Example 4.7
The line 12𝑥 + 5𝑦 = 60 forms a triangle with the coordinate axes. What is the sum of the lengths of the
altitudes of this triangle? (AMC 10B 2015/13)

Example 4.8
A line with slope 3 intersects a line with slope 5 at point (10,15). What is the distance between the x-intercepts
of these two lines? (AMC 10B 2003/11)

Example 4.9
Positive integers 𝑎 and 𝑏 are such that the graphs of 𝑦 = 𝑎𝑥 + 5 and 𝑦 = 3𝑥 + 𝑏 intersect the 𝑥-axis at the same
point. What is the sum of all possible 𝑥-coordinates of these points of intersection? (AMC 10A 2014/21)

Example 4.10
√3
A line that passes through the origin intersects both the line 𝑥 = 1 and the line 𝑦 = 1 + 𝑥. The three lines
3
create an equilateral triangle. What is the perimeter of the triangle? (AMC 10A 2014/17)

Example 4.11
A lattice point in an 𝑥𝑦-coordinate system is any point (𝑥, 𝑦) where both 𝑥 and 𝑦 are integers. The graph of 𝑦 =
1
𝑚𝑥 + 2 passes through no lattice point with 0 < 𝑥 ≤ 100 for all 𝑚 such that < 𝑚 < 𝑎. What is the maximum
2
possible value of 𝑎? (AMC 10B 2011/24)

As per the condition given:


𝑦 = 𝑚𝑥 + ⏟
2 ∉ℤ
𝐼𝑛𝑡𝑒𝑔𝑒𝑟
Since 2 is an integer, we must have:
𝑚𝑥 ∉ ℤ
𝑛𝑢𝑚𝑒𝑟𝑎𝑡𝑜𝑟 𝑛
Let us write m in the form = :
𝑑𝑒𝑛𝑜𝑚𝑖𝑛𝑎𝑡𝑜𝑟 𝑑
𝑛
𝑥∉ℤ
𝑑
Note that since 0 < 𝑥 ≤ 100, 𝑥 will take all integral values from 1 to 100:
1,2, … ,100
𝑛
If 𝑥 is a multiple of 𝑑, then 𝑑 𝑥 will be an integer. Hence:
𝑑 ∈ {1,2, … ,100}
Since 𝑚 = 1 is an integer, 𝑚 < 1, and hence

P a g e 86 | 95
Get all the files at: https://bit.ly/azizhandouts
Aziz Manva ([email protected])

1
<𝑚<1
2
𝑛
We want the smallest value of 𝑑 that meets the above conditions.
𝑛
To minimize 𝑑, we
Maximize the numerator

Example 4.12
The diagram shows 28 lattice points, each one unit from its nearest neighbors. Segment AB meets segment CD
at E. Find the length of segment AE. (AMC 10 2000/16)

B. Basics

Example 4.13
Figure 𝑂𝑃𝑄𝑅 is a square. Point 𝑂 is the origin, and point 𝑄 has
coordinates (2,2). What are the coordinates for 𝑇 so that the area of
triangle 𝑃𝑄𝑇 equals the area of square 𝑂𝑃𝑄𝑅?(AMC 8 1996/17)

C. Area
Example 4.14
What is the area of the region enclosed by the graph of the equation 𝑥 2 + 𝑦 2 = |𝑥| + |𝑦|? (AMC 10B 2016/21)

Example 4.15
The shaded region below is called a shark's fin falcata, a figure studied by Leonardo
da Vinci. It is bounded by the portion of the circle of radius 3 and center (0,0) that
3
lies in the first quadrant, the portion of the circle with radius and center (0,3/2)
2
that lies in the first quadrant, and the line segment from (0,0) to (3,0). What is the
area of the shark's fin falcata? (AMC 10B 2015/9)

Example 4.16
Two circles of radius 2 are centered at (2,0) and at (0,2). What is the area of the intersection of the interiors of
the two circles? (AMC 10B 2007/13)

Example 4.17
In rectangle 𝐴𝐵𝐶𝐷, we have 𝐴 = (6, −22), 𝐵 = (2006,178), 𝐷 = (8, 𝑦), for some integer 𝑦. What is the area of

P a g e 87 | 95
Get all the files at: https://bit.ly/azizhandouts
Aziz Manva ([email protected])

rectangle 𝐴𝐵𝐶𝐷? (AMC 10B 2006/20)

Example 4.18
A square in the coordinate plane has vertices whose 𝑦-coordinates are 0, 1, 4, and 5. What is the area of the
square? (AMC 10A 2014/18)

Example 4.19
The horizontal and vertical distances between adjacent points equal 1 unit. The area of
triangle 𝐴𝐵𝐶 is (AMC 8 1996/22)

Example 4.20
Dots are spaced one unit apart, horizontally and vertically. The number of square units
enclosed by the polygon is (AMC 8 1998/6)

Example 4.21
Consider these two geoboard quadrilaterals. Which of the following statements is
true? (AMC 8 2000/18)
A. The area of quadrilateral I is more than the area of quadrilateral II.
B. The area of quadrilateral I is less than the area of quadrilateral II.
C. The quadrilaterals have the same area and the same perimeter.
D. The quadrilaterals have the same area, but the perimeter of I is more than
the perimeter of II.
E. The quadrilaterals have the same area, but the perimeter of I is less than
the perimeter of II.

Example 4.22
Points 𝐴, 𝐵, 𝐶 and 𝐷 have these coordinates: 𝐴(3,2), 𝐵(3, −2), 𝐶(−3, −2) and 𝐷(−3,0). The area of quadrilateral
𝐴𝐵𝐶𝐷 is (AMC 8 2001/11)

Example 4.23
Which of the following polygons has the largest area? (AMC 8 2002/15)

P a g e 88 | 95
Get all the files at: https://bit.ly/azizhandouts
Aziz Manva ([email protected])

Example 4.24
What is the area enclosed by the geoboard quadrilateral below? (AMC 8 2004/14)

Example 4.25
Semicircles 𝑃𝑂𝑄 and 𝑅𝑂𝑆 pass through the center 𝑂. What is the ratio of the combined
areas of the two semicircles to the area of circle 𝑂? (AMC 8 2010/23)

Example 4.26
A triangle with vertices as 𝐴 = (1,3), 𝐵 = (5,1), and 𝐶 = (4,4) is plotted on a 6 × 5 grid.
What fraction of the grid is covered by the triangle? (AMC 8 2015/19)

Example 4.27
The twelve-sided figure shown has been drawn on 1 cm × 1 cm graph paper.
What is the area of the figure in cm2 ? (AMC 8 2018/4)

8

9 + = 9 + 4 = 13

2
𝐹𝑢𝑙𝑙 𝑆𝑞𝑢𝑎𝑟𝑒𝑠
𝐻𝑎𝑙𝑓 𝑆𝑞𝑢𝑎𝑟𝑒𝑠

D. Area with Lines

Example 4.28

P a g e 89 | 95
Get all the files at: https://bit.ly/azizhandouts
Aziz Manva ([email protected])
1
Two lines with slopes 2 and 2 intersect at (2,2). What is the area of the triangle enclosed by these two lines and
the line 𝑥 + 𝑦 = 10 ? (AMC 10A 2019/8)

Example 4.29
The triangular plot of ACD lies between Aspen Road, Brown Road and a railroad.
Main Street runs east and west, and the railroad runs north and south. The numbers
in the diagram indicate distances in miles. The width of the railroad track can be
ignored. How many square miles are in the plot of land ACD? (AMC 9 2009/7)

Example 4.30
What is the area of the triangle formed by the lines 𝑦 = 5, 𝑦 = 1 + 𝑥, and 𝑦 = 1 − 𝑥? (AMC 8 2019/21)

1 1
𝐴= × ℎ × 𝑏 = × 4 × 8 = 16
2 2

Example 4.31
A triangle with vertices (6, 5), (8, -3), and (9, 1) is reflected about the line 𝑥 = 8 to create a second triangle.
What is the area of the union of the two triangles? (AMC 10A 2013/16)

Example 4.32
A rectangular region is bounded by the graphs of the equations 𝑦 = 𝑎, 𝑦 = −𝑏, 𝑥 = −𝑐, and 𝑥 = 𝑑, where 𝑎, 𝑏, 𝑐,
and 𝑑 are all positive numbers. Which of the following represents the area of this region? (AMC 10A 2011/9)

Example 4.33
The 𝑦-intercepts, 𝑃 and 𝑄, of two perpendicular lines intersecting at the point 𝐴(6,8) have a sum of zero. What
is the area of △ 𝐴𝑃𝑄? (AMC 10A 2014/14)

Example 4.34
Let points 𝐴 = (0,0), 𝐵 = (1,2), 𝐶 = (3,3), and 𝐷 = (4,0). Quadrilateral 𝐴𝐵𝐶𝐷 is cut into equal area pieces by a
𝑝 𝑟
line passing through 𝐴. This line intersects 𝐶𝐷 at point (𝑞 , 𝑠 ), where these fractions are in lowest terms. What is
𝑝 + 𝑞 + 𝑟 + 𝑠? (AMC 10A 2013/18)

Example 4.35
Let 𝑅 be a unit square region and 𝑛 ≥ 4 an integer. A point 𝑋 in the interior of 𝑅 is called 𝑛-𝑟𝑎𝑦 partitional if

P a g e 90 | 95
Get all the files at: https://bit.ly/azizhandouts
Aziz Manva ([email protected])

there are 𝑛 rays emanating from 𝑋 that divide 𝑅 into 𝑛 triangles of equal area. How many points are 100-ray
partitional but not 60-ray partitional? (AMC 10A 2011/25)

Draw a unit square with its bottom left vertex located at the origin.2
Choose a point in the interior of 𝑅 with coordinates
(𝑝, 𝑞)

If a ray is not drawn to the vertex, then the vertex will be part of a
quadrilateral-shaped region, and not a triangle. Hence, draw rays to
the vertices, getting four triangular regions:
𝑅1 , 𝑅2 , 𝑅3 , 𝑅4

1
Using the formula for area of a triangle 𝐴 = ℎ𝑏, the area of these
2
regions is:
𝑞 1−𝑝 1−𝑞 𝑝
[𝑅1 ] = , [𝑅2 ] = , [𝑅3 ] = , [𝑅4 ] =
2 2 2 2

The number of triangles in each region must be an integer. Let:


𝑅1 , 𝑅2 , 𝑅3 , 𝑅4 ℎ𝑎𝑣𝑒 𝑎, 𝑏, 𝑐, 𝑑 𝑡𝑟𝑖𝑎𝑛𝑔𝑙𝑒𝑠 𝑟𝑒𝑠𝑝𝑒𝑐𝑡𝑖𝑣𝑒𝑙𝑦

Hence, the area of a triangle in regions:


𝑞 1−𝑝 1−𝑞 𝑝
𝑅1 , 𝑅2 , 𝑅3 , 𝑅4 𝑚𝑢𝑠𝑡 𝑏𝑒, 𝑟𝑒𝑠𝑝𝑒𝑐𝑡𝑖𝑣𝑒𝑙𝑦, , , ,
2𝑎 2𝑏 2𝑐 2𝑑

However, no matter which region the triangle is in, the area must be equal. Hence,
𝑞 1−𝑞 𝑎
= ⇒ 2𝑐𝑞 = 2𝑎 − 2𝑎𝑞 ⇒ 𝑞(2𝑐 + 2𝑎) = 2𝑎 ⇒ 𝑞 =
2𝑎 2𝑐 𝑎+𝑐
𝑝 1−𝑝 𝑑
= ⇒ 2𝑏𝑝 = 2𝑑 − 2𝑑𝑝 ⇒ 𝑝(2𝑏 + 2𝑑) = 2𝑑 ⇒ 𝑝 =
2𝑑 2𝑏 𝑏+𝑑

Also, note that


𝑞 1−𝑞 1 1−𝑝 𝑝 1
[𝑅1 ] + [𝑅3 ] = + = , [𝑅2 ] + [𝑅4 ] = + =
2 2 2 2 2 2

Since the areas are equal, the number of triangles in them must also be equal.
𝑎+𝑐 =𝑏+𝑑
100-Ray Partitional
𝑎 + 𝑏 + 𝑐 + 𝑑 = 100 ⇒ 𝑎 + 𝑐 = 𝑏 + 𝑑 = 50

The coordinates of a 100-ray partitional point must be of the form:


𝑑 𝑎 𝑑 𝑎
(𝑝, 𝑞) = ( , ) = ( , ), 𝑑, 𝑎 ∈ ℤ
𝑏+𝑑 𝑎+𝑐 50 50
Since we want a point in the interior of the square, 0 < 𝑝, 𝑞 < 1, and we must have:
1 ≤ 𝑑 ≤ 49 ⇒ 49 𝑃𝑜𝑠𝑠𝑖𝑏𝑖𝑙𝑖𝑡𝑖𝑒𝑠
1 ≤ 𝑎 ≤ 49 ⇒ 49 𝑃𝑜𝑠𝑠𝑖𝑏𝑖𝑙𝑖𝑡𝑖𝑒𝑠
And, by the multiplication principle, we can count the number of values as:
49 × 49 = 492 = 2,401
60-Ray Partitional

2
The solution here uses the same logic as this video.

P a g e 91 | 95
Get all the files at: https://bit.ly/azizhandouts
Aziz Manva ([email protected])

𝑎 + 𝑏 + 𝑐 + 𝑑 = 60 ⇒ 𝑎 + 𝑐 = 𝑏 + 𝑑 = 30

The coordinates of a 60-ray partitional point must be of the form:


𝑑′ 𝑎′
(𝑝, 𝑞) = ( , ) , 𝑑′, 𝑎′ ∈ ℤ
30 30
100-Ray, but not 60-Ray Partitional
We now count and remove points which are both 100-ray partitional and 6-ray partitional.

For a number to be both 100-ray partitional, and 60-ray partitional, its 𝑥 coordinate must satisfy both of the
conditions below simultaneously:
𝑑 𝑑′
= ⇒ 3𝑑 = 5𝑑′

50 ⏟
30
100 𝑅𝑎𝑦 60 𝑅𝑎𝑦
This is a Diophantine equation, and has solutions:
(𝑑, 𝑑′ ) = (5𝑎, 3𝑎) = (5,3), (10,6), … (45,27) ⇒ 9 𝐶ℎ𝑜𝑖𝑐𝑒𝑠

Similarly, its y coordinate must satisfy both of the conditions below simultaneously:
𝑎 𝑎′
= ⇒ 3𝑎 = 5𝑎′

50 ⏟
30
100 𝑅𝑎𝑦 60 𝑅𝑎𝑦
This is a Diophantine equation, and has solutions:
(𝑎, 𝑎′ ) = (5𝑎, 3𝑎) = (5,3), (10,6), … (45,27) ⇒ 9 𝐶ℎ𝑜𝑖𝑐𝑒𝑠

And, again by the Multiplication Principle, we can count the number of values that are both 100-ray partitional
and 60-ray partitional as:

9×⏟ 9 = 81
𝑥 𝑦

Hence, the final answer is:


2401 − 81 = 2,320

E. Graphs

Example 4.36
Jorge's teacher asks him to plot all the ordered pairs (𝑤, 𝑙) of positive integers for which 𝑤 is the width and 𝑙 is
the length of a rectangle with area 12. What should his graph look like? (AMC 8 2006/10)

Example 4.37
Amanda draws five circles with radii 1, 2, 3, 4 and 5. Then for each circle she plots the point (𝐶, 𝐴), where 𝐶 is its
circumference and 𝐴 is its area. Which of the following could be her graph? (AMC 8 2007/16)

P a g e 92 | 95
Get all the files at: https://bit.ly/azizhandouts
Aziz Manva ([email protected])

F. Midpoint Formula

Example 4.38
Let points 𝐴 = (0,0,0), 𝐵 = (1,0,0), 𝐶 = (0,2,0), and 𝐷 = (0,0,3). Points 𝐸, 𝐹, 𝐺, and 𝐻 are midpoints of line
segments 𝐵𝐷, 𝐴𝐵, 𝐴𝐶, and 𝐷𝐶 respectively. What is the area of 𝐸𝐹𝐺𝐻? (AMC 10A 2012/21)

G. Counting

Example 4.39
For how many integers 𝑥 is the point (𝑥, −𝑥) inside or on the circle of radius 10 centered at (5,5)? (AMC 10B
2015/12)

H. Conics

Example 4.40
The vertices of an equilateral triangle lie on the hyperbola 𝑥𝑦 = 1, and a vertex of this hyperbola is the centroid
of the triangle. What is the square of the area of the triangle?(AMC 10B 2017/24)

Example 4.41
All three vertices of △ 𝐴𝐵𝐶 are lying on the parabola defined by 𝑦 = 𝑥 2, with 𝐴 at the origin and 𝐵𝐶 parallel to
the 𝑥-axis. The area of the triangle is 64. What is the length of 𝐵𝐶? All three vertices of △ 𝐴𝐵𝐶 are lying on the
parabola defined by 𝑦 = 𝑥 2, with 𝐴 at the origin and 𝐵𝐶 parallel to the 𝑥-axis. The area of the triangle is 64.
What is the length of 𝐵𝐶? (AMC 10B 2016/9)

I. Mix

Example 4.42

P a g e 93 | 95
Get all the files at: https://bit.ly/azizhandouts
Aziz Manva ([email protected])

Points 𝐴(11,9) and 𝐵(2, −3) are vertices of △ 𝐴𝐵𝐶 with 𝐴𝐵 = 𝐴𝐶. The altitude from 𝐴 meets the opposite side
at 𝐷(−1,3). What are the coordinates of point 𝐶? (AMC 10B 2017/8)

Example 4.43
Five unit squares are arranged in the coordinate plane as shown, with the lower left corner at the origin. The
slanted line, extending from (𝑐, 0) to (3,3), divides the entire region into two regions of equal area. What is 𝑐?
(AMC 10B 2009/17)

Example 4.44
Let 𝑆 be a set of points (𝑥, 𝑦) in the coordinate plane such that two of the three quantities 3, 𝑥 + 2, and 𝑦 − 4 are
equal and the third of the three quantities is no greater than this common value. Which of the following is a
correct description for 𝑆? (AMC 10A 2017/12)
(𝐴) a single point
(𝐵) two intersecting lines
(𝐶) three lines whose pairwise intersections are three distinct points
(𝐷) a triangle
(𝐸) three rays with a common endpoint

Example 4.45
Distinct points 𝑃, 𝑄, 𝑅, 𝑆 lie on the circle 𝑥 2 + 𝑦 2 = 25 and have integer coordinates. The distances 𝑃𝑄 and
𝑃𝑄
𝑅𝑆 are irrational numbers. What is the greatest possible value of the ratio 𝑅𝑆 ? (AMC 10A 2017/17)

Example 4.46
Two points 𝐵 and 𝐶 are in a plane. Let 𝑆 be the set of all points 𝐴 in the plane for which △ 𝐴𝐵𝐶 has area 1.
Which of the following describes 𝑆? (AMC 10B 2007/10)
(𝐴)two parallel lines (𝐵)a parabola (𝐶)a circle (𝐷)a line segment (𝐸)two points

Example 4.47
Points 𝐴(6,13) and 𝐵(12,11) lie on circle ω in the plane. Suppose that the tangent lines to ω at 𝐴 and 𝐵 intersect
at a point on the 𝑥-axis. What is the area of ω? (AMC 10B 2019/23)

Example 4.48

P a g e 94 | 95
Get all the files at: https://bit.ly/azizhandouts
Aziz Manva ([email protected])

Andy the Ant lives on a coordinate plane and is currently at (−20,20) facing east (that is, in the positive 𝑥-
direction). Andy moves 1 unit and then turns 90∘ degrees left. From there, Andy moves 2 units (north) and then
turns 90∘ degrees left. He then moves 3 units (west) and again turns 90∘ degrees left. Andy continues his
progress, increasing his distance each time by 1 unit and always turning left. What is the location of the point at
which Andy makes the 2020th left turn? (AMC 10B 2020/13)

Example 4.49
Points 𝐴 and 𝐵 are 10 units apart. Points 𝐵 and 𝐶 are 4 units apart. Points 𝐶 and 𝐷 are 3 units apart. If 𝐴 and 𝐷
are as close as possible, then the number of units between them is (AMC 8 1996/8)

Example 4.50
Points 𝐴, 𝐵, 𝐶, 𝐷, 𝐸 and 𝐹 lie, in that order, on 𝐴𝐹, dividing it into five segments, each of length 1. Point 𝐺 is not
on line 𝐴𝐹. Point 𝐻 lies on 𝐺𝐷, and point 𝐽 lies on 𝐺𝐹. The line segments 𝐻𝐶, 𝐽𝐸, and 𝐴𝐺 𝑎𝑟𝑒 𝑝𝑎𝑟𝑎𝑙𝑙𝑒𝑙. 𝐹𝑖𝑛𝑑 𝐻𝐶/
𝐽𝐸 (AMC 10A 2002/20)

Example 4.51
Points 𝐵 and 𝐶 lie on 𝐴𝐷. The length of 𝐴𝐵 is 4 times the length of 𝐵𝐷, and the length of 𝐴𝐶 is 9 times the length
of 𝐶𝐷. The length of 𝐵𝐶 is what fraction of the length of 𝐴𝐷? (AMC 10B 2008/6)

Example 4.52
Four distinct points are arranged on a plane so that the segments connecting them have lengths 𝑎, 𝑎, 𝑎, 𝑎, 2𝑎,
and 𝑏. What is the ratio of 𝑏 to 𝑎? (AMC 10B 2012/21)

53 Examples

P a g e 95 | 95

You might also like